You are on page 1of 112

Algebra

With Sessionwise Theory & Exercises


Algebra
With Sessionwise Theory & Exercises

Dr. SK Goyal
ARIHANT PRAKASHAN (Series), MEERUT
All Rights Reserved

© AUTHOR
No part of this publication may be re-produced, stored in a retrieval system or
by any means, electronic mechanical, photocopying, recording, scanning, web or
otherwise without the written permission of the publisher. Arihant has obtained
all the information in this book from the sources believed to be reliable and true.
However, Arihant or its editors or authors or illustrators don’t take any responsibility
for the absolute accuracy of any information published, and the damages or loss
suffered thereupon.
All disputes subject to Meerut (UP) jurisdiction only.

Administrative & Production Offices


Regd. Office
‘Ramchhaya’ 4577/15, Agarwal Road, Darya Ganj, New Delhi -110002
Tele: 011- 47630600, 43518550
Head Office
Kalindi, TP Nagar, Meerut (UP) - 250002 Tel: 0121-7156203, 7156204
Sales & Support Offices
Agra, Ahmedabad, Bengaluru, Bareilly, Chennai, Delhi, Guwahati,
Hyderabad, Jaipur, Jhansi, Kolkata, Lucknow, Nagpur & Pune.

ISBN : 978-93-25298-63-7

PO No : TXT-XX-XXXXXXX-X-XX
Published by Arihant Publications (India) Ltd.
For further information about the books published by Arihant, log on to
www.arihantbooks.com or e-mail at info@arihantbooks.com
Follow us on
PREFACE

‘‘THE ALGEBRAIC SUM OF ALL THE TRANSFORMATIONS OCCURRING IN A CYCLICAL


PROCESS CAN ONLY BE POSITIVE, OR, AS AN EXTREME CASE EQUAL TO NOTHING’’
MEANS IF YOU CONTINUOUSLY PUT YOUR EFFORTS ON AN ASPECT YOU HAVE VERY
GOOD CHANCE OF POSITIVE OUTCOME i.e. SUCCESS

It is a matter of great pride and honour for me to have received such an overwhelming response to the
previous editions of this book from the readers. In a way, this has inspired me to revise this book
thoroughly as per the changed pattern of JEE Main & Advanced. I have tried to make the contents more
relevant as per the needs of students, many topics have been re-written, a lot of new problems of new
types have been added in etcetc. All possible efforts are made to remove all the printing errors that had
crept in previous editions. The book is now in such a shape that the students would feel at ease while
going through the problems, which will in turn clear their concepts too.
A Summary of changes that have been made in Revised & Enlarged Edition
— Theory has been completely updated so as to accommodate all the changes made in JEE Syllabus & Pattern in
recent years.
— The most important point about this new edition is, now the whole text matter of each chapter has been
divided into small sessions with exercise in each session. In this way the reader will be able to go through the
whole chapter in a systematic way.
— Just after completion of theory, Solved Examples of all JEE types have been given, providing the students a
complete understanding of all the formats of JEE questions & the level of difficulty of questions generally
asked in JEE.
— Along with exercises given with each session, a complete cumulative exercises have been given at the end of
each chapter so as to give the students complete practice for JEE along with the assessment of knowledge
that they have gained with the study of the chapter.
— Last 13 Years questions asked in JEE Main & Adv, IIT-JEE & AIEEE have been covered in all the chapters.
However I have made the best efforts and put my all Algebra teaching experience in revising this book.
Still I am looking forward to get the valuable suggestions and criticism from my own fraternity i.e. the
fraternity of JEE teachers.
I would also like to motivate the students to send their suggestions or the changes that they want to be
incorporated in this book.
All the suggestions given by you all will be kept in prime focus at the time of next revision of the book.

Dr. SK Goyal
CONTENTS
1. COMPLEX NUMBERS 1-102
LEARNING PART Session 4
Session 1 — nth Root of Unity
— Integral Powers of Iota (i) — Vector Representation of Complex Numbers
— Switch System Theory — Geometrical Representation of Algebraic
Session 2 Operation on Complex Numbers
— Rotation Theorem (Coni Method)
— Definition of Complex Number
— Shifting the Origin in Case of Complex
— Conjugate Complex Numbers
Numbers
— Representation of a Complex Number in
— Inverse Points
Various Forms
— Dot and Cross Product
Session 3
— Use of Complex Numbers in Coordinate
— amp (z)– amp (–z)=± p, According as amp (z)
Geometry
is Positive or Negative
— Square Root of a Complex Number PRACTICE PART
— Solution of Complex Equations — JEE Type Examples
— De-Moivre’s Theorem — Chapter Exercises
— Cube Roots of Unity

2. THEORY OF EQUATIONS 103-206


LEARNING PART Session 4
Session 1 — Equations of Higher Degree
— Polynomial in One Variable — Rational Algebraic Inequalities
— Identity — Roots of Equation with the
— Linear Equation Help of Graphs
— Quadratic Equations Session 5
— Standard Quadratic Equation — Irrational Equations
Session 2 — Irrational Inequations
— Transformation of Quadratic Equations — Exponential Equations
— Condition for Common Roots — Exponential Inequations
— Logarithmic Equations
Session 3
— Logarithmic Inequations
— Quadratic Expression
— Wavy Curve Method PRACTICE PART
— Condition for Resolution into Linear Factors — JEE Type Examples
— Location of Roots (Interval in which Roots Lie) — Chapter Exercises
3. SEQUENCES AND SERIES 207-312
LEARNING PART Session 5
Session 1 — Mean
— Sequence Session 6
— Series — Arithmetico-Geometric
— Progression Series (AGS)
Session 2 — Sigma (S) Notation

— Arithmetic Progression — Natural Numbers


Session 7
Session 3
— Application to Problems of Maxima and
— Geometric Sequence or Geometric
Minima
Progression
Session 4 PRACTICE PART
— Harmonic Sequence or Harmonic Progression — JEE Type Examples
— Chapter Exercises

4. LOGARITHMS AND THEIR PROPERTIES 313-358


LEARNING PART Session 3
Session 1 — Properties of Monotonocity of Logarithm
— Definition — Graphs of Logarithmic Functions
— Characteristic and Mantissa
PRACTICE PART
Session 2 — JEE Type Examples
— Principle Properties of Logarithm — Chapter Exercises

5. PERMUTATIONS AND COMBINATIONS 359-436


LEARNING PART Session 5
Session 1 — Combinations from Identical Objects
— Fundamental Principle of Counting Session 6
— Factorial Notation — Arrangement in Groups
Session 2 — Multinomial Theorem
— Divisibility Test — Multiplying Synthetically
— Principle of Inclusion and Exclusion Session 7
— Permutation — Rank in a Dictionary
Session 3 — Gap Method
— Number of Permutations Under Certain [when particular objects are never together]
Conditions
— Circular Permutations
PRACTICE PART
— JEE Type Examples
— Restricted Circular Permutations
— Chapter Exercises
Session 4
— Combination
— Restricted Combinations
6. BINOMIAL THEOREM 437-518
LEARNING PART Session 4
Session 1 — Use of Complex Numbers in Binomial Theorem
— Binomial Theorem for Positive Integral Index — Multinomial Theorem
— Pascal’s Triangle — Use of Differentiation
— Use of Integration
Session 2
— When Each Term is Summation Contains the
— General Term
Product of Two Binomial Coefficients or
— Middle Terms
Square of Binomial Coefficients
— Greatest Term
— Binomial Inside Binomial
— Trinomial Expansion
— Sum of the Series
Session 3
— Two Important Theorems PRACTICE PART
— Divisibility Problems — JEE Type Examples
— Chapter Exercises

7. DETERMINANTS 519-604
LEARNING PART — System of Linear Equations
Session 1 — Cramer’s Rule
— Definition of Determinants — Nature of Solutions of System of Linear
— Expansion of Determinant Equations
— Sarrus Rule for Expansion — System of Homogeneous Linear Equations
— Window Rule for Expansion Session 4
Session 2 — Differentiation of Determinant
— Minors and Cofactors — Integration of a Determinant
— Use of Determinants in Coordinate Geometry — Walli’s Formula
— Properties of Determinants — Use of S in Determinant
Session 3 PRACTICE PART
— Examples on Largest Value of a — JEE Type Examples
Third Order Determinant — Chapter Exercises
— Multiplication of Two Determinants of
the Same Order

8. MATRICES 605-690
LEARNING PART Session 2
Session 1 — Transpose of a Matrix
— Definition — Symmetric Matrix
— Types of Matrices — Orthogonal Matrix
— Difference Between a Matrix and a — Complex Conjugate (or Conjugate) of a Matrix
Determinant — Hermitian Matrix
— Equal Matrices — Unitary Matrix
— Operations of Matrices — Determinant of a Matrix
— Various Kinds of Matrices — Singular and Non-Singular Matrices
Session 3 Session 4
— Adjoint of a Matrix — Solutions of Linear Simultaneous Equations
— Inverse of a Matrix Using Matrix Method
— Elementary Row Operations
PRACTICE PART
— Equivalent Matrices
— JEE Type Examples
— Matrix Polynomial
— Chapter Exercises
— Use of Mathematical Induction

9. PROBABILITY 691-760
LEARNING PART Session 4
Session 1 — Binomial Theorem on Probability
— Some Basic Definitions — Poisson Distribution
— Mathematical or Priori or Classical Definition — Expectation
of Probability
— Multinomial Theorem
— Odds in Favours and Odds Against the Event
— Uncountable Uniform Spaces
Session 2
— Some Important Symbols PRACTICE PART
— Conditional Probability — JEE Type Examples
Session 3 — Chapter Exercises
— Total Probability Theorem
— Baye’s Theorem or Inverse Probability

10. MATHEMATICAL INDUCTION 761-784


LEARNING PART PRACTICE PART
— Introduction — JEE Type Examples
— Statement — Chapter Exercises
— Mathematical Statement

11. SETS, RELATIONS AND FUNCTIONS 785-836


LEARNING PART Session 3
Session 1 — Definition of Function
— Definition of Sets — Domain, Codomain and Range
— Representation of a Set — Composition of Mapping
— Different Types of Sets — Equivalence Classes
— Laws and Theorems — Partition of Set
— Venn Diagrams (Euler-Venn Diagrams) — Congruences
Session 2
PRACTICE PART
— Ordered Pair
— JEE Type Examples
— Definition of Relation
— Chapter Exercises
— Ordered Relation
— Composition of Two Relations
SYLLABUS
numbers. Relation between AM and GM Sum upto n
JEE MAIN terms of special series: ∑ n, ∑ n2, ∑n3. Arithmetico -
Unit I Sets, Relations and Functions Geometric progression.
Sets and their representation, Union, intersection and Unit VIII Probability
complement of sets and their algebraic properties, Power Probability of an event, addition and multiplication
set, Relation, Types of relations, equivalence relations, theorems of probability, Baye’s theorem, probability
functions, one-one, into and onto functions, composition distribution of a random variate, Bernoulli and Binomial
of functions. distribution.
Unit II Complex Numbers
Complex numbers as ordered pairs of reals,
Representation of complex numbers in the form a+ib and JEE ADVANCED
their representation in a plane, Argand diagram, algebra
of complex numbers, modulus and argument (or Algebra
amplitude) of a complex number, square root of a Algebra of complex numbers, addition, multiplication,
complex number, triangle inequality. conjugation, polar representation, properties of modulus
and principal argument, triangle inequality, cube roots of
Unit III Matrices and Determinants unity, geometric interpretations.
Matrices, algebra of matrices, types of matrices,
determinants and matrices of order two and three. Quadratic equations with real coefficients, relations
Properties of determinants, evaluation of deter-minants, between roots and coefficients, formation of quadratic
area of triangles using determinants. Adjoint and equations with given roots, symmetric functions of roots.
evaluation of inverse of a square matrix using Arithmetic, geometric and harmonic progressions,
determinants and elementary transformations, Test of arithmetic, geometric and harmonic means, sums of finite
consistency and solution of simultaneous linear arithmetic and geometric progressions, infinite geometric
equations in two or three variables using determinants series, sums of squares and cubes of the first n natural
and matrices. numbers.
Unit IV Permutations and Combinations Logarithms and their Properties
Fundamental principle of counting, permutation as an Permutations and combinations, Binomial theorem for a
arrangement and combination as selection, Meaning of positive integral index, properties of binomial
P(n,r) and C (n,r), simple applications. coefficients.
Unit V Mathematical Induction Matrices as a rectangular array of real numbers, equality
Principle of Mathematical Induction and its simple of matrices, addition, multiplication by a scalar and
applications. product of matrices, transpose of a matrix, determinant of
a square matrix of order up to three, inverse of a square
Unit VI Binomial Theorem and its matrix of order up to three, properties of these matrix
Simple Applications operations, diagonal, symmetric and skew-symmetric
Binomial theorem for a positive integral index, general matrices and their properties, solutions of simultaneous
term and middle term, properties of Binomial coefficients linear equations in two or three variables.
and simple applications.
Addition and multiplication rules of probability,
Unit VII Sequences and Series conditional probability, independence of events,
Arithmetic and Geometric progressions, insertion of computation of probability of events using permutations
arithmetic, geometric means between two given and combinations.
CHAPTER

01
Complex Numbers
Learning Part
Session 1
● Integral Powers of Iota (i)

● Switch System Theory

Session 2
● Definition of Complex Number

● Conjugate Complex Numbers

● Representation of a Complex Number in Various Forms

Session 3
● amp ( z ) - amp ( - z ) = ± p , According as amp ( z ) is Positive or Negative

● Square Root of a Complex Number

● Solution of Complex Equations

● De-Moivre’s Theorem

● Cube Roots of Unity

Session 4
● nth Root of Unity

● Vector Representation of Complex Numbers

● Geometrical Representation of Algebraic Operation on Complex Numbers

● Rotation Theorem (Coni Method)

● Shifting the Origin in Case of Complex Numbers

● Inverse Points

● Dot and Cross Product

● Use of Complex Numbers in Coordinate Geometry

Practice Part
● JEE Type Examples
● Chapter Exercises

Arihant on Your Mobile !


Exercises with the #L
symbol can be practised on your mobile. See inside cover page to activate for free.
2 Textbook of Algebra

The square of any real number, whether positive, negative Remark


or zero, is always non-negative i.e. x 2 ³ 0 for all x Î R. - a = i a ,where a is positive quantity. Keeping this result in
mind, the following computation is correct
Therefore, there will be no real value of x , which when
- a - b = i a × i b = i 2 ab = - ab
squared, will give a negative number.
where, a and b are positive real numbers.
Thus, the equation x 2 + 1 = 0 is not satisfied for any real But the computation, - a - b = ( - a)( - b) = |a||b| is wrong.
value of x. ‘Euler’ was the first Mathematician to Because the property, a b = ab is valid only when atleast one
introduce the symbol i (read ‘Iota’) for the square root of of a and b is non-negative.
If a and b are both negative, then a b = - a b .
- 1 with the property i 2 = - 1. The theory of complex
number was later on developed by Gauss and Hamilton. y Example 1. Is the following computation correct?
According to Hamilton, ‘‘Imaginary number is that If not, give the correct computation.
number whose square is a negative number ’’. Hence, the
-2 -3 = ( -2) ( -3) = 6
equation x 2 + 1 = 0
Sol. No,
Þ x2 = -1 If a and b are both negative real numbers, then a b = - ab
or x = ± -1 Here, a = - 2 and b = - 3.
(in the sense of arithmetic, -1 has no meaning). \ -2 - 3 = - ( - 2) ( - 3) = - 6
Symbolically, -1 is denoted by i (the first letter of the
y Example 2. A student writes the formula
word ‘Imaginary ’).
ab = a b . Then, he substitutes a = - 1 and b = - 1
\ Solutions of x 2 + 1 = 0 are x = ± i. and finds 1 = - 1. Explain, where he is wrong.
Also, i is the unit of complex number, since i is present in Sol. Since, a and b are both negative, therefore ab ¹ a b .
every complex number. Generally, if a is positive quantity, Infact a and b are both negative, then we have a b = - ab .
then
y Example 3. Explain the fallacy
-a ´ -a = ( -1) ´ a ´ ( -1) ´ a
- 1 = i ´ i = -1 ´ -1 = ( -1) ´ ( -1) = 1 = 1.
= -1 ´ a ´ -1 ´ a
Sol. If a and b are both negative, then
=i a ´i a a b = - |a | |b |
= i2a = - a \ -1 ´ -1 = - | -1| | -1| = - 1

Session 1
Integral Powers of Iota (i ), Switch System Theory
Integral Powers of Iota ( i ) Þ n = 4q + r
(i) If the index of i is whole number, then When, 0 £ r £ 3
\ i n = i 4 q + r = (i 4 ) q (i ) r = (1) q × (i ) r = i r
i 0 = 1, i 1 = i , i 2 = ( -1 ) 2 = - 1,
In general, i 4n = 1, i 4n + 1 = i, i 4n + 2 = - 1,
i 3 = i × i 2 = - i, i 4 = (i 2 ) 2 = ( -1) 2 = 1
i 4n + 3 = - i for any whole number n.
To find the value of i n (n > 4 ) First divide n by 4.
(ii) If the index of i is a negative integer, then
Let q be the quotient and r be the remainder.
1 i i 1
i.e. 4 ) n (q i -1 = = = = - i, i -2 = = - 1,
i i 2 -1 i 2
- 4q
1 i 1 1
r i -3 = = = i, i -4 = = = 1, etc.
3 4
i i i4 1
Chap 01 Complex Numbers 3

2
y Example 4. Evaluate. æ1 + i ö æ 1 + i 2 + 2i ö
Sol. Q a 2 = ç ÷ =ç ÷
(i) i 1998 è 2 ø è 2 ø

(ii) i - 9999 æ 1 - 1 + 2i ö
=ç ÷ =i
è 2 ø
(iii) ( - -1 ) 4n +3 , n Î N
\ a1929 = a × a1928 = a × (a 2 )964 = a (i )964
Sol. (i) 1998 leaves remainder 2, when it is divided by 4.
i.e. 4 ) 1998 (499 = a (i )4 ´ 241 = a × (i 4 )241 = a
1996
2
y Example 7. Dividing f (z ) by z - i , where i = -1, we
\ i 1998 = i 2 = - 1 obtain the remainder i and dividing it by z + i , we get
the remainder 1 + i. Find the remainder upon the
Aliter
division of f (z ) by z 2 + 1.
i 2000
1
i 1998 = = -1= Sol. z - i = 0 Þ z = i
i - 12

(ii) 9999 leaves remainder 3, when it is divided by 4. Remainder, when f (z ) is divided by (z - i ) = i


i.e. 4 ) 9999 (2499 i.e. f (i ) = i K (i)
9996 and remainder, when f (z ) is divided by (z + 1) = 1 + i
3 i.e. f ( - i ) = 1 + i [Qz + i = 0 Þ z = - i ] K (ii)
2
- 9999 1 1 i i Since, z + 1 is a quadratic expression, therefore remainder
\ i = 9999 = 3 = 4 = = i
i i i 1 when f (z ) is divided by z 2 + 1, will be in general a linear
Aliter expression. Let g (z ) be the quotient and az + b (where a
1 i i and b are complex numbers) be the remainder, when f (z ) is
i - 9999 = 9999
=
=i 10000
= divided by z 2 + 1.
i i 1
(iii) 4n + 3 leaves remainder 3, when it is divided by 4. Then, f (z ) = (z 2 + 1) g ( z ) + az + b K (iii)
i.e., 4 ) 4n + 3 (n \ 2
f (i ) = (i + 1) g (i ) + ai + b = ai + b
4n or ai + b = i [from Eq. (i)] K (iv)
3 and 2
f ( - i ) = (i + 1) g ( - i ) - ai + b = - ai + b
\ i 4n + 3 = i 3 = - i or - ai + b = 1 + i [from Eq. (ii)] …(v)
4n + 3 4n + 3 4n + 3
Now, ( - -1 ) = (- i) = - (i ) From Eqs. (iv) and (v), we get
= - (- i) 1 i
b = + i and a =
=i 2 2
Aliter ( - -1 )4n + 3 = ( - i )4n + 3 = - i 4n + 3 Hence, required remainder = az + b
1 1
= - (i 4 )n × i 3 = iz + + i
2 2
= - (1)n ( - i ) = i

y Example 5. Find the value of 1 + i 2 + i 4 + i 6 + ... + i 2n , The Sum of Four Consecutive


where i = -1 and n Î N . Powers of i (Iota) is Zero
Sol. Q 1 + i 2 + i 4 + i 6 + ... + i 2 n = 1 - 1 + 1 - 1 + ... + ( - 1)n If n Î I and i = - 1, then
Case I If n is odd, then i n + i n + 1 + i n + 2 + i n + 3 = i n (1 + i + i 2 + i 3 )
1 + i 2 + i 4 + i 6 + ... + i 2 n = 1 - 1 + 1 - 1 + ... + 1 - 1 = 0 = i n (1 + i - 1 - i ) = 0
Case II If n is even, then
1 + i 2 + i 4 + i 6 + ... + i 2 n = 1 - 1 + 1 - 1 + ... + 1 = 1 Remark
m m- p + 1

1+ i
1. å f( r ) = å f ( r + p - 1)
y Example 6. If a = , where i = - 1, then find the r =p r =1

2 m m+ p + 1

value of a 1929
.
2. å f( r ) = å f ( r - p - 1)
r =-p r =1
4 Textbook of Algebra

13
y Example 8. Find the value of å (i n + i n + 1 )
Switch System Theory
n =1
( where ,i = - 1 )
(Finding Digit in the Unit’s Place)
13 13 13
Sol. Q å (i n + i n + 1 ) = å i n + å i n + 1 = ( i + 0) + ( i 2 + 0) We can determine the digit in the unit’s place in
n =1 n =1 n =1 a b , where a, b Î N . If last digit of a are 0, 1, 5 and 6, then
é 13 n 13 ù digits in the unit’s place of a b are 0, 1, 5 and 6
êQ å i = 0 and å i
n +1
=0 ú
=i -1 n=2 respectively, for all b Î N .
ê n=2 ú
êë(three sets of four consecutive powers of i )úû
100 Powers of 2
y Example 9. Find the value of å in !
2 1 , 2 2 , 2 3 , 2 4 , 2 5 , 2 6 , 2 7 , 2 8 , 2 9 , ... the digits in unit’s place
n=0
( where , i = - 1). of different powers of 2 are as follows :
Sol. n! is divisible by 4, " n ³ 4. 2, 4, 8, 6, 2, 4, 8, 6, 2,... (period being 4)
100 97 ­ ­ ­ ­ ­ ­ ­ ­ ­
\ å in ! = å i (n + 3 )! 1 2 3 0 1 2 3 0 1 ... (switch number)
n=4 n =1

= i 0 + i 0 + i 0 + ... 97 times = 97 …(i)


(The remainder when b is divided by 4, can be 1 or 2 or 3 or 0).
100 3 100
\ åi n!
= åi n!
+ åi n! Then, press the switch number and then we get the digit
n=0 n=0 n=4 in unit’s place of a b (just above the switch number) i.e.
= i 0 ! + i 1! + i 2 ! + i 3 ! + 97 [from Eq. (i)] ‘press the number and get the answer’.
1 1 2 6
= i + i + i + i + 97 = i + i - 1 - 1 + 97
y Example 12. What is the digit in the unit’s place of
= 95 + 2i
4n + 7 ( 5172)11327 ?
y Example 10. Find the value of å ir Sol. Here, last digit of a is 2.
r =1 The remainder when 11327 is divided by 4, is 3. Then,
( where ,i = - 1 ). press switch number 3 and then we get 8.
4n + 7 4n + 7 4n + 4 Hence, the digit in the unit’s place of (5172)11327 is 8.
Sol. å r 1
i =i +i +i + 2 3
å r
i =i -1-i + å i r +3

r =1 r =4 r =1
= - 1 + 0 [(n + 1) sets of four consecutive powers of i ] Powers of 3
= -1 3 1 , 3 2 , 3 3 , 3 4 , 3 5 , 3 6 , 3 7 , 3 8 , ... the digits in unit’s place of
y Example 11. Show that the polynomial different powers of 3 are as follows:
x 4 p + x 4q + 1 + x 4r + 2 + x 4 s + 3 is divisible by 3, 9, 7, 1, 3, 9, 7, 1, ... (period being 4)
x 3 + x 2 + x + 1, where p , q, r , s Î N . ­ ­ ­ ­ ­ ­ ­ ­
Sol. Let f ( x ) = x 4 p + x 4q + 1 + x 4r +2
+ x 4s + 3 1 2 3 0 1 2 3 0 ... (switch number)
and x 3 + x 2 + x + 1 = ( x 2 + 1) ( x + 1)
The remainder when b is divided by 4, can be 1 or 2 or 3
= ( x + i ) ( x - i ) ( x + 1), or 0. Now, press the switch number and get the unit’s
where i = -1 place digit ( just above).
Now, f (i ) = i 4 p + i 4q + 1 + i 4r +2
+ i 4s + 3 = 1 + i + i 2 + i 3 = 0
[sum of four consecutive powers of i is zero]
y Example 13. What is the digit in the unit’s place
f ( - i ) = ( - i )4 p + ( - i )4q + 1 + ( - i )4r + 2 + ( - i )4s + 3
of
= 1 + ( -i )1 + ( -i )2 + ( - i )3 = 1 - i - 1 + i = 0
(143) 86 ?
and f ( - 1) = ( - 1)4 p + ( - 1)4q + 1 + ( - 1)4r +2
+ ( - 1)4s + 3 Sol. Here, last digit of a is 3.
=1-1+1-1=0 The remainder when 86 is divided by 4, is 2.
Then, press switch number 2 and then we get 9.
Hence, by division theorem, f ( x ) is divisible by
x 3 + x 2 + x + 1. Hence, the digit in the unit’s place of (143)86 is 9.
Chap 01 Complex Numbers 5

Powers of 4 Powers of 8
1 2 3 4 5
4 , 4 , 4 , 4 , 4 ,... the digits in unit’s place of different 8 1 , 8 2 , 8 3 , 8 4 , 8 5 , 8 6 , 8 7 , 8 8 ,... the digits in unit’s place of
powers of 4 are as follows: different powers of 8 are as follows:
4, 6, 4, 6, 4, ... (period being 2) 8, 4, 2, 6, 8, 4, 2, 6, ... (period being 4)
­ ­ ­ ­ ­
­ ­ ­ ­ ­ ­ ­ ­
1 0 1 0 1 ... (switch number)
1 2 3 0 1 2 3 0 ... (switch number)
The remainder when b is divided by 2, can be 1 or 0. Now, The remainder when b is divided by 4, can be 1 or 2 or 3
press the switch number and get the unit’s place digit or 0.
( just above the switch number). Now, press the switch number and get the unit’s place
y Example 14. What is the digit in unit’s place of digit (just above the switch number).
(1354 ) 22222 ? y Example 16. What is the digit in the unit’s place of
Sol. Here, last digit of a is 4. (1008 ) 786 ?
The remainder when 22222 is divided by 2, is 0. Then, Sol. Here, last digit of a is 8.
press switch number 0 and then we get 6. The remainder when 786 is divided by 4, is 2. Then, press
Hence, the digit in the unit’s place of (1354 )22222 is 6. switch number 2 and then we get 4.
Hence, the digit in the unit’s place of (1008)786 is 4.

Powers of 7
Powers of 9
7 1 , 7 2 , 7 3 , 7 4 , 7 5 , 7 6 , 7 7 , 7 8 , ... the digits in unit’s place of
9 1 , 9 2 , 9 3 , 9 4 , 9 5 ,... the digits in unit’s place of different
different powers of 7 are as follows:
powers of 9 are as follows:
7, 9, 3, 1, 7, 9, 3, 1, ... (period being 4)
9, 1, 9, 1, 9, ... ( period being 2)
­ ­ ­ ­ ­ ­ ­ ­
­ ­ ­ ­ ­
1 2 3 0 1 2 3 0 ... (switch number)
1 0 1 0 1 ... (switch number)
(The remainder when b is divided by 4, can be 1 or 2 or 3 The remainder when b is divided by 2, can be 1 or 0.
or 0). Now, press the switch number and get the unit’s Now, press the switch number and get the unit’s place
place digit ( just above). digit (just above the switch number).
y Example 15. What is the digit in the unit’s place of y Example 17. What is the digit in the unit’s place of
(13057 ) 941120579 ? (2419 )111213 ?
Sol. Here, last digit of a is 7. Sol. Here, last digit of a is 9.
The remainder when 941120579 is divided by 4, is 3. Then, The remainder when 111213 is divided by 2, is 1. Then,
press switch number 3 and then we get 3. press switch number 1 and then we get 9.
Hence, the digit in the unit’s place of (13057 )941120579 is 3. Hence, the digit in the unit’s place of (2419 )111213 is 9.
6 Textbook of Algebra

#L Exercise for Session 1


1 If (1 + i )2 n + (1 - i )2 n = - 2n + 1 (where, i = - 1) for all those n, which are
(a) even (b) odd
(c) multiple of 3 (d) None of these

2 If i = - 1, the number of values of i n + i - n for different n Î I is


(a) 1 (b) 2
(c) 3 (d) 4

3 If a > 0 and b < 0, then a b is equal to (where, i = -1)


(a) - a × b (b) a × b i

(c) a × b (d) None of these

4 Consider the following statements.


S1 : - 6 = 2i ´ 3i = ( - 4) ´ ( - 9) ( where, i = - 1) S2 : ( - 4) ´ ( - 9) = ( - 4) ´ ( - 9)

S3 : ( - 4) ´ ( - 9) = 36 S4 : 36 = 6
Of these statements, the incorrect one is
(a) S1 only (b) S 2 only
(c) S 3 only (d) None of these
50

5 The value of S
n=0
i ( 2n + 1) ! (where, i = - 1) is

(a) i (b) 47 - i
(c) 48 + i (d) 0
1003

6 The value of S
r =-3
i r ( where i = - 1) is

(a) 1 (b) - 1
(c) i (d) - i

7 The digit in the unit’s place of (153)98 is


(a) 1 (b) 3
(c) 7 (d) 9

8 The digit in the unit’s place of (141414)12121 is


(a) 4 (b) 6
(c) 3 (d) 1
Session 2
Definition of Complex Number, Conjugate Complex
Numbers, Representation of a Complex Number in
Various Forms
Definition of Complex Number Algebraic Operations on
A number of the form a + ib, where a, b Î R and i = - 1, is Complex Numbers
called a complex number. It is denoted by z i.e. z = a + ib. Let two complex numbers be z 1 = a + ib and z 2 = c + id ,
A complex number may also be defined as an ordered pair where a, b, c , d Î R and i = - 1.
of real numbers; and may be denoted by the symbol (a, b). 1. Addition z 1 + z 2 = (a + ib ) + (c + id )
If we write z = (a, b ), then a is called the real part and b is
= (a + c ) + i (b + d )
the imaginary part of the complex number z and may be
denoted by Re (z ) and Im (z), respectively i.e., a = Re (z ) 2. Subtraction z 1 - z 2 = (a + ib ) - (c + id )
and b = Im (z ). = (a - c ) + i (b - d )
Two complex numbers are said to be equal, if and only if 3. Multiplication z 1 × z 2 = (a + ib ) × (c + id )
their real parts and imaginary parts are separately equal. = ac + iad + ibc + i 2 bd
Thus, a + ib = c + id
= ac + i (ad + bc ) - bd
Û a = c and b = d
= (ac - bd ) + i (ad + bc )
where, a, b, c , d Î R and i = - 1.
z 1 (a + ib ) (c - id )
4. Division = ×
i.e. z1 = z2 z 2 (c + id ) (c - id )
Û Re (z 1 ) = Re (z 2 ) and Im (z 1 ) = Im (z 2 )
[multiplying numerator and denominator by c - id
Important Properties of Complex Numbers where atleast one of c and d is non-zero]
1. The complex numbers do not possess the property of order, ac - iad + ibc - i 2 bd ac + i (bc - ad ) + bd
= =
i.e., ( a + ib) > or < ( c + id ) is not defined. For example,
(c ) 2 - (id ) 2 c 2 - i 2d 2
9 + 6 i > 3 + 2i makes no sense.
2. A real number a can be written as a + i × 0. Therefore, every (ac + bd ) + i (bc - ad ) (ac + bd ) (bc - ad )
= = +i
real number can be considered as a complex number, whose 2 2 2 2
c +d (c + d ) (c 2 + d 2 )
imaginary part is zero. Thus, the set of real numbers (R) is a
proper subset of the complex numbers ( C ) i.e. R Ì C. Hence,
the complex number system is N Ì W Ì I Ì Q Ì R Ì C Remark
3. A complex number z is said to be purely real, if Im ( z ) = 0; and 1+ i 1- i
= i and = - i, where i = -1.
is said to be purely imaginary, if Re ( z ) = 0. The complex 1- i 1+ i
number 0 = 0 + i × 0 is both purely real and purely imaginary.
4. In real number system, a2 + b2 = 0 Þ a = 0 = b. Properties of Algebraic Operations
But if z 1 and z 2 are complex numbers, then z 12 + z 22 = 0 on Complex Numbers
does not imply z1 = z2 = 0.
For example, z 1 = 1 + i and z 2 = 1 - i
Let z 1 , z 2 and z 3 be any three complex numbers.
Here, z 1 ¹ 0, z 2 ¹ 0
Then, their algebraic operations satisfy the following
But z 12 + z 22 = ( 1 + i ) 2 + ( 1 - i ) 2 = 1 + i 2 + 2i + 1 + i 2 - 2i
properties :
= 2 + 2i 2 = 2 - 2 = 0 Properties of Addition of Complex Numbers
However, if product of two complex numbers is zero, then
(i) Closure law z 1 + z 2 is a complex number.
atleast one of them must be zero, same as in case of real
numbers. (ii) Commutative law z 1 + z 2 = z 2 + z 1
If z 1z 2 = 0, then z 1 = 0, z 2 ¹ 0 or z 1 ¹ 0, z 2 = 0 (iii) Associative law (z 1 + z 2 ) + z 3 = z 1 + (z 2 + z 3 )
or z 1 = 0, z 2 = 0
8 Textbook of Algebra

(iv) Additive identity z + 0 = z = 0 + z , then 0 is called


the additive identity.
Properties of Conjugate
(v) Additive inverse - z is called the additive inverse of Complex Numbers
z, i.e. z + ( - z ) = 0. Let z, z 1 and z 2 be complex numbers. Then,
Properties of Multiplication (i) (z ) = z
of Complex Numbers (ii) z +z = 2 Re (z )
(i) Closure law z 1 × z 2 is a complex number. (iii) z -z = 2 Im (z )
(ii) Commutative law z 1 × z 2 = z 2 × z 1 (iv) z +z = 0 Þ z = - z Þ z is purely imaginary.
(iii) Associative law (z 1 × z 2 ) z 3 = z 1 (z 2 × z 3 ) (v) z -z = 0 Þ z = z Þ z is purely real.
(iv) Multiplicative identity z × 1 = z = 1 × z , then 1 is (vi) z 1 ± z 2 = z 1 ± z 2 Ingeneral,
called the multiplicative identity.
z 1 ± z 2 ± z 3 ± ... ± z n = z 1 ± z 2 ± z 3 ± ... ± z n
(v) Multiplicative inverse If z is a non-zero complex
1 (vii) z 1 × z 2 = z 1 × z 2
number, then is called the multiplicative inverse
z In general, z 1 × z 2 × z 3 ... z n = z 1 × z 2 × z 3 ... z n
1 1
of z i.e. z. = 1 = × z
z z æz ö z
(viii) ç 1 ÷ = 1 , z 2 ¹ 0
(vi) Multiplication is distributive with respect to èz2 ø z2
addition z 1 (z 2 + z 3 ) = z 1 z 2 + z 1 z 3
(ix) z n = (z ) n
(x) z 1 z 2 + z 1 z 2 = 2 Re (z 1 z 2 ) = 2 Re (z 1 z 2 )
Conjugate Complex Numbers (xi) If z = f (z 1 , z 2 ), then z =f (z 1 , z 2 )
The complex numbers z = (a, b ) = a + ib and
z = (a, - b ) = a - ib, where a and b are real numbers, x-3 y -3
y Example 18. If + = i , where x , y ÎR and
i = -1 and b ¹ 0, are said to be complex conjugate of each 3+ i 3-i
other (here, the complex conjugate is obtained by just i = - 1, find the values of x and y.
changing the sign of i). x -3 y -3
Sol. Q + =i
Note that, sum = (a + ib ) + (a - ib ) = 2a, which is real. 3+i 3 -i
And product = (a + ib ) (a - ib ) = a 2 - (ib ) 2 Þ ( x - 3) ( 3 - i ) + ( y - 3 ) ( 3 + i ) = i ( 3 + i ) ( 3 - i )
2 2 2 2 2 Þ ( 3x - xi - 9 + 3i ) + ( 3y + yi - 9 - 3i ) = 10i
= a - i b = a - ( -1) b
Þ (3x + 3y - 18) + i (y - x ) = 10i
= a 2 + b 2 , which is real. On comparing real and imaginary parts, we get
Geometrically, z is the mirror image of z along real axis on 3x + 3y - 18 = 0
argand plane. Þ x +y = 6 …(i)
and y - x = 10 ...(ii)
Remark On solving Eqs. (i) and (ii), we get
Let z = - a - ib, a > 0, b > 0 = ( - a, - b) (III quadrant ) x = - 2, y = 8
Imaginary axis
P(z) y Example 19. If (a + ib ) 5 = p + iq, where i = - 1,
b prove that (b + ia ) 5 = q + ip .
θ a
Real
O θ axis Sol. Q (a + ib ) 5 = p + iq
b
\ (a + ib ) 5 = p + iq Þ (a - ib ) 5 = ( p - iq )
Q(z)
Þ ( - i 2a - ib ) 5 = ( -i 2 p - iq ) [Qi 2 = - 1]
Then, z = - a + ib = ( - a, b) ( II quadrant). Now,
(i) If z lies in I quadrant, then z lies in IV quadrant and
Þ ( - i )5 (b + ia ) 5 = ( - i ) (q + ip )
vice-versa. Þ ( - i ) (b + ia ) 5 = ( - i ) (q + ip )
(ii) If z lies in II quadrant, then z lies in III quadrant and
vice-versa. \ (b + ia ) 5 = (q + ip )
Chap 01 Complex Numbers 9

y Example 20. Find the least positive integral value of y Example 23. Find real values of x and y for which
n
æ 1-i ö the complex numbers - 3 + i x 2 y and x 2 + y + 4i ,
n, for which ç ÷ , where i = -1, is purely
è 1+ i ø where i = - 1, are conjugate to each other.
imaginary with positive imaginary part. Sol. Given, - 3 + ix 2y = x 2
+ y + 4i
n n n 2 2
æ1 - i ö æ1 - i 1 - i ö æ 1 + i - 2i ö æ 1 - 1 - 2i ö
2 n
Þ - 3 - ix y = x + y + 4i
Sol. ç ÷ =ç ´ ÷ =ç ÷ =ç ÷
è1 +i ø è1+ i 1 -i ø è 2 ø è 2 ø On comparing real and imaginary parts, we get
n
= ( - i ) = Imaginary x 2 +y = -3 …(i)
2
Þ n = 1, 3, 5, ... for positive imaginary part n = 3. and -x y=4 …(ii)
4
y Example 21. If the multiplicative inverse of a From Eq. (ii), we get x 2 = -
y
complex number is ( 3 + 4i ) / 19, where i = - 1, find 4 é 4 ù
complex number. Then, - + y = - 3 ê putting x 2 = - in Eq. (i)ú
y ë y û
Sol. Let z be the complex number.
æ 3 + 4i ö y 2 + 3y - 4 = 0 Þ (y + 4 ) (y - 1) = 0
Then, z × ç ÷ =1 \ y = - 4, 1
è 19 ø
For y = - 4 , x2 = 1 Þ x = ± 1
19 ( 3 - 4i )
or z= ´ For y = 1, x 2 = - 4 [impossible]
( 3 + 4i ) ( 3 - 4i )
\ x = ± 1, y = - 4
19 ( 3 - 4i )
= = ( 3 - 4i )
19 y Example 24. If x = - 5 + 2 - 4, find the value of
3 + 2 i sin q x 4 + 9 x 3 + 35x 2 - x + 4.
y Example 22. Find real q, such that ,
1 - 2 i sin q Sol. Since, x = - 5 + 2 - 4 Þ x + 5 = 4i
where i = -1, is Þ ( x + 5)2 = ( 4i )2 Þ x 2 + 10x + 25 = - 16
(i) purely real. (ii) purely imaginary. \ x 2 + 10x + 41 = 0 …(i)
3 + 2i sin q Now,
Sol. Let z =
1 - 2i sin q x 2 + 10x + 41 x 4
+ 9 x 3 + 35x 2
-x+4 x 2
-x+4
4 3 2
On multiplying numerator and denominator by conjugate x + 10x + 41x
of denominator, - - -
- x - 6x 2 - x + 4
3
(3 + 2i sin q ) (1 + 2i sin q ) (3 - 4 sin 2 q ) + 8i sin q
z= = 3 2
(1 - 2i sin q ) (1 + 2i sin q ) (1 + 4 sin 2 q ) -x - 10x - 41x
+ + +
(3 - 4 sin 2 q ) (8 sin q ) 4x 2
+ 40x + 4
= 2
+i 2
(1 + 4 sin q ) (1 + 4 sin q ) 4x 2
+ 40x + 164
(i) For purely real, Im(z ) = 0 - - -
- 160
8 sin q
Þ = 0 or sin q = 0 \ x 4 + 9x 3
+ 35x 2
-x+4
1 + 4 sin 2 q
= ( x 2 + 10x + 41) ( x 2 - x + 4 ) - 160
\ q = n p, n Î I
= 0 - 160 = - 160 [from Eq. (i)]
(ii) For purely imaginary, Re (z ) = 0
(3 - 4 sin 2 q ) y Example 25. Let z be a complex number satisfying
Þ = 0 or 3 - 4 sin 2 q = 0
(1 + 4 sin 2 q) the equation z 2 - ( 3 + i ) z + l + 2 i = 0, where l ÎR and
3 æ 3ö
2

2 i = -1. Suppose the equation has a real root, find the
æ
or sin 2 q = = ç ÷ = çsin ÷ non-real root.
4 è 2 ø è 3ø
Sol. Let a be the real root. Then,
p
\ q = np ± , n ÎI a 2 - (3 + i ) a + l + 2i = 0
3
10 Textbook of Algebra

Þ (a 2 - 3 a + l ) + i (2 - a ) = 0 Argument of z will be q, p - q , p + q and 2p - q


On comparing real and imaginary parts, we get according as the point z lies in I, II, III and IV
y
a 2 - 3a + l = 0 …(i) quadrants respectively, where q = tan - 1 .
Þ 2-a =0 …(ii) x
From Eq. (ii), a = 2
y Example 26. Find the arguments of z 1 = 5 + 5i ,
Let other root be b.
Then, a + b = 3 + i Þ 2+ b = 3 + i z 2 = - 4 + 4 i , z 3 = - 3 - 3 i and z 4 = 2 - 2i ,
\ b =1+i where i = - 1.
Hence, the non-real root is 1 + i. Sol. Since, z 1, z 2 , z 3 and z 4 lies in I, II, III and IV quadrants
respectively. The arguments are given by
5
arg (z1 ) = tan - 1 = tan - 1 1 = p / 4
Representation of a Complex 5

Number in Various Forms arg (z 2 ) = p - tan - 1


4
-4
p 3p
= p - tan -1 1 = p - =
4 4
-3 p 5p
Cartesian Form arg (z 3 ) = p + tan - 1
-3
= p + tan -1 1 = p + =
4 4
(Geometrical Representation) -2
and arg (z 4 ) = 2p - tan - 1
Every complex number z = x + iy , where x , y Î R and 2
i = - 1, can be represented by a point in the cartesian p 7p
= 2p - tan -1 1 = 2p - =
plane known as complex plane (Argand plane) by the 4 4
ordered pair ( x , y ).
Principal Value of the Argument
The value q of the argument which satisfies the inequality
Modulus and Argument of a -p < q £ p is called the principal value of the argument.
Complex Number If z = x + iy = ( x , y ), " x , y Î R and i = - 1, then
Let z = x + iy = ( x , y ) for all x , y Î R and i = - 1. æy ö
arg(z ) = tan - 1 ç ÷ always gives the principal value. It
èxø
Imaginary axis
P (x, y) depends on the quadrant in which the point ( x , y ) lies.

Y Imaginary
r
y axis

θ P (x, y)
Real axis
O x
y
The length OP is called modulus of the complex number z
denoted by z , θ
X′ X
i.e. OP = r = z = ( x 2 + y 2 ) O x
Real axis
Y′
and if ( x , y ) ¹ (0, 0 ), then q is called the argument or
amplitude of z, (i) ( x , y ) Î first quadrant x > 0, y > 0.
æy ö æy ö
i.e. q = tan - 1 ç ÷ [angle made by OP with positive X-axis] The principal value of arg (z ) = q = tan - 1 ç ÷
èxø èxø
or arg (z ) = tan - 1 (y / x ) It is an acute angle and positive.
(ii) ( x , y ) Î second quadrant x < 0, y > 0.
Also, argument of a complex number is not unique, since
The principal value of arg (z ) = q
if q is a value of the argument, so also is 2np + q, where
n Î I . But usually, we take only that value for which æ y ö
= p - tan - 1 çç ÷
÷
0 £ q < 2 p. Any two arguments of a complex number differ è x ø
by 2np.
Chap 01 Complex Numbers 11

Y
Imaginary or tan - 1 1, p - tan - 1 1, - p + tan - 1 1, - tan - 1 1
(x, y) axis p p p p p 3p 3p p
or ,p- ,-p+ , - or , , - ,-
4 4 4 4 4 4 4 4
y Hence, the principal values of the arguments of z1, z 2 , z 3
θ
p 3p 3p p
X′ x X and z 4 are , , - , - , respectively.
O 4 4 4 4
Real axis

Y′ Remark
1. Unless otherwise stated, amp z implies principal value of the
It is an obtuse angle and positive. argument.
(iii) ( x , y ) Î third quadrant x < 0, y < 0. 2. Argument of the complex number 0 is not defined.
æy ö 3. If z1 = z2 Û z1 = z2 and arg ( z1 ) = arg ( z2 ).
The principal value of arg (z ) = q = - p + tan -1 ç ÷ 4. If arg ( z ) = p /2 or - p /2, z is purely imaginary.
èxø
Y 5. If arg ( z ) = 0 or p, z is purely real.
Imaginary
axis y Example 28. Find the argument and the principal
value of the argument of the complex number
X′
x O
X 2+ i
θ Real axis z= , where i = - 1.
y 4i + (1 + i ) 2
2+i 2+i 2+i 1 1
Sol. Since, z = 2
= 2
= = - i
(x, y) 4i + ( 1 + i ) 4i + 1 + i + 2i 6i 6 3
Y′
\ z lies in IV quadrant.
It is an obtuse angle and negative.
1
(iv) ( x , y ) Î fourth quadrant x > 0, y < 0. -
-1 3 = tan - 1 2
Here, q = tan
The principal value of arg (z ) = q 1
æ y ö 6
= - tan - 1 çç ÷
÷ \ arg (z ) = 2p - q = 2p - tan -1 2
è x ø
Hence, principal value of arg (z ) = - q = - tan -1 2.
Y
Imaginary
axis Properties of Modulus
x (i) z ³ 0 Þ z = 0, iff z = 0 and z > 0, iff z ¹ 0
X′ X
O θ Real axis (ii) - z £ Re (z ) £ z and - z £ Im (z ) £ z
y
(iii) z = z = - z = - z
2
(x, y) (iv) zz = z
Y′
(v) z 1 z 2 = z 1 z 2
It is an acute angle and negative. In general, z 1 z 2 z 3 ... z n = z 1 z 2 z 3 ... z n
y Example 27. Find the principal values of the z z1
(vi) 1 = (z 2 ¹ 0 )
arguments of z 1 = 2 + 2i , z 2 = - 3 + 3i , z 3 = - 4 - 4i z2 z2
and z 4 = 5 - 5i , where i = -1.
(vii) z 1 ± z 2 £ z 1 + z 2
Sol. Since, z 1, z 2 , z 3 and z 4 lies in I, II, III and IV quadrants In general, z 1 ± z 2 ± z 3 ± ... ± z n £ z 1 + z 2
respectively. The principal values of the arguments are
+ z 3 + ... + z n
given by
æ 3 ö æ- 4ö (viii) z 1 ± z 2 ³ z 1 - z 2
æ2ö
tan -1 ç ÷ , p - tan -1 ç ÷ , - p + tan -1 ç ÷,
è2ø è -3 ø è- 4ø n
(ix) z n = z
æ -5 ö
- tan -1 ç ÷ (x) z1 - z2 £ z1 + z2 £ z1 + z2
è 5 ø
12 Textbook of Algebra

Thus, z 1 | + | z 2 is the greatest possible value of é 1ù


\ sin q i Î ê0, ú ,
z 1 + z 2 and | z 1 | - | z 2 | is the least possible value of ë 2û
z1 + z2 . 1
i.e. 0 £ sinq i £
2 2 2 2
(xi) z 1 ± z 2 = (z 1 ± z 2 ) (z 1 ± z 2 ) = z 1 + z2
\ Inequality Eq. (i) becomes,
± (z 1 z 2 + z 1 z 2 ) 1 4 1 3 1 2 1 1
2 2 2£ z + z + z + z +
or z 1 + z2 ± 2 Re (z 1 z 2 ) 2 2 2 2 2
4 3 2
Þ 3£ z + z + z + z
(xii) z 1 z 2 + z 1 z 2 = 2 z 1 z 2 cos(q 1 - q 2 ), where
q 1 = arg (z 1 ) and q 2 = arg (z 2 ) Þ 3£ z + z + z
2 3
+ z
4
< z + z
2

2 2 z 3 4
(xiii) | z 1 + z 2 |2 = z 1 + z 2 Û 1 is purely imaginary. + z + z +... + ¥
z2 2 3 4
2 2 2 2 Þ 3< z + z + z + z + ... + ¥
(xiv) z 1 + z 2 + z1 - z2 = 2 {z1 + z2 }
z
2 2 2 2 Þ 3< [here, | z | < 1]
(xv) az 1 - bz 2 + bz 1 + az 2 2
= (a + b ) ( z 12
+ z 2 ), 1- z
where a, b Î R Þ 3- 3 z < z Þ 3 < 4 z
3
(xvi) Unimodular i.e., unit modulus \ z >
4
If z is unimodular, then z = 1. In case of unimodular, 3
Hence, < z <1 [Q | z | < 1]
let z = cos q + i sin q, q Î R and i = -1. 4

Remark y Example 30. If z - 2 + i £ 2, find the greatest


1. If f ( z ) is unimodular, then f ( z ) = 1and let
f ( z ) = cos q + i sin q, q ÎR and i = -1. and least values of | z | , where i = -1.
z Sol. Given that, z - 2 + i £ 2 …(i)
2. is always a unimodular complex number, if z ¹ 0.
z
Q z-2+i ³ z - 2-i [by property (x)]
(xvii) The multiplicative inverse of a non-zero complex
number z is same as its reciprocal and is given by \ z-2+i ³ z - 5 …(ii)
1 z z From Eqs. (i) and (ii), we get
= = .
z zz 2
z z - 5 £ z-2+i £2

\ z - 5 £2
y Example 29. If q i Î [0, p / 6], i = 1 , 2, 3, 4, 5 and
sin q 1 z 4 + sin q 2 z 3 + sin q 3 z 2 + sin q 4 z Þ -2£ z - 5 £2
3 Þ 5 -2£ z £ 5 +2
+ sin q 5 = 2, show that < z < 1.
4 Hence, greatest value of z is 5 + 2 and least value of z
Sol. Given that,
is 5 - 2.
sin q 1 z 4 + sin q 2 z 3 + sin q 3 z 2 + sin q 4 z + sin q 5 = 2
or 2 = sin q 1 z 4 + sin q 2 z 3 + sin q 3 z 2 + sin q 4 z + sin q 5
y Example 31. If z is any complex number such
2 £ sin q 1 z 4 + sin q 2 z 3 + sin q 3 z 2 that z + 4 £ 3, find the greatest value of z + 1 .
+ sin q 4 z + sin q 5 [by property (vii)]
Sol. Q z + 1 = (z + 4 ) - 3
Þ 2 £ sin q 1 z 4 + sin q 2 z 3 + sin q 3 z 2
= ( z + 4 ) + ( - 3) £ z + 4 + - 3
+ sin q 4 z + sin q 5 [by property (v)]
4 3 2 = z + 4 +3
Þ 2 £ sin q 1 z + sin q 2 z + sin q 3 z
£3+3=6 [Q z + 4 £ 3 ]
+ sin q 4 z + sin q 5 [by property (ix)] …(i)
But given, q i Î[0, p / 6] \ z +1 £6
Hence, the greatest value of z + 1 is 6.
Chap 01 Complex Numbers 13

2
y Example 32. If z 1 = 1, z 2 = 2, z 3 = 3 and Þ z1 - 2z 2
2
= 2 - z1z 2
9z 1z 2 + 4z 3z 1 + z 2 z 3 = 6, find the value of Þ (z 1 - 2z 2 ) ( z1 - 2 z 2 ) = ( 2 - z1z 2 ) ( 2 - z1z 2 )
z1 +z 2 +z 3 .
[by property (iv)]
2
Sol. Q z1 = 1 Þ z1 =1 Þ ( z1 - 2z 2 ) ( z1 - 2z 2 ) = ( 2 - z1z 2 ) ( 2 - z1z 2 )
1 Þ z1z1 - 2z1z 2 - 2z 2 z1 + 4z 2 z 2
Þ z1 z1 = 1 Þ = z1
z1 = 4 - 2z1z 2 - 2z1z 2 + z1z1z 2 z 2
2 2 2 2 2
z2 = 2 Þ z2 = 4 Þ z2 z2 = 4 Þ z1 + 4 z2 = 4 + z1 z2
2 2 2 2
Þ
4
= z 2 and z 3 = 3 Þ | z 3 | 2 = 9 Þ z1 - z1 × z2 + 4 z2 - 4 =0
z2 æç z - 4 ö÷ø æçè1 - z 2 ö÷ø = 0
2 2
Þ è 1
9
Þ z3z3 =9 Þ =z3
z3 But z2 ¹ 1 [given]
2
and given 9z 1z 2 + 4z 3z 1 + z 2 z 3 = 6 \ z1 =4
Hence, z1 = 2
9 4 1
Þ z1 z 2 z 3 + + =6
z3 z 2 z1
Þ z1 z2 z3 z 3 + z 2 + z1 = 6 Properties of Arguments
é 1 4 9 ù (i) arg (z 1 z 2 ) = arg (z 1 ) + arg (z 2 ) + 2kp, k Î I
êQ z = z 1, z = z 2 and z = z 3 ú In general, arg (z 1 z 2 z 3 ... z n )
ë 1 2 3 û
Þ 1 × 2 × 3 z1 + z 2 + z 3 = 6 = arg (z 1 ) + arg (z 2 ) + arg (z 3 ) +... + arg (z n ) + 2kp,
k Î I.
\ z1 + z 2 + z 3 = 1 [Q | z | = | z | ]
æz ö
(ii) arg ç 1 ÷ = arg (z 1 ) - arg (z 2 ) + 2kp, k Î I
èz2 ø
y Example 33. Prove that
1 1 æz ö
z 1 + z 2 = (z 1 + z 2 ) + z 1 z 2 + (z 1 + z 2 ) - z 1 z 2 . (iii) arg ç ÷ = 2 arg (z ) + 2kp, k Î I
2 2 èz ø

Sol. RHS =
1 1
(z1 + z 2 ) + z1z 2 + (z1 + z 2 ) - z1z 2 (iv) arg (z n ) = n. arg (z ) + 2kp, k Î I , where proper value
2 2 of k must be chosen, so that RHS lies in ( -p, p ].
z1 + z 2 + 2 z1z 2 z + z 2 - 2 z1z 2 æz ö æz ö
= + 1 (v) If arg ç 2 ÷ = q, then arg ç 1 ÷ = 2np - q, where n Î I .
2 2 èz1 ø èz2 ø
1 2 2
(vi) arg (z ) = - arg (z)
= { z 1 + z2 + z 1 - z2 }
2
1 2 2 17 p 7p
= . 2 { z1 + z 2 } [ by property (xiv)] y Example 35. If arg (z 1 ) = and arg (z 2 ) = , find
2 18 18
= z1 + z 2 = LHS the principal argument of z 1z 2 and (z 1 / z 2 ).
Sol. arg (z1z 2 ) = arg (z1 ) + arg (z 2 ) + 2kp
y Example 34. z 1 and z 2 are two complex numbers, 17 p 7 p
= + + 2kp
z 1 - 2z 2 18 18
such that is unimodular, while z 2 is not 4p
2 - z 1 ×z 2 = + 2kp
3
unimodular. Find | z 1 |. 4p 2p
z1 - 2z 2 = - 2p = - [for k = - 1]
Sol. Here, =1 3 3
2 - z1z 2 æ z1 ö
and arg ç ÷ = arg (z1 ) - arg (z 2 ) + 2kp
z1 - 2z 2 èz2 ø
Þ =1 [by property (vi)] 17 p 7 p 10p
2 - z1z 2 = - + 2kp = + 2kp
18 18 18
Þ z1 - 2z 2 = 2 - z1z 2 5p 5p
= +0= [for k = 0]
9 9
14 Textbook of Algebra

y Example 36. If z 1 and z 2 are conjugate to each (b) Trigonometric or Polar or


other, find the principal argument of ( - z 1z 2 ).
Modulus Argument Form of a
Sol. Qz1 and z 2 are conjugate to each other i.e., z 2 = z1, there-
fore, z1z 2 = z1z1 = z1
2 Complex Number
2
Let z = x + iy , where x , y Î R and i = -1, z is represented
\arg ( - z1 z 2 ) = arg ( - z1 ) = arg [negative real number] by P ( x , y ) in the argand plane.
=p
Y
y Example 37. Let z be any non-zero complex

Imaginary axis
number, then find the value of arg (z ) + arg (z ). P (x, y)

Sol. arg (z ) + arg (z ) = arg (zz )


2 y
= arg ( z ) = arg [positive real number]
θ
=0 O x X
M Real axis

(a) Mixed Properties of Modulus By geometrical representation,


OP = ( x 2 + y 2 ) = z
and Arguments
(i) z 1 + z 2 = z 1 + z 2 Û arg (z 1 ) = arg (z 2 ) ÐPOM = q = arg (z )
(ii) z 1 + z 2 = z 1 - z 2 Û arg (z 1 ) - arg (z 2 ) = p In DOPM, x = OP cos ( Ð POM ) = z cos (arg z )
Proof (i) Let arg (z 1 ) = q and arg (z 2 ) = f and y = OP sin ( Ð POM ) = z sin (arg z )
Q z = x + iy
\ z1 + z2 = z1 + z2
\ z = z (cos (arg z ) + i sin (arg z ) )
On squaring both sides, we get
2 2 2
or z = r (cos q + i sin q )
z1 + z2 = z1 + z2 + 2 z1 z2 z = r (cos q - i sin q )
Þ z1
2
+ z2
2
+ 2 z 1 z 2 cos (q - f) where, r = z and q = principal value of arg (z ).
2 2
= z1 + z2 + 2 z1 z2 Remark
1. cos q + i sin q is also written as CiS q.
Þ cos (q - f) = 1 2. Remember
\ q - f = 0 or q = f 1 = cos 0 + i sin 0 Þ - 1 = cos p + i sin p
\ arg (z 1 ) = arg (z 2 ) p p p p
i = cos + i sin Þ - i = cos - i sin
2 2 2 2
(ii) Q | z 1 + z 2 | = | z 1 | - | z 2 |
On squaring both sides, we get 1 i 3
2 2 2
y Example 38. Write the polar form of - -
z1 + z2 = z1 + z2 - 2 z1 z2 2 2
2 2
(where, i = - 1).
Þ z1 + z2 + 2 z1 z 2 cos (q - f)
1 i 3 æ 1 3ö
2 2 Sol. Let z = - - . Since, ç - , - ÷ lies in III quadrant.
= z1 + z2 - 2 z1 z2 2 2 è 2 2 ø
- 3 /2
Þ cos (q - f) = - 1 \ Principal value of arg (z ) = - p + tan - 1
- 1/2
\ q - f = p or arg (z 1 ) - arg (z 2 ) = p -1 p 2p
= - p + tan 3 = -p + =-
3 3
Remark 2 2
1. z1 - z2 = z1 + z2 Û arg ( z1 ) = arg ( z2 ) æ 1ö æ 3ö æ1 3ö
and z = ç - ÷ + ç - ÷ = ç + ÷ = 1 =1
2. z1 - z2 = z1 - z2 Û arg ( z1 ) - arg ( z2 ) = p è 2ø è 2 ø è4 4ø
p -
3. z1 - z2 = z1 + z2 Û arg ( z1 ) - arg ( z2 ) = ± , z1 z2 \ Polar form of z = z [cos (arg z ) + i sin(arg z )]
2
z
and 1 are purely imaginary. 1 i 3 é æ 2p ö æ 2p ö ù
z2 i.e. - - = ê cos ç - ÷ + i sin ç - ÷
2 2 ë è 3 ø è 3 ø úû
Chap 01 Complex Numbers 15

z =1
(c) Euler’s Form Sol. Given,

iq
\ z =eiq …(i)
If q Î R and i = - 1, then e = cos q + i sin q is known as
Þ arg (z ) = q …(ii)
Euler’s identity. æ arg (z ) ö
1 + i tan ç ÷
Now, e - i q = cos q - i sin q è 2 ø 1 + i tan (q / 2)
RHS = = [from Eq. (ii)]
Let z = ei q æ arg (z ) ö 1 - i tan (q / 2)
1 - i tan ç ÷
è 2 ø
\ z = 1 and arg (z ) = q
iq -i q cos q / 2 + i sin q / 2 e i q/ 2
Also, e +e = 2 cos q and e i q - e - i q 2i sin q = = - i q/ 2
cos q / 2 - i sin q / 2 e
and if q, f Î R and i = - 1, then = e i q = z = LHS [from Eq. (i)]
æq+ fö
iç ÷
è 2 ø æ q - fö æ æ a - ib ö ö 2ab
(i) e iq + e if = e × 2 cos ç ÷ y Example 41. Prove that tan ç i ln ç ÷÷ = 2
è 2 ø
è è a + ib ø ø a - b 2
æ q - fö
\ e iq + e if = 2 cos ç ÷ ( where a, b ÎR + and i = - 1 ).
è 2 ø
a - ib a - ib
æ q + fö Sol. Q = =1 [Q | z | = | z | ]
and arg (e i q + e if ) = ç ÷ a + ib a + ib
è 2 ø
æq+ fö a - ib
iç ÷
è 2 øæ q - fö Let = e iq …(i)
(ii) e iq - e if = e × 2i sin ç ÷ a + ib
è 2 ø
By componendo and dividendo , we get
æ q - fö
\ e iq - e if = 2 sin ç ÷ ( a - ib ) - (a + ib ) e i q - 1 b
è 2 ø = - i = i tan (q / 2)
(a - ib ) + (a + ib ) e i q + 1 a
q+f p
and arg (e iq - e if ) = + [Qi = e ip / 2 ] æq ö b
2 2 or tan ç ÷ = - …(ii)
è2ø a
Remark
1. e iq + 1 = e iq / 2× 2cos (q/ 2) (Remember) æ æ a - ib ö ö
\ LHS = tan çi ln ç ÷÷
2. e iq
-1= e iq / 2
× 2i sin (q/ 2) (Remember) è è a + ib ø ø

3.
e iq - 1
= i tan ( q / 2) (Remember) = tan (i ln (e i q )) [from Eq. (i)]
iq
e +1 = tan (i × i q ) = - tan q
4. If z = r e iq ; z = r , then arg ( z ) = q, z = r e- iq 2 tan q / 2
=-
5. If z - z0 = 1, then z - z0 = e iq 1 - tan 2 q / 2
2 ( - b / a)
y Example 39. Given that z - 1 = 1, where z is a point =- [from Eq. (ii)]
1 - ( - b / a )2
z -2
on the argand plane, show that = i tan (arg z ) , =
2ab
= RHS
z a2 - b2
where i = -1.
Sol. Given, z - 1 = 1 Applications of Euler’s Form
\ z - 1 = e i q Þ z = e i q + 1 = e i q / 2 × 2 cos (q / 2)
If x , y , q Î R and i = -1, then
\ arg (z ) = q / 2 …(i)
iq iq let z = x + iy [cartesian form]
z -2 1+e -2 e -1
LHS = = = iq = i tan (q / 2) = z (cos q + i sin q ) [polar form]
z 1 + e iq e +1
= i tan (arg z ) = RHS [from Eq. (i)] = z e iq [Euler’s form]

y Example 40. Let z be a non-real complex number (i) Product of Two Complex Numbers
æ arg (z ) ö Let two complex numbers be
1 + i tan ç ÷
è 2 ø z 1 = | z 1 | e iq1 and z 2 = | z 2 | e iq2 ,
lying on z = 1, prove that z =
æ arg (z ) ö
1 - i tan ç ÷ where q 1 , q 2 Î R and i = - 1
( where, i = - 1 ). è 2 ø
16 Textbook of Algebra

\ z 1 × z 2 = z 1 e iq1 × z 2 e iq2 = z 1 z 2 e i ( q1 + q2 ) = - w (cos (arg w ) - i sin (arg w ))


= - w (cos ( - arg w ) + i sin ( - arg w ))
= z 1 z 2 (cos (q 1 + q 2 ) + i sin (q 1 + q 2 )) = - w (cos (arg w ) + i sin (arg w )) = - w
Thus, z1 z2 = z1 z2
and arg (z 1 z 2 ) = q 1 + q 2 = arg (z 1 ) + arg (z 2 ) y Example 44. Express ( 1 + i ) - i , (where, i = -1) in the
form A + iB.
(ii) Division of Two Complex Numbers
Sol. Let A + iB = ( 1 + i )- i
Let two complex numbers be
On taking logarithm both sides, we get
z 1 = z 1 e iq1 and z 2 = z 2 e iq2 ,
loge ( A + iB ) = - i loge (1 + i )
where q 1 , q 2 Î R and i = - 1 æ æ 1 i öö
= - i loge ç 2 ç + ÷÷
z1 z 1 e iq1 z1 è è 2 2 øø
\ = = e i ( q1 - q2 )
z2 iq2 z æ æ p p öö
z2 e 2
= - i loge ç 2 ç cos + i sin ÷ ÷
è è 4 4 øø
z1
= (cos (q 1 - q 2 ) + i sin (q 1 - q 2 )) = - i loge ( 2 e i p / 4 ) = - i (loge 2 + loge e i p / 4)
z2 ipö p
æ1 i
= - i ç loge 2 + ÷ = - loge 2 +
z1 z1 è2 4ø 2 4
Thus, = , (z 2 ¹ 0 ) i p
z2 z2 - log e 2 +
p/ 4 2 - 1/ 2
\ A + iB = e 2 4 =e × e i log e
æz ö =e p/ 4
× (cos ( loge 2- 1/ 2 ) + i sin ( loge 2- 1/ 2 ))
and arg ç 1 ÷ = q 1 - q 2 = arg (z 1 ) - arg (z 2 )
èz2 ø æ æ 1 öö æ æ 1 öö
p/ 4 p/ 4
=e × cos ç loge ç ÷ ÷ + i e sin ç loge ç ÷ ÷
è è 2 øø è è 2 øø
(iii) Logarithm of a Complex Number
iq
log e (z ) = log e ( z e ) = log e z + log e (e iq ) y Example 45. If sin ( log e i i ) = a + ib , where i = - 1,
= log e z + iq = log e z + i arg (z ) find a and b, hence and find cos ( log e i i ).
So, the general value of log e (z ) Sol. a + ib = sin ( loge i i ) = sin ( i loge i )
= log e (z ) + 2npi ( -p < arg z < p ). = sin ( i ( loge i + i arg i ))
= sin ( i ( loge 1 + (i p / 2)))
y Example 42. If m and x are two real numbers and = sin ( i ( 0 + (i p / 2))) = sin ( - p / 2) = - 1
m
-1 æ xi + 1 ö \ a = -1,b = 0
i = - 1 , prove that e 2 m i cot x ç ÷ = 1.
è xi - 1 ø Now, cos( loge i i ) = 1 - sin 2 ( loge i i )
-1
Sol. Let cot x = q , then cot q = x
= 1 - ( - 1)2 = (1 - 1) = 0
m m
-1 æ xi + 1 ö 2 m i q æ i cot q + 1 ö Aliter
\LHS = e 2 m i cot x
ç ÷ =e ç ÷
è xi - 1 ø è i cot q - 1 ø Q i i = ( e ip / 2 ) i = e - p / 2
æ i (cot q - i ) ö
m m
æ p ö
2 m i q æ cos q - i sin q ö \ sin (loge i i ) = sin (loge e - p / 2 ) = sin ç - loge e ÷
= e2 m iq ç ÷ =e ç ÷ è 2 ø
è i (cot q + i ) ø è cos q + i sin q ø
m = sin ( - p / 2) = - 1 = a + ib [given]
æe - i q ö \ a = - 1, b = 0
= e2 m i q× ç iq ÷ =e 2 mi q
× (e - 2 i q )m
èe ø and cos ( loge i i ) = cos ( loge e - p / 2 )
=e 2 mi q
× e -2 m i q = e 0 = 1 = RHS æ p ö æ pö
= cos ç - loge e ÷ = cos ç - ÷ = 0
è 2 ø è 2ø
y Example 43. If z and w are two non-zero complex
numbers such that z = w and arg (z ) + arg (w ) = p, y Example 46. Find the general value of log 2 ( 5i ),
prove that z = - w . where i = - 1.
Sol. Let arg (w ) = q , then arg (z ) = p - q loge 5i 1
Sol. log 2 5i = = { loge 5i + i arg ( 5i ) + 2npi }
\ z = z (cos (arg z ) + i sin (arg z )) loge 2 loge 2
= z (cos ( p - q ) + i sin ( p - q )) 1 ip
= { loge 5 + + 2npi } , n Î I
= z ( - cos q + i sin q ) = - z (cos q - i sin q ) loge 2 2
Chap 01 Complex Numbers 17

#L Exercise for Session 2


1 - ix
1 If = a - ib and a 2 + b 2 = 1, where a, b Î R and i = -1, then x is equal to
1 + ix
2a 2b 2a 2b
(a) (b) (c) (d)
(1 + a ) 2 + b 2 (1 + a ) 2 + b 2 (1 + b ) 2 + a 2 (1 + b ) 2 + a 2
n
æ1+ i ö 2æ -1 1 ö
2 The least positive integer n for which ç ÷ = çsec + sin- 1 x ÷ ( where, x ¹ 0 , - 1 £ x £ 1and i = - 1), is
è 1- i ø pè x ø
(a) 2 (b) 4 (c) 6 (d) 8
3 If z = (3 + 4i )6 + (3 - 4i )6, where i = - 1, then Im(z ) equals to
(a) - 6 (b) 0 (c) 6 (d) None of these
æx yö
4 If ( x + iy )1/ 3 = a + ib , where i = - 1, then ç + ÷ is equal to
èa b ø
(a) 4 a 2b 2 (b) 4 (a 2 - b 2 ) (c) 4 a 2 - b 2 (d) a 2 + b 2
3
5 If = a + ib , where i = - 1 and a 2 + b 2 = l a - 3, the value of l is
2 + cos q + i sin q
(a) 3 (b) 4 (c) 5 (d) 6
z -1
6 If is purely imaginary, then z is equal to
z +1
1
(a) (b) 1 (c) 2 (d) 2
2

7 The complex numbers sin x + i cos 2x and cos x - i sin 2x , where i = - 1, are conjugate to each other, for

(c) x = æçn + ö÷,n ÎI


1
(a) x = np,n Î I (b) x = 0 (d) no value of x
è 2ø
b- a
8 If a and b are two different complex numbers with | b | = 1, then is equal to
1- ab
1
(a) 0 (b) (c) 1 (d) 2
2

9 If x = 3 + 4i ( where, i = - 1), the value of x 4 - 12x 3


+ 70x 2
- 204 x + 225, is
(a) - 45 (b) 0 (c) 35 (d) 15

10 If z1 - 1 £ 1, z 2 - 2 £ 2, z 3 - 3 £ 3, the greatest value of z1 + z 2 + z 3 is


(a) 6 (b) 12 (c) 17 (d) 23
æ 8p ö 8p
11 The principal value of arg (z ), where z = ç1 + cos
è
÷ + i sin ( where, i = - 1) is given by
5 ø 5
p 4p p 4p
(a) - (b) - (c) (d)
5 5 5 5

12 If z1 = 2, z 2 = 3, z 3 = 4 and z1 + z 2 + z 3 = 5, then 4z 2 z 3 + 9z 3 z1 + 16 z1 z 2 is
(a) 24 (b) 60 (c) 120 (d) 240

13 If z - i £ 5 and z1 = 5 + 3i ( where, i = - 1), the greatest and least values of iz + z 1 are


(a) 7 and 3 (b) 9 and 1 (c) 10 and 0 (d) None of these
æz ö æz ö
14 If z1, z 2 and z 3, z4 are two pairs of conjugate complex numbers, then arg ç 1 ÷ + arg ç 2 ÷ equals to
è z4 ø èz3 ø
p 3p
(a) 0 (b) (c) p (d)
2 2
Session 3
amp(z) — amp (—z) = ± π; According as amp (z) is Positive or
Negative, Square Root of a Complex Number, Solution of
Complex Equations, De-Moivre’s Theorem, Cube Roots of Unity

amp ( z ) - amp (- z ) = ± p , From Eq. (ii), we get


z1 = z 2 (cos ( p + arg (z 2 )) + i sin ( p + arg (z 2 )))
According as amp ( z) is Positive [from Eq. (i) and z1 = z 2 ]
or Negative = z 2 ( - cos (arg z 2 ) - i sin (arg z 2 )) = - z 2
[from Eq. (iii)]
Case I amp (z ) is positive.
\ z1 + z 2 = 0
If amp (z ) = q , we have
Y y Example 48.Let z and w be two non-zero complex
numbers, such that z = w and
P z
amp (z ) + amp (w ) = p, then find the relation between
r z and w .
θ Sol. Given, amp (z ) + amp (w ) = p
X
O
r
– ( π – θ)
Þ amp (z ) - amp (w ) = p
Here, z =w = w [given | z | = | w | ]
P′ –z
and amp (z ) > 0
Then, z +w =0
amp ( - z ) = - ( ÐP ¢ OX ) = - ( p - q )
\ amp (z ) - amp ( -z ) = p
Case II amp (z ) is negative.
[here, OP = OP ¢]
Square Root of a Complex Number
Let z = x + iy ,
If amp (z ) = - q
where x , y Î R and i = - 1.
We have, amp ( -z ) = ÐP ¢OX = p - q
\ amp (z ) - amp ( -z ) = - p [here, OP = OP ¢] Suppose ( x + iy ) = a + ib …(i)
Y On squaring both sides, we get
( x + iy ) = (a 2 - b 2 ) + 2iab
P' –z

π–θ On comparing the real and imaginary parts, we get


r
O –θ
X
a2 - b2 = x …(ii)
r
P z and 2ab = y …(iii)
\ a2 + b2 = (a 2 - b 2 ) 2 + 4a 2 b 2 = (x 2 + y 2 )
a2 + b2 = z …(iv)
y Example 47. If z 1 = z 2 and arg (z 1 / z 2 ) = p, then
From Eqs. (ii) and (iv), we get
find the value of z 1 + z 2 .
æz ö æ z +xö æ z -xö
Sol. Q arg ç 1 ÷ = p a = ± çç ÷, b = ±
÷
ç
ç 2 ÷
÷
èz2 ø è 2 ø è ø
Þ arg (z1 ) - arg (z 2 ) = p …(i)
æ z + Re (z ) ö æ z - Re (z ) ö
Q z1 = z1 (cos ( arg z1 ) + i sin (arg z1 )) …(ii) or a = ± çç ÷ , b=± ç ÷
2 ÷ ç 2 ÷
and z 2 = z 2 (cos (arg z 2 ) + i sin (arg z 2 )) …(iii) è ø è ø
Chap 01 Complex Numbers 19

Now, from Eq. (i), the required square roots, ìïæ 3 ö 2 æ i ö 2 3 i üï


ì æ z + Re ( z ) = íç ÷ + ç ÷ + 2 × × ý
z - Re ( z ) ö ïîè 2 ø è 2ø 2 2 ïþ
ï± ç +i ÷ , if Im ( z ) > 0
ïï çè 2 2 ÷
ø 2
i.e. z = í æ3 +iö æ3 +i ö
ï æç z + Re ( z ) z - Re ( z ) ö
÷ , if Im ( z ) < 0
= ç ÷ = ±ç ÷
ï± ç -i è 2 ø è 2 ø
2 2 ÷
ïî è ø
(ii) Let z = - 5 + 12i
Aliter \ | z | = 13, Re (z ) = - 5, Im (z ) = 12 > 0
If ( x + iy ), where x , y Î R and i = - 1, then
æ z + Re (z ) z - Re (z ) ö
(i) If y is not even, then multiply and divide in y by 2, Q z =±ç +i ÷
ç 2 2 ÷
then ( x + iy ) convert in è ø
æ y2 ö æ æ 13 - 5 ö æ 13 + 5 ö ö
x +y - 1 = çx + 2 - ÷. \ ( - 5 + 12i ) = ± ç ç ÷ +i ç ÷÷
ç
è 4 ÷
ø è è 2 ø è 2 øø
= ± (2 + 3i )
y2
(ii) Factorise: - say a , b (a < b). Aliter
4
( - 5 + 12i ) = ( - 5 + 12 - 1 )
Take that possible factor which satisfy
x = (ai ) 2 + b2 , if x > 0 or x = a 2 + (ib)2 , if x < 0 = ( -5 +2 ( - 36 )
(iii) Finally, write x + iy = (ai)2 + b2 + 2iab = ( -5 +2 ( - 9 ´ 4) )
or a 2 + (ib)2 + 2iab = (- 9 + 4 + 2 ( - 9 ´ 4 ))
and take their square root.
= (3i ) 2 + 2 2 + 2 × 3i × 2
ì± (ai + b) ì± (b - ia )
(iv) ( x + iy ) = í and ( x - iy ) = í
îor ± (a + ib) îor ± (a -ib) = ( 2 + 3i ) 2 = ± (2 + 3i )
(iii) Let z = - 8 - 15i
Remark
1+ iö \ z = 17, Re (z ) = - 8, Im (z ) = - 15 < 0
1. The square root of i is ± æç ÷, where i = -1.
è 2 ø æ æ 17 - 8 ö æ 17 + 8 ö ö
1- iö \ ( - 8 - 15i ) = ± ç ç ÷ -i ç ÷÷
2. The square root of ( - i ) is æç ÷. è è 2 ø è 2 øø
è 2 ø
æ 3 - 5i ö
y Example 49. Find the square roots of the following =±ç ÷
è 2 ø
(i) 4 + 3i (ii) - 5 + 12i
Aliter ( - 8 - 15i ) = ( - 8 - 15 - 1 )
(iii) - 8 - 15i (iv) 7 - 24i (where, i = - 1)
Sol. (i) Let z = 4 + 3i æ æ 225 ö ö æ æ 25 9 ö ö
\ | z | = 5, Re (z ) = 4, Im (z ) = 3 > 0 = ç- 8 - 2 ç- ÷ ÷ = ç- 8 - 2 ç- ´ ÷÷
è è 4 øø è è 2 2ø ø
æ z + Re (z )z - Re (z ) ö
z =±ç +i ÷
æ 9 25 æ 25 9 ö ö
Q
ç 2 2 ÷ = ç - - 2 ç- ´ ÷÷
è ø
è2 2 è 2 2ø ø
æ æ5 + 4ö æ5 - 4ö ö æ3 +iö
\ ( 4 + 3i ) = ± ç ç ÷ +i ç ÷÷ =±ç ÷
è è 2 ø è 2 øø è 2 ø 2 2
Aliter æ 3 ö æ 5i ö 3 5i
= ç ÷ + ç ÷ -2× ×
æ 9ö è 2ø è 2ø 2 2
( 4 + 3i ) = 4 + 3 - 1 = 4 + 2 ç - ÷
è 4ø 2
æ 3 - 5i ö æ 3 - 5i ö
9 1 æ 9ö = ç ÷ =±ç ÷
= - + 2 ç- ÷ è 2 ø è 2 ø
2 2 è 4ø
20 Textbook of Algebra

(iv) Let z = 7 - 24i æ x 2 + x + 1ö æ x 2


- x + 1ö
ç ÷-ç ÷
\ z = 25, Re (z ) = 7, Im (z ) = - 24 < 0 è 2 ø è 2 ø
=
æ z + Re (z ) z - Re (z ) ö éæ x 2
+ x + 1ö æx 2
- x + 1öù
Q z =±ç -i ÷ + 2 êç ÷´-ç ÷ú
ç 2 2 ÷ êë è 2 ø è 2 ø úû
è ø
æ æ 25 + 7 ö æ 25 - 7 ö ö
\ (7 - 24i ) = ± ç ç ÷ -i ç ÷÷ ìæ 2
ö æ æ x 2 - x +1ö ö
2 ü
è è 2 ø è 2 øø ïç ç æ x 2
+ x + 1 ö ï
÷ ÷ + çi ç ÷÷
ïï çè è 2 ø ÷ø çè è 2 ø ÷ø ïï
= ± ( 4 - 3i ) = í ý
Aliter ï æ x 2 + x +1ö æ x 2 - x + 1ö ï
ï+ 2 ç ÷ ×i ç ÷ ï
(7 - 24 i ) = (7 - 24 -1 ) = 7 - 2 ( - 144 ) ïî è 2 ø è 2 ø ïþ
2
= 7 - 2 (16 ´ - 9 ) æ æ x 2 + x +1ö æ x 2 - x +1ö ö
= ç ç ÷ +i ç ÷÷
ç è 2 ø è 2 ø ÷ø
è
= 16 - 9 - 2 (16 ´ - 9 )
æ æx 2
+ x + 1ö æx 2
- x + 1 ö ö÷
= ( 4 ) 2 + (3i ) 2 - 2 × 4 × 3i =±ç ç ÷ +i ç ÷
ç è 2 ø è 2 ø ÷ø
è
= ( 4 - 3i ) 2 = ± ( 4 - 3i )

y Example 50. Find the square root of Solution of Complex Equations


x + ( - x4 - x 2
- 1).
Putting z = x + iy , where x , y Î R and i = - 1 in the given
4 2
Sol. Let z = x + (- x - x - 1) equation and equating the real and imaginary parts, we
= x + i (x 4 + x 2
+ 1) [Q -1 = i ]
get x and y, then required solution is z = x + iy .

\ z = x 2
+ (x + x 4 2
+ 1) y Example 51. Solve the equation z 2 + z = 0.
= ( x 4 + 2x 2
+ 1) = ( x 2
+ 1) 2 Sol. Let z = x + iy , where x , y Î R and i = - 1 …(i)

\ z = ( x 2 + 1) Þ z 2 = ( x + iy )2 = x 2
- y 2 + 2ixy

Re (z ) = x and z = (x 2 + y 2 )
Im (z ) = ( x 4 + x 2
+ 1) > 0 Then, given equation reduces to
2
æ z + Re (z ) z - Re (z ) ö x - y 2 + 2ixy + ( x 2
+ y 2) = 0
Q z = ± çç +i ÷
÷
è 2 2 ø On comparing the real and imaginary parts, we get
2
x - y 2 + (x 2
+ y 2) = 0 …(ii)
\ x + (- x 4 - x 2
- 1)
and 2xy = 0 …(iii)
æ æx 2
+1+ xö æx 2
+ 1 - x ö ö÷ From Eq. (iii), let x = 0 and from Eq. (ii),
=±ç ç ÷ +i ç ÷
ç è 2 ø è 2 ø ÷ø - y2 + y2 = 0
è
2
Aliter Þ - y + y =0
æ- x - x 4 2
- 1ö \ y = 0, 1
x + (- x 4 - x 2
- 1) = x + 2 ç ÷
è 4 ø Þ y = 0, ± 1
From Eq. (iii), let y = 0 and from Eq. (ii),
æ - ( x 2 + x + 1) ( x 2 - x + 1) ö 2 2
= x +2 ç ÷ x + x =0
è 4 ø 2
Þ x + x =0
éæ x 2
+ x + 1ö æx 2
- x + 1öù Þ x
2
+ x =0 Þ x =0
= x + 2 êç ÷´-ç ÷ú
êë è 2 ø è 2 ø úû \ x + iy are 0 + 0 ×i, 0 + i, 0 - i
i.e. z = 0, i , - i are the solutions of the given equation.
Chap 01 Complex Numbers 21

y Example 52. Find the number of solutions of the


2
equation z 2 + z = 0.
De-Moivre’s Theorem
2 Statements
Sol. Q z2 + z = 0 or z 2 + z z = 0
(i) If q 1 , q 2 , q 3 , ..., q n Î R and i = - 1, then
Þ z (z + z ) = 0
(cos q 1 + i sin q 1 ) (cos q 2 + i sin q 2 )
\ z =0 …(i) (cos q 3 + i sin q 3 )... (cos q n + i sin q n )
and z + z = 0 Þ 2 Re (z ) = 0 = cos (q 1 + q 2 + q 3 + ...+ q n )
\ Re (z ) = 0 + i sin (q 1 + q 2 + q 3 + ... + q n )
If z = x + iy [Q x = Re (z )]
(ii) If q Î R, n Î I (set of integers) and i = - 1, then
= 0 + iy , y Î R
and i = -1 …(ii) (cos q + i sin q ) n = cos nq + i sin nq
On combining Eqs. (i) and (ii), then we can say that the (iii) If q Î R, n Î Q (set of rational numbers)
given equation has infinite solutions. and i = - 1, then cos n q + i sin n q is one of the values
y Example 53. Find all complex numbers satisfying of (cos q + i sin q ) n .
2
the equation 2 z + z 2 - 5 + i 3 = 0, where i = - 1. Proof
Sol. Let z = x + iy , where x , y Î R and i = - 1 (i) By Euler’s formula, e iq = cos q + i sin q
Þ z 2 = ( x + iy )2 = x 2 - y 2 + 2ixy LHS = (cos q 1 + i sin q 1 ) (cos q 2 + i sin q 2 )
and z = (x 2 + y 2 ) (cos q 3 + i sin q 3 ) ... (cos q n + i sin q n )
Then, given equation reduces to = e iq1 × e iq2 × e iq3 ... e iqn = e i ( q1 + q2 + q3 + ... + qn )
2 ( x 2 + y 2 ) + x 2 - y 2 + 2ixy - 5 + i 3 = 0 = cos (q 1 + q 2 + q 3 + ... + q n )
Þ 2 2
(3x + y - 5) + i (2xy + 3 ) = 0 = 0 + i × 0 + i sin (q 1 + q 2 + q 3 + ... + q n ) = RHS
On comparing the real and imaginary parts, we get (ii) If q 1 = q 2 = q 3 = ... = q n = q, then from the above
3x 2 + y 2 - 5 = 0 …(i) result (i), (cos q + i sin q ) (cos q + i sin q )
and 2xy + 3 = 0 …(ii) (cos q + i sin q ) ... upto n factors
On substituting the value of x from Eq. (ii) in Eq. (i), we get = cos (q + q + q + ... upton times)
æ 3ö
2 + i sin (q + q + q + ... upto n times)
2
3 ç- ÷ +y -5=0
è 2y ø i.e., (cos q + i sin q ) n = cos nq + i sin nq
9
Þ + y2 = 5 p
4y 2 (iii) Let n = , where p, q Î I and q ¹ 0, from above result (ii),
q
or 4y 4 - 20y 2 + 9 = 0 q
æ æp ö æ p öö
Þ (2y 2 - 9 ) (2y 2 - 1) = 0 we have ç cos ç q ÷ + i sin ç q ÷ ÷
è èq ø è q øø
9 1 3 1
\ y 2 = , y 2 = or y = ± ,y = ± ææ p ö ö ææ p ö ö
2 2 2 2 = cos ç ç q ÷ q ÷ + i sin ç ç q ÷ q ÷ = cos p q + i sin p q
or y=-
3 3
, ,-
1 1
,
èè q ø ø èè q ø ø
2 2 2 2 æ pq ö æ pq ö
From Eq. (ii), we get Þ cos ç ÷ + i sin ç ÷ is one of the values of
è q ø è q ø
1 1 3 3
x= ,- , ,- (cos p q + i sin p q ) 1 /q
6 6 2 2
\ z = x + iy æ pq ö æ pq ö
Þ cos ç ÷ + i sin ç ÷ is one of the values of
1 3i 1 3i 3 i 3 i è q ø è q ø
= - ,- + , - ,- +
6 2 6 2 2 2 2 2 [(cos q + i sin q ) p ]1 /q
are the solutions of the given equation.
22 Textbook of Algebra

æ pq ö æ pq ö To Find the Roots of (a + ib ) p /q , where a , b Î R ;


Þ cos ç ÷ + i sin ç ÷ is one of the values of
è q ø è q ø p , q Î I , q ¹ 0 and i = - 1
/q
(cos q + i sin q ) p Let a + ib = r (cos q + i sin q ) [polar form]
\ (a + ib )p / q = {r (cos (2np + q )
Other Forms of De-Moivre’s Theorem + i sin (2np + q) )} p /q
,n ÎI
1. (cos q - i sin q) n = cos n q - i sin n q, " n Î I
p /q p /q
Proof (cos q - i sin q) n = (cos ( - q) + i sin ( - q)) n =r (cos (2np + q ) + i sin (2np + q ))
= cos ( - nq) + i sin ( - nq) = cos nq - i sin nq /qæ æp ö æp öö
2. (sin q + i cos q) n = ( i ) n (cos nq - i sin n q), " n Î I
=rp ç cos ç (2np + q )÷ + i sin ç (2np + q )÷ ÷ ,
è èq ø èq øø
Proof (sin q + i cos q) n = ( i (cos q - i sin q)) n where, n = 0, 1, 2, 3, ..., q - 1
= i n (cos q - i sin q) n = ( i ) n (cos nq - i sin n q)
[from remark (1)] y Example 56. Find all roots of x 5 - 1 = 0.
3. (sin q - i cos q) n = ( - i ) n (cos nq + i sin nq), " n Î I
Proof (sin q - i cos q) n = ( - i (cos q + i sin q)) n
Sol. Q x 5 - 1 = 0 Þ x 5 = 1
= ( - i ) n (cos q + i sin q) n \ x = (1)1 / 5 = (cos 0 + i sin 0)1 / 5 ,
= ( - i ) n (cos nq + i sin nq)
n
4. (cos q + i sin f) ¹ cos nq + i sin nf, " n Î I
where i = -1
[here, q ¹ f\ De-Moivre’s theorem is not applicable] = [cos (2np + 0) + i sin (2np + 0)]1/ 5
1
5. = (cos q + i sin q) - 1
cos q + i sin q æ 2np ö æ 2n p ö
= cos ç ÷ + i sin ç ÷,
è 5 ø è 5 ø
= cos ( - q) + i sin ( - q) = cos q - i sin q
where, n = 0, 1, 2, 3, 4
p p
y Example 54. If z r = cos æç r ö÷ + i sin æç r ö÷ , where
\ Roots are
è3 ø è3 ø æ 2p ö æ 2p ö æ 4p ö
1, cos ç ÷ + i sin ç ÷, cos ç ÷ + i sin
æ 4p ö
ç ÷,
i = - 1, prove that z 1 z 2 z 3 ... upto infinity = i. è 5 ø è 5 ø è 5 ø è 5 ø
æpö æpö æ 6p ö æ 6p ö æ 8p ö æ 8p ö
Sol. We have, zr = cos ç r ÷ + i sin ç r ÷ cos ç ÷ + i sin ç ÷, cos ç ÷ + i sin ç ÷
è3 ø è3 ø è 5 ø è 5 ø è 5 ø è 5 ø
æp p p ö æ 6p ö æ 6p ö
\ z1 z 2 z 3 ... ¥ = cos ç + 2 + 3 + ... + ¥ ÷ Now, cos ç ÷ + i sin ç ÷
è3 3 3 ø è 5 ø è 5 ø
æp p p ö æ 4p ö æ 4p ö
+ i sin ç + 2 + 3 +... + ¥ ÷ = cos ç2p - ÷ + i sin ç2p - ÷
è3 3 3 ø è 5 ø è 5 ø
æ p ö æ p ö æ 4p ö æ 4p ö
ç ÷ ç ÷ æpö æpö = cos ç ÷ - i sin ç ÷
= cos ç 3 ÷ + i sin ç 3 ÷ = cos ç ÷ + i sin ç ÷ è 5 ø è 5 ø
1
ç1 - ÷ ç1 - ÷1 è 2 ø è2ø
è ø è ø æ 8p ö æ 8p ö
3 3 and cos ç ÷ + i sin ç ÷
è 5 ø è 5 ø
= 0 + i ×1 = i
æ 2p ö æ 2p ö
= cos ç2p - ÷ + i sin ç2p - ÷
(cos q + i sin q ) 4 è 5 ø è 5 ø
y Example 55. Express in a + ib
(sin q + i cos q ) 5 æ 2p ö æ 2p ö
= cos ç ÷ - i sin ç ÷
è 5 ø è 5 ø
form, where i = - 1.
Sol. Q (sin q + i cos q )5 = (i )5 (cos q - i sin q ) 5 æ 2p ö æ 2p ö
Hence, roots are 1, cos ç ÷ ± i sin ç ÷
è 5 ø è 5 ø
= i (cos q + i sin q )- 5
æ 4p ö æ 4p ö
(cos q + i sin q )4 (cos q + i sin q )4 and cos ç ÷ ± i sin ç ÷.
\ = è 5 ø è 5 ø
(sin q + i cos q ) 5 i (cos q + i sin q )- 5
(cos q + i sin q )9 Remark
=
i Five roots are 1, z1, z2, z1, z2 (one real, two complex and two
cos 9 q + i sin 9 q conjugate of complex roots).
= = - i cos 9 q + sin 9 q
i
= sin 9 q - i cos 9 q
Chap 01 Complex Numbers 23

y Example 57. Find all roots of the equation


x 6 - x 5 + x 4 - x 3 + x 2 - x + 1 = 0.
Properties of Cube Roots of Unity
2 3 4 5 6
(i) 1 + w + w2 = 0 and w 3 = 1
Sol. Q 1- x + x - x + x - x + x =0
[1 - ( - x )7 ] (ii) To find the value of wn (n > 3 ).
Þ 1× = 0, 1 + x ¹ 0
1 - (- x ) First divide n by 3. Let q be the quotient and r be the
or 1 + x 7 = 0, x ¹ - 1 or x 7 = - 1 remainder. 3 ) n (q
\ x = ( - 1)1 / 7 = (cos p + i sin p)1 / 7 , i = -1 - 3q
r
= [cos (2n + 1) p + i sin (2n + 1) p ]1 / 7
i.e. n = 3q + r , where 0 £ r £ 2
æ (2n + 1)p ö æ (2n + 1)p ö
= cos ç ÷ + i sin ç ÷
è 7 ø è 7 ø \ wn = w 3q + r = ( w 3 ) q × wr = wr
for n = 0, 1, 2, 4, 5, 6. In general, w 3n = 1 , w 3n + 1 = w , w 3n + 2 = w 2
Remark ì3, when n is a multiple of 3
(iii) 1 + wr + w2r = í
QFor n = 3, x = - 1 but here x ¹ - 1 î0, when n is not a multiple of 3
\ n¹ 3
(iv) Cube roots of - 1 are - 1, - w and - w2 .
(v) a + b w + c w2 = 0 Þ a = b = c , if a, b, c Î R.
Cube Roots of Unity (vi) If a, b, c are non-zero numbers such that
Let z = (1) 1/ 3
Þ z =1 Þ z -1=0 3 3 a + b + c = 0 = a 2 + b 2 + c 2 , then a : b : c = 1 : w : w2 .
Þ (z - 1) (z 2 + z + 1) = 0 Þ z - 1 = 0 or z 2 + z + 1 = 0 (vii) A complex number a + ib (where i = -1), for which
- 1 ± (1 - 4 )
-1 ± i 3 a : b = 1 : 3 or 3 : 1 can always be expressed in
\ z = 1 or z = =
2 2 terms of w or w2 .
- 1 + i 3 - 1 -i 3
Therefore, z = 1 , , , where i = -1. For example,
2 2 (a) 1 + i 3 = - ( - 1 - i 3 ) [Q |1 : 3 | = 1 : 3 ]
If second root is represented by w (omega), third root will
be w2 . æ-1-i 3ö 2
= -2 ç ÷ = -2w
\ Cube roots of unity are 1, w, w2 and w, w2 are called è 2 ø
non-real complex cube roots of unity. i ( 3 + i ) (- 1 + i 3 )
(b) 3 +i = =
i i
Remark æ - 1 + i 3 ö æ2ö
1. w = w2, ( w) 2 = w 2. w = ± w2, w2 = ± w =ç ÷ç ÷ [Q | 3 : 1| = 3 : 1]
è 2 ø èiø
3. w = w2 = 1
2w
= = -2i w
Aliter i
Let z = (1) 1 / 3 = (cos 0 + i sin 0 ) 1 / 3 , i = - 1 (viii) The cube roots of unity when represented on complex
plane lie on vertices of an equilateral triangle
= [cos (2 np + 0 ) + i sin (2 np + 0 )]1 / 3 inscribed in a unit circle, having centre at origin. One
æ 2 np ö æ 2 np ö vertex being on positive real axis.
= cos ç ÷ + i sin ç ÷ , where, n = 0, 1, 2
è 3 ø è 3 ø Y
i
Therefore, roots are 2π/3
ω
æ 2p ö æ 2p ö æ 4p ö æ 4p ö 2π/3
1, cos ç ÷ + i sin ç ÷ , cos ç ÷ + i sin ç ÷
è 3 ø è 3 ø è 3 ø è 3 ø X′ X
–1 1
2pi / 3 4pi / 3 O
or 1, e ,e
ω2 2π/3
If second root is represented by w, then third root will be w2 –i
or if third root is represented by w, then second root will be w2 . Y′
24 Textbook of Algebra

Important Relations in Terms y Example 59. If a , b and g are the roots of


of Cube Root of Unity x 3 - 3x 2 + 3x + 7 = 0, find the value of
(i) a 2+ ab + b 2 = (a - bw ) (a - bw2 ) a - 1 b - 1 g -1
+ + .
(ii) a 2- ab + b 2 = (a + bw ) (a + bw2 ) b -1 g -1 a -1
(iii) a 3+ b 3 = (a + b ) (a + bw ) (a + bw2 ) Sol. We have, x 3 - 3x 2 + 3x + 7 = 0
(iv) a 3 - b 3 = (a - b ) (a - bw ) (a - bw2 ) Þ ( x - 1 )3 + 8 = 0
(v) a 2 + b 2 + c 2 - ab - bc - ca
Þ ( x - 1) 3 + 2 3 = 0
= (a + bw + cw2 ) (a + bw2 + cw )
(vi) a 3 + b 3 + c 3 - 3abc Þ ( x - 1 + 2) ( x - 1 + 2 w) ( x - 1 + 2 w2 ) = 0

= (a + b + c ) (a + bw + cw2 ) (a + bw2 + cw ) Þ ( x + 1) ( x - 1 + 2 w) ( x - 1 + 2 w2 ) = 0

y Example 58. If w is a non-real complex cube root of \ x = - 1, 1 - 2 w, 1 - 2 w2


unity, find the values of the following. Þ a = - 1, b = 1 - 2 w, g = 1 - 2 w2
1999
(i) w a - 1 b -1 g - 1 -2 - 2 w - 2 w2
- 998 Then, + + = + +
(ii) w b - 1 g - 1 a - 1 - 2 w - 2 w2 -2
3n + 2
æ - 1+ i 3ö 1 1
(iii) ç ÷ , n Î N and i = - 1 = + + w2 = w2 + w2 + w2 = 3 w2
è 2 ø w w

(iv) (1 + w) (1 + w 2 ) (1 + w 4 ) (1 + w 8 )... upto 2n factors 3+i


y Example 60. If z = , where i = - 1, find the
2
æ a + bw + gw 2 + dw 2 ö
(v) ç ÷ , where a, b, g , d ÎR value of (z 101 + i 103 )105 .
2
è b + aw + gw + dw ø
3 + i 1 æi 3 + i 2 ö
2 2 Sol. Q z= = ç ÷ [Qi 2 = - 1]
(vi) 1 × (2 - w) (2 - w ) + 2 × ( 3 - w) ( 3 - w ) + 3 × 2 i è 2 ø
(4 - w)(4 - w 2 ) + K + K + (n - 1) ×(n - w)(n - w 2 ) æ- 1 + i 3ö
= -i ç ÷ = -iw
Sol. (i) w 1999
=w 3 ´ 666 + 1
=w è 2 ø
1 w \ z 101 = ( - iw)101 = - i 101 × w101 = - i w2 and i 103 = i 3 = - i
(ii) w- 998 = 998
= 999
=w
w w Then, z 101 + i 103 = - i w2 - i = - i ( w2 + 1)
3n + 2
æ - 1+i 3 ö
(iii) ç ÷ = w3n + 2 = w3n × w2 = ( w3 )n × w2 = - i ( - w) = i w
è 2 ø
101 103 105
Hence, (z +i ) = (iw)105 = i 105 × w105 = i × 1 = i
= (1)n × w2 = w2
50
(iv) (1 + w) (1 + w2 ) (1 + w4 ) (1 + w8 ) ... upto 2n factors æ3 i 3ö
y Example 61. If ç + ÷ = 3 25 ( x - iy ), where
= (1 + w) (1 + w2 ) (1 + w) (1 + w2 ) ... upto 2n factors è 2 2 ø
= ( - w2 ) ( - w) ( - w2 ) (- w) ... upto 2n factors x , y ÎR and i = - 1, find the ordered pair of ( x , y ).
= ( w3 ) ( w3 )... upto n factors = 1 × 1× 1 × ...upto n factors 3 i 3 æ 3 +iö 3 æi 3 + i 2 ö
Sol. Q + = 3ç ÷ = ç ÷
= (1)n = 1 2 2 è 2 ø i è 2 ø

æ a + b w + g w2 + d w2 ö w (a + b w + g w2 + d w2 ) æ- 1 + i 3ö
(v) ç ÷= = -i 3 ç ÷ = -i 3 w
è b + a w2 + g w + d w ø ( b w + a w3 + g w2 + d w2 ) è 2 ø
50
w (a + b w + g w2 + d w2 ) æ3 i 3 ö
= =w \ç + ÷ = ( - i 3 w) 50 = i 50 × 325 × w50
(b w + a + g w + d w ) 2 2 è2 2 ø

(vi) S (n - 1) (n - w) (n - w2 ) = S (n 3 - 1) = S n3 - S 1 æ- 1 - i 3ö
= - 1 × 325 × w2 = - 325 × ç ÷
2 è 2 ø
ì n ( n + 1) ü
=í ý -n
î 2 þ
Chap 01 Complex Numbers 25

æ1 i 3 ö Then, f ( - w) = 0 and f ( - w2 ) = 0
= 325 ç + 25
÷ = 3 ( x - iy ) [given]
è2 2 ø Þ - 7 w3 - aw + b = 0 and - 7 w6 - aw2 + b = 0
1 3 or - 7 - aw + b = 0
\ x = ,y = -
2 2 and - 7 - aw2 + b = 0
æ1 3ö On adding, we get
Þ Ordered pair is ç , - ÷.
è2 2 ø - 14 - a ( w + w2 ) + 2b = 0
or - 14 + a + 2b = 0 or a + 2b = 14 …(i)
y Example 62. If the polynomial 7 x 3 + ax + b is
and on subtracting, we get
divisible by x 2 - x + 1, find the value of 2a + b . - a ( w - w2 ) = 0
3
Sol. Let f (x ) = 7x + ax + b Þ a=0 [Q w - w2 ¹ 0]
2 2
and x - x + 1 = ( x + w) ( x + w ) From Eq. (i), we get b = 7
Q f ( x ) is divisible by x - x + 1 2 \ 2a + b = 7

#L Exercise for Session 3


1 The real part of (1 - i )- i , where i = - 1 is

(a) e - p / 4 cos æç loge 2ö÷ (b) - e - p / 4 sin æç loge 2ö÷


1 1
è2 ø è2 ø

cos æç loge 2ö÷ (d) e - p / 4 sin æç loge 2ö÷


p/4 1 1
(c) e
è2 ø è2 ø
- iq
2 The amplitude of e e , where q ÎR and i = - 1 is
(a) sinq (b) - sinq
(c) e cos q (d) e sinq

3 If z = i loge (2 - 3 ), where i = - 1, then the cos z is equal to


(a) i (b) 2i (c) 1 (d) 2
i
4 If z = i i , where i = - 1, then z is equal to
(a) 1 (b) e - p / 2 (c) e - p (d) e p

5 ( - 8 - 6i ) is equal to (where, i = -1)


(a) 1 ± 3i (b) ± (1 - 3i ) (c) ± (1 + 3 i ) (d) ± (3 - i )

(5 + 12i ) + (5 - 12i )
6 is equal to (where, i = - 1)
(5 + 12i ) - (5 - 12i )
3 3 3 3
(a) - i (b) i (c) - i (d) -
2 4 4 2

7 If 0 < amp (z ) < p, then amp (z ) - amp ( - z ) is equal to


(a) 0 (b) 2 amp (z ) (c) p (d) - p

8 If z1 = z 2 and amp (z1) + amp (z 2 ) = 0, then


(a) z1 = z 2 (b) z1 = z 2 (c) z1 + z 2 = 0 (d) z1 = z 2

9 The solution of the equation z - z = 1 + 2i , where i = - 1, is


3 3 3 3
(a) 2 - i (b) + 2i (c) - 2i (d) - 2 + i
2 2 2 2
26 Textbook of Algebra

10 The number of solutions of the equation z 2 + z = 0, is


(a) 1 (b) 2
(c) 3 (d) 4
æ ra ö æ ra ö
11 If z r = cos ç 2 ÷ + i sin ç 2 ÷, where r = 1, 2, 3, ..., n and i = - 1, then lim z1z 2 z 3 K z n is equal to
èn ø èn ø n®¥

(a) e ia (b) e - ia / 2
(c) e ia / 2 (d) 3 e ia
n
æ 1 + sin q + i cos q ö
12 If q ÎR and i = - 1, then ç ÷ is equal to
è 1 + sin q - i cos q ø
np np np np
(a) cos æç - nqö÷ + i sin æç - nqö÷ (b) cos æç + nqö÷ + i sin æç + nqö÷
è 2 ø è 2 ø è 2 ø è 2 ø
np np æ p ö æ p ö
(c) sin æç - nqö÷ + i cos æç - nqö÷ (d) cos çn æç + 2qö÷ ÷ + i sinçn æç + 2 qö÷ ÷
è 2 ø è 2 ø è è2 øø è è2 øø

13 If i z 4 + 1 = 0, where i = - 1, then z can take the value


1+ i p p
(a) (b) cos æç ö÷ + i sin æç ö÷
2 è 8ø è 8ø
1
(c) (d) i
4i
14 If w ( ¹ 1) is a cube root of unity, then (1 - w + w2 ) (1 - w2 + w4 ) (1 - w4 + w8 ) ... upto 2n factors, is
(a) 2n (b) 22 n
(c) 0 (d) 1
x a+ yb+z g
15 If a, b and g are the cube roots of p ( p < 0), then for any x , y and z , is equal to
xb+ y g +z a
1 1
(a) (- 1 -i 3 ), i = -1 (b) (1 + i 3 ), i = -1
2 2
1
(c) (1 - i 3 ), i = -1 (d) None of these
2
Session 4
nth Root of Unity, Vector Representation of Complex Numbers,
Geometrical Representation of Algebraic Operation on Complex Numbers,
Rotation Theorem (Coni Method), Shifting the Origin in Case of Complex
Numbers, Inverse Points, Dot and Cross Product, Use of Complex Numbers
in Coordinate Geometry

nth Root of Unity Remark


1 × a × a2 × a3 ... an - 1 = ( - 1) n - 1 is the basic concept to be
Let x be the nth root of unity, then
understood.
x = (1) 1 /n = (cos 0 + i sin 0 ) 1 /n
(c) If a is an imaginary nth root of unity, then other roots
= (cos (2kp + 0 ) + i sin (2kp + 0 ) 1 /n are given by a 2 , a 3 , a 4 , ... , a n .
[where k is an integer]
1 /n
= (cos 2kp + i sin 2kp) Imaginary
axis
æ 2kp ö æ 2kp ö
\ x = cos ç ÷ + i sin ç ÷ Y
è n ø è n ø
i
where, k = 0, 1, 2, 3, ..., n - 1 A2(α2)
r
2p 2p A1(α)
Let a = cos + i sin , the n, nth roots of unity are θ
n n θ A0
X′ X Real axis
a k (k = 0, 1, 2, 3, ... , n - 1) i.e, the n, nth roots of unity are –1 θ 1

1, a, a 2 , a 3 , ... , a n - 1 which are in GP with common ratio An-1(αn-1)

= e 2pi / n . –i

(a) Sum of n, nth roots of unity Y′

1 × (1 - a n ) (d) Q 1 + a + a 2 + ... + a n - 1 = 0
1 + a + a 2 + a 3 + ... + a n - 1 =
(1 - a ) n -1

=
1 - (cos 2 p + i sin 2 p) Þ S ak = 0
k=0
1-a
n -1 n -1
æ 2 pk ö æ 2 pk ö
=
1 - (1 + 0 )
=0 or S cos ç ÷ + i S sin ç ÷ =0
1-a k=0 è n ø k=0 è n ø
n -1
Remark Þ S cos æçè 2pk ö÷ø = 0
k=0
2 3
1 + a + a + a + ... + a n -1
= 0 is the basic concept to be n
understood. n -1
æ 2 pk ö
(b) Product of n , nth roots of unity and S
k=0
sin ç
è
÷ =0
ø n
1 ´ a ´ a 2 ´ a 3 ´ ... ´ a n - 1 = a 1 + 2 + 3 + ... + (n - 1 )
(n - 1 ) n These roots are located at the vertices of a regular
(n - 1 ) n
æ 2p 2p ö 2 plane polygon of n sides inscribed in a unit circle
=a 2 = ç cos + i sin ÷ having centre at origin, one vertex being on positive
è n n ø
real axis.
= cos (n - 1) p + i sin (n - 1) p
(e) x n - 1 = ( x - 1) ( x - a ) ( x - a 2 ) ... ( x - a n - 1 ).
= (cos p + i sin p)n - 1 = ( - 1) n - 1
28 Textbook of Algebra

Important Benefits ì
iq
niq
2 × 2i sin æ nq ö ï
ü
1. If 1, a1, a2, a3, ... , an - 1 are the n, nth root of unity, then q q
ï e × e ç ÷
ìe { (e ) - 1} ü
i i n
ï è 2 øï
( 1) p + ( a1 ) p + ( a2 ) p + ... + ( an - 1 ) p = Im í ý = Im í ý
iq
ì0, if p is not an integral multiple of n î e -1 þ ï e iq /2 × 2i sin æç q ö÷ ï

în, if p is an integral multiple of n ï è2ø ï
î þ
ì0 , if nis even
2. ( 1 + a1 ) ( 1 + a2 ) ... ( 1 + an - 1 ) = í ì æ nq ö ü æ nq ö
î1, if nis odd ïsin çè 2 ÷ø æç n + 1 ö÷iq ï sin çè 2 ÷ø éæ n + 1ö ù
ï è ø ï
3. ( 1 - a1 ) ( 1 - a2 ) ... ( 1 - an - 1 ) = n = Im í ×e 2 ý= × sin ê ç ÷ qú
( n - 2) / 2
æ
ï sin ç ÷q ö ï sin ç ÷ æ q ö ë è 2 ø û
æ z 2 - 2z cos 2rp + 1ö,
4. z n - 1 = ( z - 1) ( z + 1) P
r =1
ç
è
÷
ø
ïî è2ø ïþ è2ø
n
if ‘n’ is even.
( n - 2) / 2 Remark
5. z n + 1 = P
æ 2 æ ( 2r + 1) p ö + 1ö, if n is even. 2p
ç z - 2z cos ç ÷ ÷ For q = , we get
r =0 è è n ø ø
n
( n - 3) / 2
æ 2 æ ( 2r + 1) p ö + 1ö, 2p 4p 6p ( 2n - 2) p ö
6. z n + 1 = ( z + 1) P ç z - 2z cos ç ÷ ÷ 1. 1 + cos æç ö÷ + cos æç ö÷ + cos æç ö÷ + ... + cos æç ÷ =0
r =0 è è n ø ø è nø è nø è nø è n ø
if ‘n’ is odd. 2p 4p 6p ( 2n - 2) p ö
2. sin æç ö÷ + sin æç ö÷ + sin æç ö÷ + ... + sin æç ÷ =0
è nø è nø è nø è n ø

The Sum of the Following y Example 63. If 1, w , w 2 , ..., wn - 1 are n, nth roots of
Series Should be Remembered unity, find the value of (9 - w) (9 - w 2 ) ...(9 - wn -1 ).
(i) cos q + cos 2 q + cos 3 q + ... + cos nq Sol. Let x = (1)1/n Þ xn - 1 = 0
æ nq ö has n roots 1, w, w2 , ... , wn - 1
sin ç ÷
è 2 ø éæ n + 1ö ù \ x n - 1 = ( x - 1) ( x - w) ( x - w2 ) K ( x - wn - 1 )
= . cos ê ç ÷q
q
æ ö ë è 2 ø úû On putting x = 9 in both sides, we get
sin ç ÷
è2ø 9n - 1
= (9 - w) (9 - w2 ) (9 - w3 ) ... (9 - wn - 1 )
(ii) sin q + sin 2 q + sin 3 q + ... + sin nq 9 -1
æ nq ö (9 - w) (9 - w2 ) ... (9 - wn - 1 ) =
9n - 1
sin ç ÷ or
è 2 ø éæ n + 1ö ù 8
= × sin ê ç ÷q
q
æ ö ë è 2 ø úû Remark
sin ç ÷
è2ø x n -1
= ( x - w) ( x - w2 ) ... ( x - wn - 1 )
Proof x -1
xn -1
(i) cos q + cos 2 q + cos 3 q + ... + cos n q \ lim = lim ( x - w) ( x - w2 ) ... ( x - wn - 1 )
x ®1 x -1 x ®1
= Re {e iq + e 2iq + e 3i q + ... + e niq }, where i = -1 Þ n = ( 1 - w) ( 1 - w2 ) ... ( 1 - wn - 1 )
ì iq niq /2 æ nq ö ü
ïe × e × 2i sin ç ÷ ï æ 2p ö æ 2p ö
ìe iq { (e iq ) n - 1} ü ï è 2 øï y Example 64. If a = cos ç ÷ + i sin ç ÷ , where
= Re í ý = Re í ý è 7 ø è 7 ø
iq iq /2
î e -1 þ ï e × 2i sin (q /2 ) ï i = -1, find the quadratic equation whose roots
ïî ïþ
are a = a + a 2 + a 4 and b = a 3 + a 5 + a 6 .
ì æ nq ö ü æ nq ö æ 2p ö æ 2p ö
ïsin çè 2 ÷ø æç n + 1 ö÷iq ï sin çè 2 ÷ø éæ n + 1ö ù
Sol. Q a = cos ç ÷ + i sin ç ÷
ï è ø ï è7 ø è7 ø
= Re í ×e 2 ý= × cos ê ç ÷ qú
ï sin æç q ö÷ ï sin æç q ö÷ ëè 2 ø û \ a 7 = cos 2p + i sin 2p = 1 + 0 = 1
ïî è2ø ïþ è2ø or a = (1)1 / 7
(ii) sin q + sin 2 q + sin 3 q + ... + sin nq \ 1 , a , a 2 , a 3 , a 4 , a 5 , a 6 are 7, 7 th roots of unity.
iq 2iq 3iq niq
= Im {e +e +e + ... + e }, where i = -1 \ 1 + a + a2 + a3 + a4 + a5 + a6 = 0 …(i)
2 4 3 5 6
Þ (a + a + a ) + (a + a + a ) = - 1 or a + b = - 1
Chap 01 Complex Numbers 29

and ab = (a + a 2 + a 4 ) (a 3 + a 5 + a 6 ) y Example 67. If n ³ 3 and 1, a 1 , a 2 , a 3 , ..., a n -1 are


= a 4 + a 6 + a 7 + a 5 + a 7 + a 8 + a 7 + a 9 + a10 the n, nth roots of unity, then find the value of
= a4 + a6 + 1 + a5 + 1 + a + 1 + a2 + a3 [Qa 7 = 1] S S ai a j .
= (1 + a + a 2 + a 3 + a 4 + a 5 + a 6 ) + 2 1 £ i < j £ n -1

=0+2 [from Eq. (i)] Sol. Let x = (1)1/n


=2
\ x n = 1 or x n - 1 = 0
Therefore, the required equation is
\ 1 + a 1 + a 2 + a 3 + ... + a n - 1 = 0
x 2 - (a + b ) x + ab = 0 or x 2 + x + 2 = 0
or a 1 + a 2 + a 3 + ... + a n - 1 = - 1
y Example 65. Find the value of On squaring both sides, we get
2 2 2
10 a 1 + a 2 + a 3 + ... + a 2n - 1 + 2 (a 1a 2 + a 1a 3
é æ 2pk ö æ 2pk ö ù
S
k =1 ê
sin ç
è
÷ - i cos ç
ø
÷ , where i = - 1.
è 11 ø úû + ... + a 1a n - 1 + a 2 a 3 + ... + a 2 a n - 1
ë 11
10
+ ... + a n - 2 a n - 1 ) = 1
S ésin æçè 2pk ö÷ø - i cos æçè 2pk ö÷ø ùú or 1 + (a 1 ) + (a 2 ) + (a 3 ) + ... + (a n - 1 )2
2 2 2 2
Sol.
k =1 êë 11 11 û
10
+2 SS a i a j = 1 + 12
1£i < j £n -1
= -i S é cos æçè 2pk ö÷ø + i sin æçè 2pk ö÷ø ùú
k =1 ê 0+2 SS ai a j = 2
ë 11 11 û
1£i < j £n -1
ìï 10
é æ 2p k ö æ 2p k ö ù üï [here, p is not a multiple of n]
= - i í S ê cos ç ÷ + i sin ç ÷ ú - 1ý
ïîk = 0 ë è 11 ø è 11 ø û ïþ \ SS ai a j = 1
1£i < j £n -1
= - i ( 0 - 1) [sum of 11, 11th roots of unity] Aliter
=i
Q x n - 1 = ( x - 1) ( x - a 1 ) ( x - a 2 ) ... ( x - a n -1 )
y Example 66. If a 0 , a 1 , a 2 , ..., a n - 1 are the n, nth
n -1 On comparing the coefficient of x n - 2 both sides, we get
ai
roots of the unity, then find the value of S . 0= SS a i a j + a 1 + a 2 + ... + a n - 1
i =0 2 - a i 0 £i < j £n -1

Sol. Let x = (1)1/n Þ x n = 1 \ xn - 1 = 0 0= SS ai a j - 1


n 1£i < j £n -1
or x - 1 = ( x - a 0 ) ( x - a 1 ) ( x - a 2 ) ... ( x - a n - 1 ) [ Q1 + a 1 + a 2 + ... + a n - 1 = 0]
n -1
\ SS ai a j = 1
= P (x - ai )
i =0
1£i < j £n -1

On taking logarithm both sides, we get


n -1 Vector Representation of
loge ( x n - 1) = S loge ( x - a i )
i =0 Complex Numbers
On differentiating both sides w.r.t. x, we get If P is the point ( x , y ) on the argand plane corresponding
nx n - 1
n -1
æ 1 ö to the complex number z = x + iy , where x , y Î R and
= S ç ÷ i = - 1.
xn - 1 i =0 è x - ai ø
Y
On putting x = 2, we get P (x, y)
n -1
n -1
n ( 2)
= S 1
…(i)
n
2 -1 i =0 (2 - a i ) y
n -1 n -1
θ
ai æ 2 ö
Now, S = S ç- 1 + ÷ O x M
X
i =0 (2 - a i ) i = 0 è 2 - ai ø
n -1 n -1 ¾® ¾®
2 × n × 2n - 1 OP = x $i + y $j Þ OP = ( x 2 + y 2 ) = z
=- S 1+2 S 1
= - (n ) + n [from Eq. (i)]
Then,
¾®
i =0 i =0 (2 - a i ) 2 -1
and arg (z ) = direction of the vector OP = tan - 1 (y / x ) = q
n × 2n n ¾®
= -n + n
= n Therefore, complex number z can also be represented by OP.
2 -1 2 -1
30 Textbook of Algebra

Geometrical Representation (c) Product


z 1 = r1 (cos q 1 + i sin q 1 ) = r1 e i q1
of Algebraic Operation on Let
\ z 1 = r1 and arg (z 1 ) = q 1
Complex Numbers and z 2 = r2 (cos q 2 + i sin q 2 ) = r2 e iq2
(a) Sum \ z 2 = r2 and arg (z 2 ) = q 2
Let the complex numbers z 1 = x 1 + iy 1 =( x 1 , y 1 ) and Then, z 1 z 2 = r1 r2 (cos q 1 + i sin q 1 ) (cos q 2 + i sin q 2 )
z 2 = x 2 + iy 2 = ( x 2 , y 2 ) be represented by the points P and = r1 r2 {cos (q 1 + q 2 ) + i sin (q 1 + q 2 )}
Q on the argand plane. \ z 1 z 2 = r1 r2 and arg (z 1 z 2 ) = q 1 + q 2
Y
Y R(z1z2)
R (z1 + z2)
Q (z2)

r1 r2
Q(z2)
P (z1)
X P(z1)
r2
O θ1 r1
θ2 θ1
Complete the parallelogram OPRQ. Then, the mid-points O A
X
of PQ and OR are the same. The mid-point of
æ x + x2 y1 + y2 ö Let P and Q represent the complex numbers z 1 and z 2 ,
PQ = ç 1 , ÷. respectively.
è 2 2 ø
\ OP = r1 , OQ = r2
Hence, R = ( x 1 + x 2 , y 1 + y 2 )
ÐPOX = q 1 and ÐQOX = q 2
Therefore, complex number z can also be represented by
¾® Take a point A on the real axis OX, such that OA = 1 unit.
OR = ( x 1 + x 2 ) + i (y 1 + y 2 ) = ( x 1 + iy 1 ) + ( x 2 + iy 2 ) Complete the ÐOPA
= z 1 + z 2 = (x 1 , y 1 ) + (x 2 , y 2 ) Now, taking OQ as the base, construct a DOQR similar to
In vector notation, we have OR OP
DOPA, so that =
¾® ¾® ¾® ¾® ¾® OQ OA
z 1 + z 2 = OP + OQ = OP + PR = OR
i.e. OR = OP × OQ = r1 r2 [since, OA = 1 unit]
and ÐROX = ÐROQ + ÐQOX = q 1 + q 2
(b) Difference
Hence, R is the point representing product of complex
We first represent - z 2 by Q ¢, so that QQ ¢ is bisected at O. numbers z 1 and z 2 .
Complete the parallelogram OPRQ ¢. Then, the point R
represents the difference z 1 - z 2 . Remark
1. Multiplication by i
Y p p
Since, z = r (cos q + i sin q) and i = æçcos + i sin ö÷
Q(z2) è 2 2ø
é æ p ö æ p ö ù
\ iz = r êcos ç + q÷ + i sin ç + q÷ ú
P(z1) ë è2 ø è2 øû
O
X′ X Hence, multiplication of z with i, then vector for z rotates a
right angle in the positive sense.
R(z1 – z2) 2. Thus, to multiply a vector by ( - 1) is to turn it through two
Q ′( –z2) right angles.
Y′ 3. Thus, to multiply a vector by (cos q + i sin q) is to turn it
through the angle q in the positive sense.
¾® ¾®
We see that ORPQ is a parallelogram, so that OR = QP
We have in vectorial notation, (d) Division
¾® ¾® ¾® ¾®
z 1 - z 2 = OP - OQ = OP + QO Let z 1 = r1 (cos q 1 + i sin q 1 ) = r1 e iq1
¾® ¾® ¾® ¾®
\ z 1 = r1 and arg (z 1 ) = q 1
= OP + PR = OR = QP and z 2 = r2 (cos q 2 + i sin q 2 ) = r2 e iq2
Chap 01 Complex Numbers 31

\ | z 2 | = r2 and arg (z 2 ) = q 2 ¾® ¾®
Then, we have AC = z 3 - z 1 and AB = z 2 - z 1
z r (cos q 1 + i sin q 1 )
Then, 1 = 1 × [z 2 ¹ 0, r2 ¹ 0 ] ¾®
z 2 r2 (cos q 2 + i sin q 2 ) and let arg AC = arg (z 3 - z 1 ) = q
z 1 r1 ¾®
= [cos (q 1 - q 2 ) + i sin(q 1 - q 2 )] and arg AB = arg (z 2 - z 1 ) = f
z 2 r2
z1 r æz ö Let ÐCAB = a
\ = 1 , arg ç 1 ÷ = q 1 - q 2 ¾® ¾®
z2 r2 èz2 ø ÐCAB = a = q - f = arg AC - arg AB
Let P and Q represent the complex numbers z 1 and z 2 , = arg (z 3 - z 1 ) - arg (z 2 - z 1 )
respectively.
\ OP = r1 , OQ = r2 , ÐPOX = q 1 and ÐQOX = q 2 æz - z1 ö
= arg ç 3 ÷
Let OS be new position of OP, take a point A on the real è z2 - z1 ø
axis OX, such that OA = 1 unit and through A draw a line or angle between AC and AB
making with OA an angle equal to the ÐOQP and meeting æ affix of C - affix of A ö
OS in R. = arg ç ÷
è affix of B - affix of A ø
Then, R represented by (z 1 /z 2 ).
Y Q(z2) For any complex number z, we have
i (arg z )
z= z e
r2 é æ z - z1 ö ù
i ê arg ç 3 ÷ú
æz - z1 ö z 3 - z1 ë è z 2 - z1 ø û
r1 P(z1) Similarly, ç 3 ÷= e
θ2 è z2 - z1 ø z2 - z1
θ1 A
θ1 – θ2 X
O
z 3 - z1 z 3 - z 1 i ( Ð CAB ) AC i a
or = e = e
R z2 - z1 z2 - z1 AB
S
Now, in similar DOPQ and DOAR. Remark
OR OP r 1. Here, only principal values of the arguments are considered.
= Þ OR = 1 æ z - z2 ö
OA OQ r2 2. arg ç 1 ÷ = q, if AB coincides with CD, then
è z3 - z4 ø
since OA = 1 and ÐAOR = ÐPOR - ÐPOX = q 2 - q 1 æ z - z2 ö z1 - z2
arg ç 1 ÷ = 0 or p, so that is real. It follows that
Hence, the vectorial angle of R is - (q 2 - q 1 ) i.e., q 1 - q 2 . è z3 - z4 ø z3 - z4
z - z2
if 1 is real, then the points A, B, C, D are collinear.
Remark z3 - z4
If q1 and q2 are the principal values of z1 and z2 , then q1 + q2 and D
q1 - q2 are not necessarily the principal value of arg ( z1z2 ) and
arg ( z1 / z2 ). P(z1)
S(z4) θ
A
Rotation Theorem (Coni Method)
B
R(z3)

Let z 1 , z 2 and z 3 be the affixes of three points A, B and C Q(z2)


respectively taken on argand plane. C

Y 3. If AB is perpendicular to CD, then


C(z3)
æ z - z2 ö p z - z2
B(z2)
arg ç 1 ÷ = ± , so 1 is purely imaginary.
α è z3 - z4 ø 2 z3 - z4

A(z1) 4. It follows that, if z1 - z2 = ± k ( z3 - z4 ), where k is purely


α imaginary number, then AB and CD are perpendicular to
each other.
φ θ
X
O
32 Textbook of Algebra

y Example 68. Complex numbers z 1 , z 2 and z 1 - z 2 a ip / 3


From Coni method, = e …(i)
z 3 - z2 a
z 3 are the vertices A, B, C respectively of an isosceles
p
right angled triangle with right angle at C. Show and ÐBAC =
3
that (z 1 - z 2 ) 2 = 2 (z 1 - z 3 ) (z 3 - z 2 ). z 3 - z 1 a ip / 3
From Coni method, = e …(ii)
Sol. Since, ÐACB = 90° and AC = BC , then by Coni method z 2 - z1 a
z1 - z 3 AC ip/ 2
= e =i A(z1)
z 2 - z 3 BC
B(z2)
π/3
π/4
a a

π/3 π/3
π/4 B(z2) a C(z3)
A(z1) C(z3)
z1 - z 2 z 3 - z1
Þ z1 - z 3 = i ( z 2 - z 3 ) From Eqs. (i) and (ii), we get =
z 3 - z 2 z 2 - z1
On squaring both sides, we get Þ (z1 - z 2 )(z 2 - z1 ) = (z 3 - z1 )(z 3 - z 2 )
( z 1 - z 3 ) 2 = -( z 2 - z 3 ) 2
2 2 2 2
Þ z 12 + z 22 + z 23 = z1z 2 + z 2 z 3 + z 3z1
Þ z1 + z 3 - 2z1z 3 = -(z 2 + z 3 - 2z 2z 3 )
Þ
2
z1 + z2
2
- 2z1z 2 = 2(z1z 3 - z1z 2 - z 3 + z 2z 3 )
2 Remark
Triangle with vertices z1, z2, z3, then
Therefore, (z1 - z 2 )2 = 2(z1 - z 3 ) (z 3 - z 2 )
(i) ( z1 - z2 ) 2 + ( z2 - z3 ) 2 + ( z3 - z1 ) 2 = 0
1
Aliter CA = CB = BA (ii) ( z1 - z2 ) 2 = ( z2 - z3 )( z3 - z1 )
2
1
B(z2) (iii) å( z1 - z2 )( z2 - z3 ) = 0 (iv) å =0
( z1 - z2 )
π/4

Complex Number as a Rotating Arrow


π/4 in the Argand Plane
A(z1) C(z3) Let z = r (cos q + i sin q ) = re iq …(i)
Q ÐBAC = ( p / 4 ) be a complex number representing a point P in the argand
z 2 - z1 BA (i p / 4 ) plane.
\ = e Y Q(ze if)
z 3 - z1 CA
z1 - z2 (i p/ 4 ) r
or = 2e …(i) P(z)
z1 - z 3 φ
r
θ
X′ X
and Ð CBA = (p / 4) O
z 3 - z 2 CB (i p/ 4) z - z2 1 (i p/ 4)
\ = e or 3 = e …(ii)
z 1 - z 2 AB z1 - z2 2 Y′
On dividing Eq. (i) by Eq. (ii), we get Then, OP = z = r and ÐPOX = q
( z1 - z 2 ) 2 = 2 ( z1 - z 3 ) ( z 3 - z 2 ) Now, consider complex number z 1 = ze if
or z 1 = re iq × e if = re i ( q + f ) [from Eq. (i)]
y Example 69. Complex numbers z 1 , z 2 , z 3 are the
vertices of A, B, C respectively of an equilateral Clearly, the complex number z 1 represents a point Q in the
triangle. Show that argand plane, when OQ = r and ÐQOX = q + f
¾®
z 12 + z 22 + z 32 = z 1z 2 + z 2z 3 + z 3z 1 . Clearly, multiplication of z with e if rotates the vector OP
Sol. Let AB = BC = CA = a through angle f in anti-clockwise sense. Similarly,
¾®
p multiplication of z with e - if will rotate the vector OP in
Q ÐABC =
3 clockwise sense.
Chap 01 Complex Numbers 33

Remark
1. If z1, z2 and z3 are the affixes of the C(z3)
Shifting the Origin in Case
three points A, B and C, such that
AC = AB and ÐCAB = q. Therefore,
of Complex Numbers
¾® ¾®
AB = z2 - z1 , AC = z3 - z1.
B(z2) Let O be the origin and P be a point with affix z 0 . Let a
θ
¾® point Q has affix z with respect to the coordinate system
Then, AC will be obtained by rotating A(z1) passing through O. When origin is shifted to the point P
¾®
AB through an angle q in anti- (z 0 ), then the new affix Z of the point Q with respect to
clockwise sense and therefore, new origin P is given by Z = z - z 0 .
¾® ¾®
AC = AB e i q i.e., to shift the origin at z 0 , we should replace z by Z + z 0 .
iq z 3 - z1 iq
or ( z 3 - z1 ) = ( z2 - z1 ) e or =e y
z2 - z1 Y
Q
2. If A, B and C are three points in argand plane, such that
AC = AB and ÐCAB = q, then use the rotation about Ato find
e iq , but if AC ¹ AB, then use Coni method. X
P(z0)

y Example 70. Let z 1 and z 2 be roots of the equation


z 2 + pz + q = 0, where the coefficients p and q may be O M
x

complex numbers. Let A and B represent z 1 and z 2 in


the complex plane. If Ð AOB = a ¹ 0 and OA = OB,
where O is the origin, prove that p 2 = 4 q cos 2 (a / 2). y Example 71. If z 1 , z 2 and z 3 are the vertices of an
¾® ¾®
equilateral triangle with z 0 as its circumcentre, then
Sol. Clearly, OB is obtained by rotating OA through angle a.
¾® ¾® changing origin to z 0 , show that Z 1 2+ Z 2 2 + Z 3 2 = 0,
\ OB = OA e i a
where Z 1 , Z 2 , Z 3 are new complex numbers of the
Þ z 2 = z1 e i a
vertices.
z2
Þ =eia …(i) Sol. In an equilateral triangle, the circumcentre and the
z1
centroid are the same point.
B(z2)
z1 + z 2 + z 3
So, z0 =
3
A(z1) \ z1 + z 2 + z 3 = 3z 0 …(i)
α To shift the origin at z 0 , we have to replace z1, z 2 , z 3 and z 0
O by Z 1 + z 0 , Z 2 + z 0 , Z 3 + z 0 and 0 + z 0 .
z2 Then, Eq. (i) becomes
or + 1 = ( e i a + 1)
z1 ( Z 1 + z 0 ) + ( Z 2 + z 0 ) + ( Z 3 + z 0 ) = 3( 0 + z 0 )
( z1 + z 2 ) Þ Z1 + Z 2 + Z 3 = 0
Þ = e i a / 2 × 2 cos (a / 2)
z1 On squaring, we get
2 2 2
On squaring both sides, we get Z 1 + Z 2 + Z 3 + 2 (Z 1Z 2 + Z 2Z 3 + Z 3Z 1 ) = 0 …(ii)
( z1 + z 2 ) 2 But triangle with vertices Z 1 , Z 2 and Z 3 is equilateral, then
= e i a × ( 4 cos 2 a / 2) 2 2 2
z1
2 Z 1 + Z 2 + Z 3 = Z 1Z 2 + Z 2 Z 3 + Z 3Z 1 …(iii)
( z1 + z 2 ) 2 z2 From Eqs. (ii) and (iii), we get
Þ = × ( 4 cos 2 a / 2) [from Eq. (i)] 2 2 2
z12 z1 3 (Z 1 + Z 2 + Z 3 ) = 0
2 2 2
(z1 + z 2 )2 = 4 z1z 2 cos 2 (a / 2) Therefore, Z1 + Z 2 + Z 3 = 0
( - p )2 = 4 q cos 2 (a / 2)
éQz1 and z 2 are the roots of z 2 + pz + q = 0ù
ê ú
Inverse Points
ë\z1 + z 2 = - p and z1 z 2 = q û (a) Inverse points with respect to a line Two points P
2 2
or p = 4 q cos (a / 2) and Q are said to be the inverse points with respect to
the line RS. If Q is the image of P in RS, i.e., if the line
RS is the right bisector of PQ.
34 Textbook of Algebra

y Example 72. Show that z 1 , z 2 are the inverse points y Example 73. Show that inverse of a point a with
with respect to the line z a + a z = b , if z 1 a + a z 2 = b . respect to the circle z - c = R (a and c are complex
R2
Sol. Let RS be the line represented by the equation, numbers, centre c and radius R) is the point c + .
z a +az =b …(i) a -c
Let P and Q are the inverse points with respect to the line RS. Sol. Let a¢ be the inverse point of a with respect to the circle
The point Q is the reflection (inverse) of the point P in the z - c = R, then by definition,
line RS, if the line RS is the right bisector of PQ. Take any
point z in the line RS, then lines joining z to P and z to Q are
equal. c a′
P a |z – c | = R

The points c , a, a ¢ are collinear.


R S We have, arg (a ¢ - c ) = arg(a - c )
-
= - arg ( a - c ) [Q arg z = - arg z ]
Þ arg (a ¢ - c ) + arg ( a - c ) = 0
Q Þ arg {(a ¢ - c ) ( a - c )} = 0
2 2 \ (a ¢ - c ) ( a - c ) is purely real and positive.
i.e., z - z1 = z - z 2 or z - z1 = z - z2
By definition, a ¢ - c a - c = R 2 [QCP × CQ = r 2 ]
i.e., ( z - z1 ) ( z - z1 ) = ( z - z 2 ) ( z - z 2 )
Þ z ( z 2 - z1 ) + z ( z 2 - z1 ) + ( z1z1 - z 2 z 2 ) = 0 …(ii) Þ a¢ - c a - c = R2 [Q | z | = | z | ]
2
Hence, Eqs. (i) and (ii) are identical, therefore, comparing Þ (a ¢ - c ) ( a - c ) = R
coefficients, we get
Þ (a ¢ - c ) ( a - c ) = R 2
P z1
[Q(a ¢ - c )( a - c ) is purely real and positive]
R2 R2
Þ a¢ - c = Þ a¢ = c +
a -c a -c
R S
A(z)

Dot and Cross Product


Q z2 Let z 1 = x 1 + iy 1 º ( x 1 , y 1 ) and z 2 = x 2 + iy 2 º ( x 2 , y 2 ),
a a -b where x 1 , y 1 , x 2 , y 2 Î R and i = - 1, be two complex
= =
z 2 - z1 z 2 - z1 z1z1 - z 2 z 2 numbers.
z1a az 2
If ÐPOQ = q, then from Coni method,
So that, =
z1( z 2 - z1 ) z 2 (z 2 - z1 ) z2 - 0 z 2 iq
= e Q(z2)
-b z a + az 2 - b z1 - 0 z1
= = 1
z1z1 - z 2 z 2 0 z 2 iq
z2z1
[by ratio and proportion rule] Þ = e θ
P(z1)
z1z1 z1
z1a + az 2 - b = 0 or z1 a + az 2 = b
O
(b) Inverse points with respect to a circle If C is the z2z1 z2 iq
Þ = e
centre of the circle and P , Q are the inverse points z1
2 z1
with respect to the circle, then three points C , P , Q are
collinear and also CP × CQ = r 2 , where r is the z 2 z 1 = | z 1 || z 2 | e iq
radius of the circle. z 2 z 1 = z 1 z 2 (cos q + i sin q )
Þ Re (z 2 z 1 ) = z 1 z 2 cos q …(i)
and Im (z 2 z 1 ) = z 1 z 2 sin q …(ii)
Q The dot product z 1 and z 2 is defined by,
C P
z 1 ×z 2 = z 1 z 2 cos q
= Re (z 1 z 2 ) = x 1 x 2 + y 1 y 2 [from Eq. (i)]
Chap 01 Complex Numbers 35

and cross product of z 1 and z 2 is defined by Remark


z 1 ´ z 2 = z 1 z 2 sin q 1. The distance of a point z from origin, z - 0 = z
2. Three points A( z1 ), B ( z2 ) and C ( z3 ) are collinear, then
= Im (z 1 z 2 ) = x 1 y 2 - x 2 y 1 [from Eq. (ii)] AB + BC = AC
Hence, z 1 ×z 2 = x 1 x 2 + y 1 y 2 = Re (z 1 z 2 ) C(z3)
and z 1 ´ z 2 = x 1 y 2 - x 2 y 1 = Im (z 1 z 2 )

Results for Dot and Cross B(z2)

Products of Complex Number A(z1)


1. If z1 and z2 are perpendicular, then z1 × z2 = 0 i.e. z1 - z2 + z2 - z3 = z1 - z3 .
2. If z1 and z2 are parallel, then z1 ´ z2 = 0
3. Projection of z1 on z2 = ( z1× z2 ) / z2 y Example 75. Show that the points representing the
4. Projection of z2 on z1 = ( z1× z2 ) / z1 complex numbers ( 3 + 2i ), (2 - i ) and - 7i, where
5. Area of triangle, if two sides represented by z 1 and z2 is
1
i = - 1, are collinear.
z1 ´ z2 . Sol. Let z 1 = 3 + 2i , z 2 = 2 - i and z 3 = - 7i .
2
6. Area of a parallelogram having sides z 1 and z2 is z1 ´ z2 . Then, z 1 - z 2 = 1 + 3i = 10, z 2 - z 3 = 2 + 6i
7. Area of parallelogram, if diagonals represented by z1 and z2 is
1 = 40 = 2 10
z1 ´ z2 .
2 and z1 - z 3 = 3 + 9i = 90 = 3 10
y Example 74. If z 1 = 2 + 5i , z 2 = 3 - i , where i = -1, find \ z1 - z 2 + z 2 - z 3 = z1 - z 3
(i) z 1 × z 2 (ii) z 1 ´ z 2 Hence, the points (3 + 2i ), (2 - i ) and - 7i are collinear.
(iii) z 2 × z 1 (iv) z 2 ´ z 1
(v) acute angle between z 1 and z 2 . (b) Equation of the
(vi) projection of z 1 on z 2 . Perpendicular Bisector
Sol. (i) z1×z 2 = x 1x 2 + y1y 2 = (2) (3) + (5) ( - 1) = 1
If P (z 1 ) and Q (z 2 ) are two fixed points and R (z ) is
(ii) z1 ´ z 2 = x 1y 2 - x 2 y1 = (2) ( - 1) - (3) (5) = - 17
moving point, such that it is always at equal distance from
(iii) z 2 ×z1 = x 1 x 2 + y1y 2 = (2) (3) + (5) ( - 1) = 1 P (z 1 ) and Q (z 2 ).
(iv) z 2 ´ z1 = x 2 y1 - x 1y 2 = (3) (5) - (2) ( - 1) = 17 P(z1)

(v) Let angle between z1 and z 2 be q, then


R (z)
z1 × z 2 = z1 z 2 cosq
Þ 1 = ( 4 + 25) (9 + 1) cosq Q(z2)

1 æ 1 ö i.e. PR = QR
\ cosq = \ q = cos - 1 ç ÷
290 è 290 ø or z - z1 = z - z2
z ×z 1 1
(vi) Projection of z1 on z 2 = 1 2 = = or z (z 1 - z 2 ) + z (z 1 - z 2 ) = z 1 z 1 - z 2 z 2
z2 ( 9 + 1) 10 2 2
or z (z 1 - z 2 ) + z (z 1 - z 2 ) = z 1 - z2

Use of Complex Numbers in Hence, z lies on the perpendicular bisectors of z 1 and z 2 .

Coordinate Geometry y Example 76. Find the perpendicular bisector of 3 + 4i


and - 5 + 6i, where i = - 1.
(a) Distance Formula Sol. Let z1 = 3 + 4i and z 2 = - 5 + 6i
The distance between two points P (z 1 ) and Q (z 2 ) is given by If z is moving point, such that it is always equal distance
Q(z2) from z1 and z 2 .
i.e. z - z1 = z - z 2
2
or z ( z1 - z 2 ) + z ( z1 - z 2 ) = z1 2 - z 2
Þ z (( 3 - 4i ) - ( - 5 - 6i )) + z (( 3 + 4i ) - ( - 5 + 6i )) = 25 - 61
P(z1) Hence, ( 8 + 2i ) z + ( 8 - 2i ) z + 36 = 0
PQ = z 2 - z 1 = affix of Q - affix of P which is required perpendicular bisector.
36 Textbook of Algebra

z1 + z 2 + z 3 2 ×0 + 1 × z
(c) Section Formula Þ
3
=
2+1
If R (z ) divides the joining of P (z 1 ) and Q (z 2 ) in the ratio Þ z1 + z 2 + z 3 = z
m 1 : m 2 (m 1 , m 2 > 0 ). Therefore, z1 + z 2 + z 3 - z = 0
Q(z2)
(m2) y Example 78. Let z 1 , z 2 and z 3 be three complex
(m 1 ) numbers and a, b , c ÎR, such that a + b + c = 0 and
R(z)
az 1 + bz 2 + cz 3 = 0, then show that z 1 , z 2 and z 3 are
P(z1) collinear.
Sol. Given, a+b+c =0 …(i)
(i) If R (z ) divides the segment PQ internally in the ratio and az1 + bz 2 + cz 3 = 0 …(ii)
m z + m2 z 1 Þ az1 + bz 2 - (a + b ) z 3 = 0 [from Eq. (i)]
of m 1 : m 2 , then z = 1 2
m1 + m2 az + bz 2
or z3 = 1
(ii) If R (z ) divides the segment PQ externally in the ratio a+b
m z - m2 z 1 It follows that z 3 divides the line segment joining z1 and z 2
of m 1 : m 2 , then z = 1 2 internally in the ratio b : a. (If a, b are of same sign and
m1 - m2
opposite sign, then externally.)
Hence, z1,z 2 and z 3 are collinear.
m2 R(z)
m1
(d) Area of Triangle
Q(z2)
If z 1 , z 2 and z 3 are the affixes of the vertices of a triangle,
P(z1) z1 z1 1
1
Remark then its area = | z 2 z 2 1 |
4
z1 + z2 z3 z3 1
1. If R ( z ) is the mid-point of PQ, then affix of R is .
2 A(z1)
2. If z1 , z2 and z3 are affixes of the vertices of a triangle, then
z + z2 + z 3
affix of its centroid is 1 .
3
3. In acute angle triangle, orthocentre ( O), nine point centre ( N ),
OG 2
centroid ( G) and circumcentre ( C) are collinear and = ,
GC 1
ON 1 B(z2) C(z3)
= .
NG 1
4. If z1, z2, z 3 and z4 are the affixes of the vertices of a Remark
parallelogram taken in order, then z1 + z 3 = z2 + z4 . The area of the triangle with vertices z, wz and z + w z is
3 2
z ,
4
y Example 77. If z 1 , z 2 and z 3 are the affixes of the where w is the cube root of unity.
vertices of a triangle having its circumcentre at the y Example 79. Show that the area of the triangle on
origin. If z is the affix of its orthocentre, prove that
the argand plane formed by the complex numbers z, iz
z 1 + z 2 + z 3 - z = 0. 1 2
Sol. We know that orthocentre O, centroid G and circumcentre and z + iz is z , where i = - 1.
2
C of a triangle are collinear, such that G divides OC in the
z1 + z 2 + z 3 z z 1
ratio 2 : 1. Since, affix of G is and C is the 1
3 Sol. Required area = | iz iz 1|
4
origin. Therefore, by section formula, we get z + iz z + iz 1
C
1 z z 1
1
2 G = | iz iz 1|
4
z + iz z + iz 1
O
Chap 01 Complex Numbers 37

z z 1 Proof Equation of the straight line joining points having


1
= | iz -iz 1 | affixes z 1 and z 2 is
4
z + iz z - iz 1 z = tz 1 + (1 - t ) z 2 , where t Î R ~ {0 }
On applying R 3 ® R 3 - ( R 1 + R 2 ), we get Þ z - z 2 = t (z 1 - z 2 ) …(i)
z z 1 and z - z 2 = t (z 1 - z 2 )
1 1
Area = | iz -iz 1 | = ( - 1) ( - izz - izz ) or z - z 2 = t (z 1 - z 2 ) …(ii)
4 4
0 0 -1
From Eqs. (i) and (ii), we get
=
1
2izz =
1
zz =
1 2 z - z2 z1 - z2 z - z2 z - z2
4 2
i
2
z = Þ =
z - z2 z1 - z2 z1 - z2 z1 - z2
Aliter
z - z2 z - z2 0
We have, iz = z (cos( p / 2) + i sin( p / 2)) = ze (ip / 2 ) iz is the
vector obtained by rotating vector z in anti-clockwise Þ z1 - z2 z1 - z2 0 =0
direction through ( p / 2). Therefore, OA ^ AB, z2 z2 1
Imaginary axis
Y Now, applying R 1 ® R 1 + R 3 and R 2 ® R 2 + R 3 , we get
B z z 1
z1 z1 1 = 0
iz z2 z2 1
z + iz
π/2
A
or z (z 1 - z 2 ) - z (z 1 - z 2 ) + z 1 z 2 - z 1 z 2 = 0
z Aliter
X Real axis
O
Let P (z ) be an arbitrary point on the line, which pass
through A (z 1 ) and B (z 2 ).
1 1
Now, area of DOAB = OA ´ AB = z iz \ Ð BAP = 0 or p
2 2
1 1 æ z - z1 ö
= z i z = z
2
\ arg ç ÷ = 0 or p [by rotation theorem]
2 2 èz2 - z1 ø
z - z1
Þ is purely real.
(e) Equation of a Straight Line z2 - z1
æ z - z1 ö æ z - z1 ö z - z1 z - z1
(i) Parametric form \ ç ÷ =ç ÷ Þ =
èz2 - z1 ø èz2 - z1 ø z2 - z1 z2 - z1
Equation of the straight line joining the points having
affixes z 1 and z 2 is A(z1)
z = tz 1 + (1 - t ) z 2 , where t Î R ~ {0 } B(z2)
Proof P(z)
tz + (1 - t ) z 2
Q z = tz 1 + (1 - t ) z 2 = 1
t + (1 - t )
z (z 1 - z 2 ) - z (z 1 - z 2 ) + z 1 z 2 - z 1 z 2 = 0
Hence, z divides the line joining z 1 and z 2 in the ratio
z z 1
1 - t : t . Thus, the points z 1 , z 2 , z are collinear.
or z1 z1 1 = 0
(ii) Non-parametric form z2 z2 1
Equation of the straight line joining the points having
affixes z 1 and z 2 is Remark
z z 1 z1 z1 1
z1 z1 1 = 0 If z1, z 2 and z 3 are collinear, z2 z2 1 =0
z3 z3 1
z2 z2 1
or z (z 1 - z 2 ) - z (z 1 - z 2 ) + z 1 z 2 - z 1 z 2 = 0
or å z1( z2 - z3 ) = 0.
38 Textbook of Algebra

(iii) General form The general equation of a straight (i) If the lines are perpendicular, then
line is of the form a z + a z + b = 0, where a is a (z3)
complex number and b is a real number.
Sol. The equation of a straight line passing through points
having affixes z1 and z 2 is given by (z1) (z2)
z (z1 - z 2 ) - z (z1 - z 2 ) + z1z 2 - z1z 2 = 0 …(i)
On multiplying Eq. (i) by i (where, i = -1), we get
(z4)
zi ( z1 - z 2 ) - z i ( z1 - z 2 ) + i ( z1z 2 - z1z 2 ) = 0
Þ z { - i ( z1 - z 2 )} + z {i ( z1 - z 2 )} + i ( z1z 2 - z1z 2 ) = 0
(z 1 - z 2 ) (z 3 - z 4 ) ip / 2
Þ z { - i ( z1 - z 2 )} + z { - i ( z1 - z 2 )} + {i ( 2i Im (z1z 2 ))} = 0 = e
z1 - z2 z3 -z4
Þ z { - i ( z1 - z 2 )} + z { - i ( z1 - z 2 )} + { ( - 2 Im (z1z 2 )} = 0
(z 1 - z 2 ) 2 (z 3 - z 4 ) 2
Þ z a + z a + b = 0, Þ = e ip
2 2
where, a = - i (z1 - z 2 ), b = - 2 Im(z1z 2 ) z1 - z2 z3 -z4
Hence, the general equation of a straight line is of the form
(z 1 - z 2 ) 2 (z 3 - z 4 ) 2
a z + a z + b = 0, Þ = e ip
where a is complex number and b is a real number. (z 1 - z 2 ) (z 1 - z 2 ) (z 3 - z 4 ) (z 3 - z 4 )
(iv) Slope of the line a z + a z + b = 0 (z 1 - z 2 ) (z 3 - z 4 )
Þ = ( - 1)
Let A (z 1 ) and B (z 2 ) be two points on the line (z 1 - z 2 ) (z 3 - z 4 )
a z + a z + b = 0, then Þ a1 = - a2 [from Eq. (i)]
a z1 + a z1 + b = 0 \ a1 + a2 = 0
and a z2 + a z2 + b = 0 (ii) If the lines are parallel, then
\ a (z 1 - z 2 ) + a (z 1 - z 2 ) = 0 z1 - z2 z -z4 0
= 3 e
z1 - z2 a z1 - z2 z3 -z4
Þ =- [Remember]
z1 - z2 a
(z 1 - z 2 ) 2 (z 3 - z 4 ) 2
a coefficient of z Þ =
Complex slope of AB = - = - z1 - z2
2
z3 -z4
2
a coefficient of z
Thus, the complex slope of the line a z + a z + b = 0 is (z 1 - z 2 ) 2 (z 3 - z 4 ) 2
Þ =
a (z 1 - z 2 ) (z 1 - z 2 ) (z 3 - z 4 ) (z 3 - z 4 )
- .
a (z 1 - z 2 ) (z 3 - z 4 )
Þ =
(z 1 - z 2 ) (z 3 - z 4 )
Remark
The real slope of the line a z + az + b = 0 is Þ a1 = a2
Re ( a) Re (coefficient of z )
- , i.e. - .
Im ( a) Im (coefficient of z ) Remark
1. The equation of a line parallel to the line a z + az + b = 0 is
Important Theorem a z + az + l = 0, where l ÎR.
2. The equation of a line perpendicular to the line
If a 1 and a 2 are the complex slopes of two lines on the a z + a z + b = 0 is a z - a z + i l = 0
argand plane, then prove that the lines are where, l ÎR and i = - 1
(i) perpendicular, if a 1 + a 2 = 0.
.

(v) Length of perpendicular from a given point on a


(ii) parallel, if a 1 = a 2 . given line
Proof Let z 1 and z 2 be the affixes of two points on one The length of perpendicular from a point P (z 1 ) to the
line with complex slope a 1 and z 3 and z 4 be the affixes of line
two points another line with complex slope a 2 . Then,
az 1 + a z 1 + b
z - z2 z 3 -z4 a z + a z + b = 0 is given by .
a1 = 1 and a 2 = …(i) 2 a
z1 - z2 z 3 -z4
Chap 01 Complex Numbers 39

Proof Let PM be perpendicular from P on the line a - a¢ æ b ö


Þ + ç- ÷ = 0
a z + a z + b = 0 and let the affix of M be z 2 , then a - a¢ è b ø
P(z1) Þ b (a - a ¢ ) - b ( a - a ¢ ) = 0 ...(ii)
On subtracting Eq. (ii) from Eq. (i), we get
a ¢ b + ab + c = 0
Aliter
M(z2) Equation of perpendicular bisector of PQ is
PM = z 1 - z 2 z ( a ¢ - a ) + z (a ¢ - a ) - a ¢ a ¢ + aa = 0 …(i)
and given line zb + z b + c = 0 …(ii)
a z +a z +b =0
Since, Eqs. (i) and (ii) are identical, we have
and M(z 2 ) lies on a z + a z + b = 0, then
a ¢ - a a ¢ - a aa - a ¢ a ¢
a z2 + a z2 + b = 0 …(i) = = =k [say]
b b c
Since, PM perpendicular to the line (a z + a z + b = 0 ). Q a ¢ - a = b k , a ¢ - a = bk
z1 - z2 æ a ö aa - a ¢ a ¢ = ck
Therefore, + ç- ÷ = 0 and
z1 - z2 è a ø ì æa¢ - a ö æ a ¢ - a öü
Now, a ¢ b + ab = ía ¢ ç ÷+a ç ÷ý
Þ a z1 - a z2 - a z1 + a z2 = 0 î è k ø è k øþ
Þ a z1 + a z1 + b = 2 a z1 + a z2 - a z2 + b 1
= {a ¢ a ¢ - a a } =
1
( - ck ) = - c
= 2 a z 1 - az 2 + (az 2 + b ) k k
Hence, a ¢b + ab + c = 0
= 2 a z1 - a z2 - a z2 [Qa z 2 + b = - a z 2 ]
= 2 a (z 1 - z 2 )
or a z1 + a z1 + b = 2 a z1 - z2 (f) Circle
= 2 a z1 - z2 [ Q|z | = |z | ] The equation of a circle whose centre is at point affix z 0
and radius r, is z - z 0 = r .
= 2 a PM
a z1 + a z1 + b P(z)
\ PM = r
2 a
C(z0)
y Example 80. Show that the point a ¢ is the reflection
of the point a in the line z b + z b + c = 0, if
a ¢b + ab + c = 0. Remark
Sol. Since, a¢ is the reflection of point a through the line. 1. If the centre of the circle is at origin and radius r, then its
So, the mid-point of PQ equation is z = r .
P a 2. z - z0 < r represents interior of a circle z - z0 = r and
z - z0 > r represent exterior of the circle z - z0 = r.
3. r < z - z0 < R, this region is known as annulus.
B A(z)
(i) General Equation of a Circle
Q a' The general equation of the circle is
a + a¢ z z + a z + a z + b = 0,
i.e., lies on z b + z b + c = 0
2 where a is a complex number and b Î R, having centre at ( -a )
æ a + a¢ ö æ a + a¢ ö 2
or bç ÷ +bç ÷ +c =0 and radius = a - b.
è 2 ø è 2 ø
Þ b (a + a ¢ ) + b (a + a ¢ ) + 2c = 0 …(i) Proof The equation of circle having centre at z 0 and
Since, PQ ^ AB. Therefore, radius r is
Complex slope of PQ + Complex slope of AB = 0 z -z0 =r
40 Textbook of Algebra

2 From Eqs. (i) and (ii), we get


Þ z -z0 =r2
(z - z 1 ) (z 2 - z 3 )
Þ (z - z 0 ) (z - z 0 ) = r 2 = Real …(iii)
(z - z 2 ) (z 1 - z 3 )
Þ zz - zz 0 - z 0 z + z 0 z 0 = r 2
2 Remark
Þ zz + ( - z 0 ) z + ( - z 0 ) z + z 0 -r2 =0 ( z4 - z1 ) ( z2 - z3 )
If four points z1, z2, z 3, z4 are concyclic, then =
Þ zz + az + az + b = 0 ( z4 - z2 ) ( z1 - z3 )
2 real [replacing z by z4 in Eq. (iii)]
where, a = - z 0 and b = z 0 -r2
é ( z - z 3 ) ( z4 - z1 ) ù
or arg ê 2 ú = p, 0.
Þ zz + a z + a z + b = 0 êë ( z1 - z 3 ) ( z4 - z2 ) ûú
where, b Î R represents a circle having centre at ( - a ) and
radius = z0
2
- b = | a |2 - b . (iii) Equation of Circle in Diametric Form
If end points of diameter represented by A (z 1 ) and B (z 2 )
Remark and P (z ) is any point on circle.
Rule to find the centre and radius of a circle whose equation is \ Ð APB = 90°
given
1. Make the coefficient of z z equal to 1 and right hand side
\ Complex slope of PA + Complex slope of PB = 0
equal to zero. P(z)
2. The centre of circle will be = ( - a) = ( - coefficient of z ).
90°
3. Radius = (|a|2 - constant term) B(z2)
ter
Di ame
y Example 81. Find the centre and radius of the circle A(z1)
2 z z + ( 3 - i ) z + ( 3 + i ) z - 7 = 0, where i = -1.
Sol. The given equation can be written as æz - z1 ö æz - z2 ö
æ3 + i ö Þ ç ÷ +ç ÷ =0
æ3 + i ö 7 èz - z1 ø èz - z2 ø
zz + ç ÷z +ç ÷z - =0
è 2 ø è 2 ø 2
Hence, ( z - z 1 ) ( z - z 2 ) + ( z - z 2 ) ( z - z 1 ) = 0
æ3 + i ö
So, it represent a circle with centre at - ç ÷ and radius which is required equation of circle in diametric form.
è 2 ø
æ æ3 + i ö 2
7ö æ9 1 7ö (iv) Other Forms of Circle
= ç -ç + ÷ = ç + + ÷= 6
ç è 2 ÷ø 2 ÷ø è 4 4 2ø (a) Equation of all circles which are orthogonal to
è
z - z 1 = r1 and z - z 2 = r2 .
(ii) Equation of Circle Through Three Let the circle be z - a = r cut given circles
Non-Collinear Points orthogonally.
2 2
Let A (z 1 ), B (z 2 ), C (z 3 ) be three points on the circle and \ r 2 + r1 = a - z 1 …(i)
P (z ) be any point on the circle, then 2 2 2
and r + r2 = a - z 2 …(ii)
P(z)
On solving,
2 2 2 2
θ
C(z3)
r2 - r1 = a (z 1 - z 2 ) + a (z 1 - z 2 ) + z 2 - z1
θ and let a = a + ib , i = - 1, a, b Î R
A(z1)
z - z1
B(z2) (b) Apollonius circle =k ¹1
z - z2
Ð ACB = Ð APB It is the circle with join of z 3 and z 4 as a diameter,
Using Coni method, z + kz 2 z - kz 2
where z 3 = 1 , z4 = 1
z 2 - z 3 BC iq 1+k 1-k
in DACB, = e …(i)
z 1 - z 3 CA for k = 1, the circle reduces to the straight line which
z 2 - z BP iq is perpendicular bisector of the line segment from z 1
in DAPB, = e …(ii) to z 2 .
z 1 - z AP
Chap 01 Complex Numbers 41

æz - z1 ö z + z + 2a
(c) Circular arc arg ç ÷ =a z -a =
èz - z2 ø 2
This is an arc of a circle in which the chord joining z 1 1 2
or z z - 4a (z + z ) = {z + (z ) 2 }
and z 2 subtends angle a at any point on the arc. 2
p where, a Î R (focus), directrix is z + z + 2a = 0.
If a = ± , then locus of zis a circle with the join of
2
z 1 and z 2 as diameter. If a = 0 or p, then locus is a (h) Equation of Ellipse
straight line through the points z 1 and z 2 .
2 2 For ellipse
(d) The equation z - z 1 + z - z2 = k, will represent a Imaginary axis
1 2 P(z)
circle, if k ³ z 1 - z 2 .
2
y Example 82. Find all circles which are orthogonal
Real axis
to z = 1 and z - 1 = 4 . S′(z2) C S(z1)

Sol. Let z -a = k …(i)


(where, a = a + ib and a, b, k Î R and i = - 1) be a circle
which cuts the circles
SP + S ¢ P = 2a
z =1 …(ii)
Þ z - z 1 + z - z 2 = 2a
and z -1 = 4 …(iii)
Orthogonally, then using the property that the sum of where, 2a > z 1 - z 2 [since, eccentricity < 1]
squares of their radii is equal to square of distance between Then, point z describes an ellipse having foci at z 1 and z 2
centres. Thus, the circle (i) will cut the circles (ii) and (iii) and a Î R + .
orthogonally, if
k 2 + 1 = a -0
2
= aa (i) Equation of Hyperbola
2 2
and k + 16 = a - 1 = ( a - 1) ( a - 1) For hyperbola
= aa - ( a + a ) + 1 Imaginary axis

\ 1 - ( a + a ) - 15 = 0 Þ a + a = -14
\ 2a = - 14 Þ a = - 7 P(z)
Þ a = a + ib = - 7 + ib
2 Real axis
Also, k2 = a - 1 = ( - 7 )2 + b 2 - 1 = b 2 + 48 S′(z2) C S(z1)

Þ k = (b 2 + 48)
Therefore, required family of circles is given by
z + 7 - ib = ( 48 + b 2 ).
SP - S ¢ P = 2a Þ z - z 1 - z - z 2 = 2a
where, 2a < z 1 - z 2 [since, eccentricity > 1]
(g) Equation of Parabola
Then, point z describes a hyperbola having foci at z 1 and
Now, for parabola z 2 and a Î R + .
Imaginary axis

M
P(z)
Examples on Geometry
y Example 83. Let z 1 = 10 + 6i , z 2 = 4 + 6i , where
Real axis
A (O) S (a + i . 0)
z + z + 2a = 0

N
i = - 1. If z is a complex number, such that the
argument of (z - z 1 ) / (z - z 2 ) is p / 4, then prove that
z - 7 - 9i = 3 2.
æ z - z1 ö p
Sol. Q arg ç ÷=
SP = PM èz - z2 ø 4
42 Textbook of Algebra

It is clear that z , z1, z 2 are non-collinear points. Always a y Example 85. In the argand plane, the vector
circle passes through z , z1 and z 2 . Let z 0 be the centre of the
z = 4 - 3i , where i = - 1, is turned in the clockwise
circle.
A(z) sense through 180° and stretched three times. Then,
find the complex number represented by the new
π/4 vector.
O(z0) Sol. Q z = 4 - 3i Þ z = ( 4 ) 2 + ( - 3) 2 = 5
r r
π/2 Let z1 be the new vector obtained by rotating z in the
(z2)B C(z1) clockwise sense through 180°, therefore
z1 = z e - ip = - z = - ( 4 - 3i ) = - 4 + 3i .
4 3
On applying rotation theorem in DBOC, The unit vector in the direction of z1 is -
+ i.
5 5
z1 - z 0 OC (ip / 2 )
= e =i [QOC = OB ] æ 4 3 ö
z 2 - z 0 OB Therefore, required vector = 3 z ç - + i ÷
è 5 5 ø
Þ ( z1 - z 0 ) = i ( z 2 - z 0 )
æ 4 3 ö
Þ 10 + 6i - z 0 = i ( 4 + 6i - z 0 ) = 15 ç - + i ÷ = - 12 + 9i
è 5 5 ø
Þ 16 + 2i = (1 - i ) z 0
(16 + 2i ) (1 + i ) Aliter
or z0 = × Imaginary axis
(1 - i ) (1 + i )
z1 3
16 + 16i + 2i + 2i 2
=
2 4 Real axis
14 + 18i –4 O
= = 7 + 9i
2
and radius, r = OC = z 0 - z1 = 7 + 9i - 10 - 6i –3 z
= - 3 + 3i
= (9 + 9 ) = 3 2 Here, z1 = - 4 + 3i
Hence,3 z1 = - 12 + 9i
Hence, required equation is
z - z0 = r y Example 86.ABCD is a rhombus. Its diagonals AC
Þ z - 7 - 9i = 3 2 and BD intersect at the point M and satisfy BD = 2AC .
If the points D and M represents the complex numbers
y Example 84.If z - 2 + i £ 2, where i = - 1, then 1 + i and 2 - i, where i = - 1, respectively, find A.
find the greatest and least value of z . Sol. Let A ºz
Sol. Q Radius = 2 units Q BD = 2AC or DM = 2 AM
C B
Y

M
O
X (2 – i)
A 2
C(2, –1) π/2
2 A(z)
D(1 + i)
B
Now, in DDMA,
i.e., AC = CB = 2 units Applying Coni method, we have

\ Least value of z = OA = OC - AC = 5 - 2 z - (2 - i ) AM ip / 2 1
= e = i
(1 + i ) - (2 - i ) DM 2
and greatest value of z = OB = OC + CB = 5 + 2
i i 3
Þ z -2+i = ( - 1 + 2i ) = - - 1 or z = 1 - i
Hence, greatest value of z is 5 + 2 and least value of z 2 2 2
is 5 - 2. 3 i
\ A º 1 - i or 3 -
2 2
[if positions of A and C interchange]
Chap 01 Complex Numbers 43

b y Example 87. Find the maximum and minimum


If z ± = a , then the greatest and least values of | z |
z 1
values of z satisfying z + = 2.
a + (a 2 + 4 | b |) - a + (a 2 + 4 | b | ) z
are and ,
2 2 Sol. Here, b = 1 and a = 2
respectively.
2 + (4 + 4)
b b \ Maximum and minimum values of z are
Proof z± ³ z - 2
z z
- 2 + (4 + 4)
|b | and i.e., 1 + 2 and - 1 + 2, respectively.
Þ a³ z - 2
|z |
4
or -a£ z -
|b |
£a
y Example 88. If z + = 2, find the maximum and
|z | z
|b | minimum values of z .
Now, z - £a
|z | Sol. Here, b = 4 and a = 2.

Þ |z |2 - a |z | - |b | £ 0 \ Maximum and minimum values of z are


2 + ( 4 + 16) - 2 + ( 4 + 16)
a - (a 2 + 4 | b| ) a + (a 2 + 4 | b | ) and
\ £ |z | £ 2 2
2 2 i.e. 1 + 5 and - 1 + 5, respectively.
a + (a 2 + 4 | b| )
or 0 £ |z | £ …(i) y Example 89. If z ³ 3, then determine the least
2
|b | 1
and |z | - ³ - a Þ |z |2 + a |z | - |b | ³ 0 value of z + .
|z | z
- a + (a 2 + 4 | b | ) 1 1 1
\ |z | ³ …(ii) Sol. Q z+ ³ z - = z - …(i)
2 z z z
From Eqs. (i) and (ii), we get 1 1 1 1
Q z ³3 Þ £ or - ³-
- a + (a 2 + 4 | b | ) a + (a 2 + 4 | b | ) z 3 z 3
£ |z | £
2 2 1 1 8 1 8
\ z - ³3- = Þ z - ³
2 z 3 3 z 3
a + (a + 4 | b | )
Hence, the greatest value of | z | is
2 1 8
2
or z - ³ …(ii)
- a + (a + 4 | b | ) z 3
and the least value of | z | is .
2 From Eqs. (i) and (ii), we get
1 1 8
Corollary For b = 1, z± =a z+ ³
z z 3
- a + (a 2 + 4 ) a + (a 2 + 4 ) 1 8
Then, £ z £ \ Least value of z + is .
2 2 z 3
44 Textbook of Algebra

#L Exercise for Session 4


4
1 If z1, z 2, z 3 and z4 are the roots of the equation z 4 = 1, the value of S z i 3 is
i =1

(a) 0 (b) 1 (c) i , i = -1 (d) 1 + i , i = -1

2 If z1, z 2, z 3, ... , z n are n, nth roots of unity, then for k = 1, 2, 3, ... , n


(a) z k = k z k + 1 (b) z k + 1 = k z k

(c) z k + 1 = z k + z k - 1 (d) z k = z k + 1

3 If 1, a1, a 2, a 3, ... , a n - 1 are n, nth roots of unity, then (1 - a1) (1 - a 2 ) (1 - a 3 ) ... (1 - a n - 1) equals to
(a) 0 (b) 1 (c) n (d) n 2
6
æ æ 2 pk ö æ 2 pk ö ö
4 The value of å çsin çè
è 7
÷ - i cos ç
ø
÷ , where i = - 1, is
è 7 ø ÷ø
k =1

(a) - 1 (b) 0 (c) - i (d) i

5 2
If a ¹ 1 is any nth root of unity, then S = 1 + 3 a + 5 a + ... upto n terms is equal to
2n 2n n n
(a) (b) - (c) (d) -
1- a 1- a 1- a 1- a

6 If a and b are real numbers between 0 and 1, such that the points z1 = a + i ,z 2 = 1+ bi and z 3 = 0 form an
equilateral triangle, then
(a) a = b = 2 + 3 (b) a = b = 2 - 3
(c) a = 2 - 3, b = 2 + 3 (d) None of these

7 If z = 2, the points representing the complex numbers - 1 + 5z will lie on


(a) a circle (b) a straight line (c) a parabola (d) an ellipse

8 If z - 2 / z - 3 = 2 represents a circle, then its radius is equal to


1 3 2
(a) 1 (b) (c) (d)
3 4 3

9 If centre of a regular hexagon is at origin and one of the vertex on argand diagram is 1 + 2i , where i = - 1, its
perimeter is
(a) 2 5 (b) 6 2 (c) 4 5 (d) 6 5

10 If z is a complex number in the argand plane, the equation z - 2 + z + 2 = 8 represents


(a) a parabola (b) an ellipse (c) a hyperbola (d) a circle

11 If z - 2 - 3i + z + 2 - 6i = 4, where i = - 1, then locus of P (z ) is


(a) an ellipse (b) f
(c) line segment of points 2 + 3i and - 2 + 6i (d) None of these

12 Locus of the point z satisfying the equation iz - 1 + z - 1 = 2, is (where, i = - 1)


(a) a straight line (b) a circle (c) an ellipse (d) a pair of straight lines

13 If z, iz and z + iz are the vertices of a triangle whose area is 2 units, the value of z is
(a) 1 (b) 2 (c) 4 (d) 8
4
14 If z - = 2, the greatest value of z is
z
(a) 5 - 1 (b) 3 + 1 (c) 5 + 1 (d) 3 - 1
Shortcuts and Important Results to Remember
1 z1 - z2 £ z1 + z2 £ z1 + z2 æ z - z1 ö
12 If arg ç ÷ = a (fixed), then the locus of z is a segment
è z - z2 ø
Thus, z1 + z2 is the greatest possible value of
z1 + z2 and z1 - z2 is the least possible value of of circle. P(z)
z1 + z2 . P(z)
α
b α
If z ± = a, then the greatest and least values of z are
z
a + (a2 + 4| b|) - a + (a2 + 4| b|) A(z1)
and , respectively.
2 2
B(z2)
z1 + ( z12 2
- z2 ) + z2 - ( z12 2
- z2 ) æ z - z1 ö
If arg ç ÷ = ± p / 2 , the locus of z is a circle with z1
è z - z2 ø
= z1 + z2 + z1 - z2
and z2 as the vertices of diameter.
z1 + z2 = z1 + z2 Û arg ( z1 ) = arg ( z2 )
æ z - z1 ö
14 If arg ç ÷ = 0 or p, the locus of z is a straight line
i.e. z1 and z2 are parallel. è z - z2 ø
passing through z1 and z2 .
z1 + z2 = z1 - z2 Û arg ( z1 ) - arg ( z2 ) = p
15 If three complex numbers are in AP, they lie on a straight
6 z1 + z2 = z1 - z2 Û arg ( z1 ) - arg ( z2 ) = ± p / 2 line in the complex plane.
16 If three points z1, z2 , z3 connected by relation
7 If z1 = z2 and arg ( z1 ) + arg ( z2 ) = 0, then z1 and z2 are a z1 + b z2 + c z3 = 0, where a + b + c = 0, the three points
conjugate complex numbers of each other. are collinear.
2 2 17 If z1, z2 , z3 are vertices of a triangle, its centroid
The equation z - z1 + z - z2 = k, k Î R will
1
represent a circle with centre at ( z1 + z2 ) and radius is
2 z0 =
z1 + z2 + z 3
, circumcentre z1 =
å| z1|2 ( z2 - z3 ) ,
1 2 1 2
3 å z1( z2 - z3 )
2 k - z1 - z2 provided k ³ z1 - z2 .
2 2
orthocentre z =
å z1( z2 - z3 ) + å| z1|2 ( z2 - z3 )
9 Area of triangle whose vertices are z, iz and z + iz, å ( z1 z2 - z1 z2 )
1 z1 z1 1
where i = - 1, is | z|2 .
2 1
and its area = | z2 z2 1 |.
4
10 Area of triangle whose vertices are z, w z and z + w z is z3 z3 1
3 2
z , where w is cube root of unity. 18 If| z1| = n1,| z2 | = n2 ,| z 3| = n 3 , ...,| z m | = n m,
4

11 arg ( z ) - arg (- z ) = p or - p according as arg ( z ) is n12 n22 n 23 n2


then + + + ... + m = | z1 + z2 + z3 + ... + zm|.
positive or negative. z1 z2 z3 zm
JEE Type Solved Examples :
Single Option Correct Type Questions
n This section contains 10 multiple choice examples. a b c
Sol. (a) We have, + + =1+i
Each example has four choices (a), (b), (c) and (d) out of a1 b1 c 1
which ONLY ONE is correct.
On squaring both sides, we get
a2 b2 c 2 æ ab bc ca ö
l Ex. 1 If z1 and z 2 be the n th root of unity which subtend + + + 2ç + + ÷
a12 b12 c 12 è a1b1 b1c 1 c 1a1 ø
right angled at the origin. Then, n must be of the form
(a) 4k + 1 (b) 4k + 2 (c) 4k + 3 (d) 4k = 1 + i 2 + 2i
Sol. (d) The nth roots of unity is given by a2 b2 c2 abc æ c 1 a1 b1 ö
Þ + + +2 ç + + ÷
æ 2r p ö æ 2r p ö 2r pi /n a12 b12 c 12 a1b1c 1 è c a bø
cos ç ÷ + i sin ç ÷ =e ,
è n ø è n ø = 1 - 1 + 2i
where r = 0, 1, 2, K, (n - 1) a2 b2 c2
2 pr 1 i / n 2 pr 2 i / n
Þ + + + 0 = 2i
So, let z1 = e and z 2 = e , where 0 £ r1, r 2 < n , a12 b12 c 12
r1 ¹ r 2 . a2 b2 c2
\ + + = 2i
It is given that the line segment joining the points having a12 b12 c 12
affixes z1 and z 2 , subtends a right angled at the origin.
Therefore, l Ex. 4 Let z and w be complex numbers. If Re( z ) = | z - 2|,
æz ö p p
arg ç 1 ÷ = ± Re( w ) = | w -2 | and arg ( z - w ) = , the value of Im ( z + w ) is
èz2 ø 2 3
2pr1 2pr 2 p 1 2 4
Þ - =± (a) (b) (c) 3 (d)
n n 2 3 3 3
Þ n = ± 4(r1 - r 2 ) Sol. (d) Let z = x + iy , x, y Î R and i = -1
\ n = 4k , where k = ± (r1 - r 2 ) Q Re(z ) = | z - 2|
Þ x = ( x - 2) 2 + y 2
z -1
l Ex. 2 If | z | =1 and w = (where z ¹ -1), then Re( w ) is Þ y 2 = 4( x - 1)
z +1
\ z = 1 + t 2 + 2ti , parametric form and let w = p + iq
1
(a) 0 (b) - Similarly, w = 1 + s 2 + 2si
| z + 1|2
\ z - w = (t 2 - s 2 ) + 2i (t - s )
½ z ½ ½× 1 2
(c)½ (d) p
½z + 1½ | z + 1|
2
| z + 1|2 Þ arg(z - w) = [given]
3
Sol. (a) We have, | z | = 1.
æ 2 ö p 2
Let z = e i q , where q Î R and i = -1. \ tan -1 ç ÷= Þ = 3
èt + s ø 3 t +s
z -1 eiq -1 æq ö
Then, w= = = i tan ç ÷ Þ (t + s ) =
2
z +1 eiq +1 è2ø
3
\ Re( w) = 0 Now, z + w = 2 + t 2 + s 2 + 2i (t + s )
4
l Ex. 3 If a, b, c, a 1 , b1 and c 1 are non-zero complex \ Im(z + w) = 2(t + s ) =
3
a b c a b c
numbers satisfying + + = 1 + i and 1 + 1 + 1 = 0,
a1 b1 c 1 a b c æ z - 3ö p
a 2
b 2
c 2
l Ex. 5 The mirror image of the curve arg ç ÷= ,
where i = -1, the value of + + is è z -i ø 6
2 2
a 1 b1 c 12 i = -1 in the real axis, is
(a) 2i (b) 2 + 2i (c) 2 (d) None of these
Chap 01 Complex Numbers 47

æz + 3ö p æz - 3ö p 1 æ 1ö
(a) arg ç ÷= (b) arg ç ÷= \ a = z 2017 + = - ç w + ÷ = - ( w + w2 ) = 1
èz +iø 6 èz +iø 6 z 2017 è wø
2n 4× 2 n - 4 n-4
æz +iö p æz +iö p and 2 =2 = 162 has last digit 6.
(c) arg ç ÷= (d) arg ç ÷=
èz + 3ø 6 èz - 3ø 6 \ b=6-1=5
Sol. (d) Q The image of z in the real axis is z. Hence, a 2 + b 2 = 12 + 52 = 26
The image is given by
æz - 3ö p l Ex. 8 If w is complex cube root of unity and a, b, c are
arg ç ÷=
èz - i ø 6 such that 1 + 1 + 1 = 2 w 2 and
æz - 3ö p a +w b+w c +w
Þ - arg ç ÷= [Q arg(z ) = - arg(z )]
èz + i ø 6 1 1 1
+ + = 2 w, then the value of
2 2
æz + i ö p é æ z1 ö æz2 öù a +w b+w c + w2
Þ arg ç ÷= êQ arg çè z ÷ø = - arg çè z ÷ø ú
èz - 3ø 6 ë 1 û
1 1 1
2 + + is equal to
a +1 b +1 c +1
æz +iö p (a) -2 (b) -1
(c) 1 (d) 2
l Ex. 6 The mirror image of the curve arg ç ÷= , 1 1 1 2
è z - 1ø 4 Sol. (d) Q + + = 2 w2 =
a+w b+w c +w w
i = -1 in the line x - y = 0, is
1 1 1 2
æz + iö p æ z + 1ö p and 2
+ 2
+ 2
= 2w =
(a) arg ç ÷= (b) arg ç ÷= a+w b+w c +w w2
è z + 1ø 4 èz - iø 4
It is clear that, w and w2 are the roots of the equation
æz - iö p æz + iö p 1 1 1 2
(c) arg ç ÷= (d) arg ç ÷= + + = …(i)
è z + 1ø 4 è z - 1ø 4 a+x b+x c +x x
Sol. (c) Q The image of z in the line y = x is iz. Þ x å (b + x )(c + x ) = 2(a + x )(b + x )(c + x )
\ The image of the given curve is Þ x 3 - (ab + bc + ca )x - 2abc = 0
æiz + i ö p
arg ç ÷= Q Coefficient of x 2 = 0, the sum of roots = 0
èi z - 1ø 4
Þ a + w + w2 = 0 Þ a - 1 = 0
æz + 1ö p
Þ arg ç ÷= \ a =1
èz + i ø 4
\ Third root is 1.
æz + 1ö p From Eq. (i), we get
Þ - arg ç ÷= [Q arg( z ) = - arg(z )]
èz - i ø 4 1 1 1
+ + =2
æz - i ö p é æz ö æz öù a+1 b+1 c +1
Þ arg ç ÷= êQ arg ç 1 ÷ = - arg ç 2 ÷ ú
èz + 1ø 4 ë è z2 ø è z1 ø û
l Ex. 9 If a, b and c are distinct integers and w( ¹1) is a
1 1 cube root of unity, then the minimum value of
l Ex. 7 If z + = 1 and a = z 2017 + and b is the last
z z 2017 | a + bw + cw 2 | + | a + bw 2 + c w |, is
n
digit of the number 2 2 -1,, when the integer n >1, the value (a) 2 3 (b) 3 (c) 4 2 (d) 2
2
2
of a + b is 2 Sol. (a) Let z = a + bw + c w . Then,
(a) 23 (b) 24 (c) 26 (d) 27 | z | 2 = zz = (a + bw + cw2 )(a + b w + c w2 )
1
Sol. (c) Qz + = 1 Þ z2 - z + 1 = 0 = (a + bw + cw2 )(a + bw2 + cw)
z
= a 2 + b 2 + c 2 - ab - bc - ca
-( - 1 ) ± ( 1 - 4 )
\ z= = - w, - w2 1
2 = [(a - b )2 + (b - c )2 + (c - a )2 ]
2
[w is cube root of unity]
éQ a ¹ b ¹ c ù
and z 2017 = ( - w)2017 = - w, 1 ê\ | a - b | ³ 1, | b - c | ³ 1ú
Þ |z |2 ³ ´ 6 = 3
z 2017 = ( - w2 )2017 = - w2 2 ê ú
êë and | a - c | ³ 2 úû
48 Textbook of Algebra

\ |z | ³ 3 Sol. (b) Q | 3 - i (z - 1)| = | - i (z - 1 - 3i )| = | -i || z - 1 - 3i |


\ | a + bw + cw2 | + | a + bw2 + cw| = | z - 1 - 3i | …(i)
= | a + bw + cw2 | + | a + b w2 + c w| and | z - 1 - 3i | = |(z - 2i ) + ( -1 - i )| £ | z - 2i | + | -1 - i |
= | a + bw + cw2 | + | a + bw + cw2 | [Q | z1 + z 2 | £ | z1 | + | z 2 | ]
= 2| a + bw + cw2 | = 2| z | ³ 2 3 \ | z - 1 - 3i | £ | z - 2i | + 2
Hence, the minimum value of | a + bw + cw2 | + | a + bw2 + cw|
Þ | z - 1 - 3i | £ 2 + 2 [Q | z - 2i | £ 2 ]
is 2 3.
Þ | z - 1 - 3i | £ 2 2
l Ex. 10 If | z - 2i | £ 2, where i = -1, then the maximum From Eq. (i), we get
value of | 3 - i ( z - 1) | , is | 3 - i (z - 1)| £ 2 2
(a) 2 (b) 2 2 (c) 2 + 2 (d) 3 + 2 2 Hence, the maximum value of | 3 - i (z - 1)| is 2 2.

JEE Type Solved Examples :


More than One Correct Option Type Questions
n This section contains 5 multiple choice examples. Each Q (z 2 - z 3 ) = (1 + i )(z1 - z 3 )
example has four choices (a), (b), (c) and (d) out of which B(z2)
more than one may be correct.

l Ex. 11 If z 1 = a + ib and z 2 = c + id are complex numbers


such that | z 1 | = | z 2 | = 1 and Re( z 1 z 2 ) = 0, where i = -1,
then the complex numbers w1 = a + ic and w 2 = b + id
π/2
satisfy A(z1)
C(z3)
(a) | w1| = 1 (b) | w2 | = 1
(c) Re( w1 w2 ) = 0 (d) None of these Þ ( z 2 - z1 ) = i ( z1 - z 3 )
Þ ( z 2 - z1 ) = - i ( z 3 - z1 )
Sol. (a, b, c)
or ( z 3 - z1 ) = i ( z 2 - z1 )
Q | z1| = 1, | z 2 | = 1 Þ z1 = CiS q, z 2 = CiS f
z 3 - z1
Re(z1z 2 ) = Re(CiS (q - f )) = 0 [given] Þ = e i p/ 2
z 2 - z1
p p p
Þ cos(q - f ) = 0 Þ q - f = Þ f=q - AC is obtained by rotating AB about A through
2 2 2
anti-clockwise.
and a = cosq, b = sinq, c = cos f, d = sin f p
\ w1 = a + ic = cos q + i cos f = cos q + i sin q \ AB = AC , ÐCAB =
2
é pù
êQ f = q - 2 ú
Hence, z1, z 2 , z 3 form isosceles right angled triangle.
ë û
and w2 = b + id = sin q + i sin f = sin q - i cos q l Ex. 13 If z satisfies the inequality | z - 1 | < | z + 1 |, then
é pù
êQ f = q - 2 ú one has
ë û
(a) | z - 2 - i | < | z + 2 - i |, i = -1
\ | w1| = 1, | w2 | = 1
p
and Re ( w1 w2 ) = Re (cos q + i sin q )(sin q + i cos q ) = 0 (b) |arg( z + i )| < , i = -1
2
l Ex. 12 The complex numbers z 1 , z 2 , z 3 satisfying (c) Re( z ) < 0
( z 2 - z 3 ) = (1 + i )( z 1 - z 3 ), where i = -1, are vertices of a (d) Im(i z ) > 0, i = -1

triangle which is Sol. (a, b, d)


(a) equilateral (b) isosceles On putting z = x + iy in the given relation, we have
(c) right angled (d) scalene ( x - 1) 2 + y 2 < ( x + 1) 2 + y 2
Sol. (b,c) Þ x >0
Chap 01 Complex Numbers 49

i.e. Re(z ) > 0 …(i) Case II When y = - x , then 2xy = ( x - 1)2 + y 2 reduces to
and on putting z = x + iy in alternate (a), then - 2x 2 = ( x - 1) 2 + x 2
( x - 2) 2 + ( y - 1) 2 < ( x + 2) 2 + ( y - 1) 2 Þ ( x - 1) 2 + 3x 2
= 0 which is not possible. …(ii)
Þ x >0 [from Eq. (i)] 1+i
which is true. From Eqs. (i) and (ii), we get z =
2
\Real part of (z + i ) = x > 0, i.e., no real and no purely imaginary roots
p p
then arg(z + i ) lies between - and and |z | =
1
<1
2 2
p 2
and hence |arg (z + i )| <
2 l Ex. 15 Let z 1 and z 2 be two complex numbers represented
and Im(i z ) = Im(i ( x - iy )) = Im(y + ix )
= x >0 [from Eq. (i)] by points on circles | z | =1 and | z | = 2 respectively, then
which is true. (a) max.| 2z 1 + z 2 | = 4 (b) min.| z 1 - z 2 | = 1
½ 1½ ½ 2½
(c)½z 2 + ½ £ 3 (d)½z 1 + ½ £ 2
l Ex. 14 The equation z 2 - i| z - 1 | 2 = 0, where i = -1, has ½ z 1½ ½ z 2½

(a) no real root Sol. (a, b, c, d)


(b) no purely imaginary root Q | z1| = 1 and | z 2 | = 2
(c) all roots inside | z | = 1 \ |2z1 + z 2 | £ | 2z1| + | z 2 | = 2| z1| + | z 2 |
(d) atleast two roots Þ | 2z1 + z 2 | £ 2| z1| + | z 2 | = 2 + 2 = 4
Sol. (a, b, c) \ | 2z1 + z 2 | £ 4
On putting z = x + iy , we have Þ max.|2z1 + z 2 | = 4 [alternate (a)]
( x + iy )2 - i | x + iy - 1| 2 = 0 and | z1 - z 2 | ³ || z1| - | z 2 || = |1 - 2| = 1
2 Þ | z1 - z 2 | ³ 1
Þ x - y 2 + 2ixy - i (( x - 1)2 + y 2 ) = 0
\ min.| z1 - z 2 | = 1 [alternate (b)]
On comparing real and imaginary parts, we get
½
½z 2 + 1½ ½ £ | z 2 | +½
½ 1½½ = |z 2 | + 1 = 2 + 1 = 3
x 2 - y 2 = 0 and 2xy = ( x - 1)2 + y 2
½ z1½ ½z1½ | z1| 1
Case I When y = x , then 2xy = ( x - 1)2 + y 2 reduces to ½
\ ½z 2 + 1½ ½£ 3 [alternate (c)]
2x 2 = ( x - 1) 2 + x 2 ½ z1½
Þ 0 = - 2x + 1 ½
and ½z1 + 2 ½ ½ £ | z1| +½
½ 2½
½ = | z1| + 2 = 1 + 2 = 2
1
\ x = =y ½ z 2½ ½ 2½
z |z 2 | 2
2
½
½ 2½
1+i \ z1 + ½ £ 2 [alternate (d)]
Þ z = x + iy = …(i) ½ z 2½
2

JEE Type Solved Examples :


Passage Based Questions
n
This section contains 2 solved passages based upon each (b) (ab + ab ) (bc + bc ) + (ca - ca )2 = 0
of the passage 3 multiple choice examples have to be
answered. Each of these examples has four choices (a), (b), (c) (ab - ab ) (bc - bc ) + (ca + ca )2 = 0
(c) and (d) out of which ONLY ONE is correct. (d) (ab + ab ) (bc - bc ) + (ca - ca )2 = 0
Passage I (Ex. Nos. 16 to 18) Sol. (b) Q az 2 + bz + c = 0 …(i)
2
Consider a quadratic equation az + bz + c = 0, where a, b and c \ 2
az + bz + c = 0
are complex numbers. Þ a (z )2 + bz + c = 0
16. The condition that the equation has one purely For purely imaginary root,
imaginary root, is z = -z
2
(a) (ab - ab ) (bc + bc ) + (ca - ca )2 = 0 Then, az - bz + c = 0 …(ii)
50 Textbook of Algebra

From Eqs. (i) and (ii), we get Passage II (Ex. Nos. 19 to 21)
z2 z 1
= = Let P be a point denoting a complex number z on the complex
bc + bc ca - ca - ab - ab plane.
bc + bc ca - ca i.e. z = Re (z ) + i Im (z ), where i = -1
Þ z= =
ca - ca - ab - ab If Re (z ) = x and Im (z ) = y , then z = x + iy
2
\ (ab + ab ) (bc + bc ) + (ca - ca ) = 0 19. If P moves such that
17. The condition that the equation has one purely real |Re ( z )| + | Im( z )| = a (a Î R + )
root, is The locus of P is
(a) (ab + ab ) (bc - bc ) = (ca + ca )2 (a) a parallelogram which is not a rhombus
2 (b) a rhombus which is not a square
(b) (ab - ab ) (bc + bc ) = (ca + ca )
(c) a rectangle which is not a square
(c) (ab - ab ) (bc - bc ) = (ca - ca )2 (d) a square
(d) (ab - ab ) (bc - bc ) = (ca + ca )2 Sol. (d) Q |Re( z )| + |Im( z )| = a
Sol. (c) Q az 2 + bz + c = 0 …(i) Þ | x | + |y | = a

Þ az 2 + bz + c = 0 Y
a
Þ a (z )2 + bz + c = 0
For purely real root, z = z X′ X
–a O a
2
Then, az + bz + c = 0 …(ii)
From Eqs. (i) and (ii), we get –a
2 Y′
z z 1
= =
bc - bc ca - ca ab - ab \ Locus of P is a square.

bc - bc ca - ca 20. The area of the circle inscribed in the region denoted


Þ z= = by |Re ( z )| + |Im ( z )| =10 equals to
ca - ca ab - ab
(a) 50 p sq units (b) 100 p sq units
Þ (ab - ab ) (bc - bc ) = (ca - ca )2 (c) 55 sq units (d) 110 sq units
18. The condition that the equation has two purely Sol. (a) From above, a = 10
imaginary roots, is Diameter of circle = Distance between sides of square
a b c a b c = Length of side of square = a 2 = 10 2
(a) = = (b) - = =
a b c a b c or 2r = 10 2
a b c a b c
(c) = = - (d) = - = Þ r =5 2
a b c a b c
\ Area of a circle = pr 2 = 50p sq units
Sol. (d) Q az 2 + bz + c = 0 …(i)
21. Number of integral solutions satisfying the inequality
\ az 2 + bz + c = 0
|Re ( z )| + |Im ( z )| < 21, is
Þ a ( z )2 + bz + c = 0 (a) 841 (b) 839
Since, both roots are purely imaginary. (c) 840 (d) 842
Sol. (c) Q | x | + | y | < 21 Þ 0 £ | x | + | y | £ 20
\ z = -z
Then, az 2 - bz + c = 0 …(ii) If x > 0, y > 0, 0 £ x + y £ 20
Hence, Eqs. (i) and (ii) are identical. 21 × 20
Number of solutions = 21C 2 = = 210
a b c 2
\ =- =
a b c \ Total integral solutions = 4 ´ 210 = 840
Chap 01 Complex Numbers 51

JEE Type Solved Examples :


Single Integer Answer Type Questions
n This section contains 2 examples. The answer to each Y
example is a single digit integer ranging from 0 to 9
(both inclusive). B

l Ex. 22 If z 1 , z 2 Î C , z 12 + z 22 Î R , z 1 ( z 12 - 3 z 22 ) = 2 and X′ X
A′ S1 O S2 A
z 2 (3 z 12 - z 22 ) = 11, the value of z 12 + z 22 is
Sol. (5) We have, z1 (z12 - 3z 22 ) = 2 …(i) B′
Y′
and z 2 (3z12 - z 22 ) = 11 …(ii)
multiplying Eq. (ii) by i ( -1 ) and then adding in Eq. (i), we Also, | z - ( 3 + 5i )| + | z - ( 5 + 11i )| = 4 5
get represents an ellipse.
z13 - 3z1z 22 + i (3z12z 2 - z 32 ) = 2 + 11i \ |(3 + 5i ) - (5 + 11i )| = 4 + 36 = 40 < 4 5

Þ (z1 + iz 2 )3 = 2 + 11i …(iii) with foci S1(3, 5) and S 2 (5, 11).


Distance between foci = S1S 2 = 40 = 2 10 = Diameter of
Again, multiplying Eq. (ii) by (- i) and then adding in Eq. (i),
we get circle
2
z13 - 3z1z 22 - i (3z1 z 2 - z 23 ) = 2 - 11i i.e., 2ae = 2 10
Þ ae = 10 and 2a = 4 5 Þ a = 2 5
Þ (z1 - iz 2 )3 = 2 - 11i …(iv)
ae 1
Now, on multiplying Eqs. (iii) and (iv), we get \ e= =
a 2
(z12 + z 22 )3 = 4 + 121 = 125 = 53
æ 1ö
\ z12 + z 22 = 5 Now, b = a (1 - e 2 ) = 2 5 ç1 - ÷ = 10 = Radius of circle
è 2ø
l Ex. 23 The number of solutions of the equations \ Centre of the ellipse = Mid-point of S1 and S 2
| z - ( 4 + 8i )| = 10 and | z - (3 + 5i )| + | z - ( 5 + 11i )| = 4 5, 3 + 5i + 5 + 11i
= = 4 + 8i i.e., ( 4, 8)
where i = - 1. 2
which coincides with the centre of the circle and length of
Sol. (2) Here, | z - ( 4 + 8i )| = 10 minor-axis is equal to the radius of the circle. Hence, there
represents a circle with centre (4, 8) and radius 10. are only (2) two solutions of the given equations.

JEE Type Solved Examples :


Matching Type Questions
n
This section contains 2 examples. Examples 24 and 25 have three statements (A, B and C) given in Column I and four
statements (p, q, r and s) in Column II. Any given statement in Column I can have correct matching with one or more
statement(s) given in Column II.

l Ex. 24
Column I Column II
(A) If l and m are the greatest and least values of | z - 1 | , if | z + 2 + i | £ 1, where i = -1, then (p) l + m = rational
(B) If l and m are the greatest and least values of | z - 2 | , if | z + i | £ 1, where i = -1, then (q) l + m = irrational

(C) If l and m are the greatest and least values of | z + 2i | , if 1 £ | z - 1| £ 3, where i = -1, then (r) l - m = rational
(s) l - m = irrational
52 Textbook of Algebra

Sol. (A) ® (q, r); (B) ® (q, r); (C) ® (p, s) From the figure, the greatest value of | z - 2| = | w |
(A) Q |z + 2 + i | £ 1 = | w - 0| = OB
Þ |(z - 1) + (3 + i )| £ 1 = OP + PB = 5 + 1
Þ | w + (3 + i )| £ 1 where, w = z - 1 \ l = 5 +1
From the figure, the greatest value of and the least value of | z - 2| = | w |
| z - 1| = | w| = | w - 0| = OB = OP + PB = 10 + 1 = | w - 0| = OA = OP - AP = 5 - 1
\ l = 10 + 1 \ m= 5 -1
Y Þ l + m = ( 5 + 1) + ( 5 - 1) = 2 5 = irrational
and l - m = ( 5 + 1) - ( 5 - 1) = 2 = rational
B (–3, –1) Aliter
P Q |z + i | £ 1
1 A Þ |(z - 2) + (2 + i )| £ 1
X′ X Þ | w + (2 + i )| £ 1
O
where, w= z - 2 …(i)
and the least value of | z - 1| = | w| Q | w + (2 + i )| ³ || w| - | 2 + i ||
= | w - 0 | = OP - AP = 10 - 1 or | w + (2 + i )| ³ || w| - 5 | …(ii)
\ m = 10 - 1 From Eqs. (i) and (ii), we get
|| w| - 5| £ | w + 2 + i | £ 1
Þ l + m = ( 10 + 1) + ( 10 - 1) = 2 10 = irrational
Þ || w| - 5| £ 1
and l - m = ( 10 + 1) - ( 10 - 1) = 2 = rational
Aliter or - 1 £ | w| - 5 £ 1 or 5 - 1 £ | w| £ 5 + 1
Q |z + 2 + i | £ 1 \ l = 5 + 1 and m = 5 - 1
Þ |(z - 1) + (3 + i )| £ 1 Þ l + m = 2 5 = irrational
Þ | w + (3 + i )| £ 1 …(i) and l - m = 2 = rational
where, w = z - 1 (C) Q 1 £ | z - 1| £ 3
\ | w + (3 + i )| ³ || w| - | 3 + i || Þ 1 £ |(z + 2i ) - (1 + 2i )| £ 3
or | w + (3 + i )| ³ || w| - 10 | …(ii) Þ 1 £ | w - (1 + 2i )| £ 3 …(i)
From Eqs. (i) and (ii), we get where, w = z + 2i
|| w| - 10 | £ | w + 3 + i | £ 1
Y
Þ || w| - 10 | £ 1 A
or - 1 £ | w| - 10 £ 1
or 10 - 1 £ | w| £ 10 + 1
\ l = 10 + 1 and m = 10 - 1 P
(1, 2)
Þ l + m = 2 10 = irrational
O
and l - m = 2 = rational X
(B) Q |z + i | £ 1 B
Þ |(z - 2) + ( 2 + i )| £ 1
Þ | w + (2 + i )| £ 1 From the figure, the greatest value of | z + 2i | = | w|
where, w= z - 2 = | w - 0| = OA = OP + PA = 5 + 3
Y \ l =3+ 5
and the least value of | z + 2i | = | w|
(–2, –1)
= | w - 0| = OB = PB - OP = 3 - 5
B
P \ m=3- 5
1 A Þ l + m = (3 + 5 ) + (3 - 5 ) = 6 = rational
X
O and l - m = (3 + 5 ) - (3 - 5 ) = 2 5 = irrational
Chap 01 Complex Numbers 53

l Ex. 25 = 3 - 3i, 1 + i, - 1 - i, - 3 + 3i
p p 3p 3p
Column I Column II \ Principal values of arg (z ) = - , , - ,
4 4 4 4
(A) If (3 - 4 i ) + (- 3 - 4 i ) = z, the principal (p) 0 æ 13 + 5 13 - 5 ö
value of arg (z) can be (where i = -1) (B) (5 + 12i ) = ± ç +i ÷
è 2 2 ø
(B) If (5 + 12i ) + (- 5 + 12i ) = z, the principal (q) p
± = ± (3 + 2i )
value of arg (z) can be (where i = -1) 4
æ 13 - 5 13 + 5 ö
p ( - 5 + 12i ) = ± ç +i ÷
(C) If (-15 + 8i ) + (- 15 - 8i ) = z, the principal (r) ± è 2 2 ø
2
value of arg (z) can be (where i = -1)
= ± (2 + 3i )
(s) 3p z = (5 + 12i ) + ( - 5 + 12i )
± Q
4
= ± (3 + 2i ) ± (2 + 3i )
Sol. (A) ® (q, s); (B) ® (q, s); (C) ® (p, r) = 5 + 5i , 1 - i , - 1 + i , - 5 - 5i
æ | z | + Re (z ) | z | - Re(z ) ö p p 3p 3p
Q z =±ç +i ÷, Im (z ) > 0 \ Principal values of arg (z ) = , - , , -
è 2 2 ø 4 4 4 4
æ 17 - 15 17 + 15 ö
æ | z | + Re(z ) | z | - Re (z ) ö (C) - 15 + 8i = ± ç +i ÷
=±ç -i ÷, Im (z ) < 0 è 2 2 ø
è 2 2 ø
= ± (1 + 4i )
æ 5+3 5 - 3ö
(A) (3 - 4i ) = ± ç -i ÷ = ± (2 - i ) and - 15 - 8i = - 15 + 8i = ± (1 + 4i )
è 2 2 ø
= ± (1 - 4i )
æ 5-3 5 + 3ö
( - 3 - 4i ) = ± ç -i ÷ = ± (1 - 2i ) Q z = ( - 15 + 8i ) + ( - 15 - 8i )
è 2 2 ø
= ± (1 + 4i ) ± (1 - 4i ) = 2, 8i , - 8i , - 2
Q z = ( 3 - 4i ) + ( - 3 - 4i ) p p
\ Principal values of arg (z ) = 0, , - , p.
\ z = ± (2 - i ) ± (1 - 2i ) 2 2

JEE Type Solved Examples :


Statement I and II Type Questions
n
Directions Example numbers 26 and 27 are Sol. (c) Statement-1
Assertion-Reason type examples. Each of these examples AB = | z1 - z 2 | = | 4i | = 4,
contains two statements:
BC = | z 2 - z 3 | = | 4 | = 4,
Statement-1 (Assertion) and Statement-2 (Reason)
Each of these examples also has four alternative choices, CD = | z 3 - z 4 | = | - 4i | = 4
only one of which is the correct answer. You have to select DA = | z 4 - z1| = | - 4 | = 4
the correct choice as given below.
AC = | z1 - z 3 | = | 4 + 4i | = 4 2
(a) Statement-1 is true, Statement-2 is true; Statement-2
is a correct explanation for Statement-1 and BD = | z 2 - z 4 | = | 4 - 4i | = 4 2
(b) Statement-1 is true, Statement-2 is true; Statement-2
is not a correct explanation for Statement-1 Y
(c) Statement-1 is true, Statement-2 is false
(d) Statement-1 is false, Statement-2 is true z4 = –2 + 2i z1 = 2 + 2i
D A
l Ex. 26 Consider four complex numbers z 1 = 2 + 2i,
z 2 = 2 - 2i, z 3 = - 2 - 2i and z 4 = - 2 + 2i, where i = -1. X′ X
O
Statement-1 z 1 , z 2 , z 3 and z 4 constitute the vertices of a
square on the complex plane because C B
Statement-2 The non-zero complex numbers z , z , - z , - z z3 = –2 – 2i z2 = 2 – 2i
always constitute the vertices of a square. Y′
54 Textbook of Algebra

It is clear that, AB = BC = CD = DA and AC = BD Þ (| z1 | + | z 2 | )2 = | z1| 2 + | z 2 | 2 + 2 | z1| | z 2 | cos (q 1 - q 2 )


Hence, z1, z 2 , z 3 and z 4 are the vertices of a square. [from Eq. (i)]
\ Statement-1 is true.
Þ | z1 | 2 + | z 2 | 2 + 2 | z1 | | z 2 |
Statement-2 If z = a + ib
If a ¹ b = | z1 | 2 + | z 2 | 2 + 2 | z1 | | z 2 | cos (q 1 - q 2 )
Then, AB = | z - z | = |(a + ib ) - (a - ib )| = 2| b | \ cos (q 1 - q 2 ) = 1
BC = | z - ( - z )| = | z + z | = | a - ib + a + ib | = 2 | a |
\ AB ¹ BC z1 z2 O
Statement-2 is false. Þ q1 - q 2 = 0
or amp (z1 ) - amp (z 2 ) = 0
l Ex. 27 Consider z 1 and z 2 are two complex numbers \ Statement-1 is true.
such that | z 1 + z 2 | = | z 1 | + | z 2 | Statement-2 Since, z1, z 2 and O (origin) are collinear,
Statement-1 amp ( z 1 ) - amp ( z 2 ) = 0 then
Statement-2 The complex numbers z 1 and z 2 are collinear æ O - z1 ö
amp ç ÷ =0
with origin. èO - z2 ø
Sol. (b) Statement-1 æz ö
Þ amp ç 1 ÷ = 0
Q | z1 + z 2 | = | z1| + | z 2 | …(i) èz2 ø
If amp (z1 ) = q 1 and amp (z 2 ) = q 2 , then Þ amp (z1 ) - amp (z 2 ) = 0
| z1 + z 2 | 2 = | z1 | 2 + | z 2 | 2 + 2 | z1 | | z 2 | cos (q 1 - q 2 ) \ Statement-2 is true, which is not a correct explanation of
Statement-1.

Subjective Type Examples


n
In this section, there are 24 subjective solved examples. Hence, the least value of z1 + z 2 is 19 and the greatest
value of | z1 + z 2 | is 31.
l Ex. 28 If z - i Re ( z ) = z - Im ( z ) , then prove that z
lies on the bisectors of the quadrants, where i = -1. l Ex. 30 Let S denotes the real part of the complex number
Sol. Let z = x + iy , where x , y Î R and i = -1 5 + 2i 20 + 5i
z= + + 3i, where i = -1, K denotes the sum
2 - 5i 7 + 6i
\ Re (z ) = x and Im (z ) = y
Then, z - i Re (z ) = z - Im (z ) of the imaginary parts of the roots of the equation
Þ x + iy - ix = x + iy - y z 2 - 8 (1 - i ) z + 63 - 16i = 0 and G denotes the value of
2012
Þ x - i ( x - y ) = ( x - y ) + iy
S i r , where i = -1, find the value of S - K + G.
Þ x 2 2
+ (x - y ) = (x - y ) + y 2 2 r =4

Þ x 2
+ ( x - y )2 = ( x - y )2 + y 2 Sol. For S,
(5 + 2i ) (20 + 5i )
Þ x 2
= y 2 or y = ± x z= + + 3i
(2 - 5i ) (7 + 6i )
Hence, z lies on the bisectors of the quadrants.
(5 + 2i ) (2 + 5i ) (20 + 5i ) (7 - 6i )
= + + 3i
l Ex. 29 Find the greatest and the least values of z 1 + z 2 , 29 85
0 + 29i 170 - 85i
if z 1 = 24 + 7i and z 2 = 6, where i = -1. = + + 3i
29 85
Sol. Q z1 = 24 + 7i = i + 2 - i + 3i = 2 + 3i
\ z1 = (24 )2 + (7 )2 = 25 \ Re (z ) = 2 \ S =2
For K,
z1 - z 2 £ z1 + z 2 £ z1 + z 2
Put z = x + iy in the given equation, then
Þ 25 - 6 £ z1 + z 2 £ 25 + 6 ( x + iy )2 - 8 (1 - i ) ( x + iy ) + 63 - 16i = 0
or 19 £ z1 + z 2 £ 31
Chap 01 Complex Numbers 55

On comparing the real and imaginary parts, we get 1


l Ex. 32 If arg ( z 1/ 3 ) = arg ( z 2 + z z 1/ 3
), find the value
x 2 - y 2 - 8 ( x + y ) + 63 = 0 …(i) 2
of z .
and xy + 4 ( x - y ) = 8 …(ii)
1
On substituting the value of x from Eq. (ii) in Eq. (i), we get Sol. We have, arg (z 1 / 3 ) = arg (z 2 + z z 1 / 3 )
2
y 4 + 16y 3 + ... = 0
Þ 2 arg (z 1/ 3 ) = arg (z 2 + z z 1/ 3 )
\ K = - 16
2012 2009 Þ arg (z 2 / 3 ) = arg (z 2 + z z 1/ 3 ) [by property]
For G, G= S r
i = S i r +3
=i 1+ 3
+ 0 =1 Þ 2
arg (z + z z 1/ 3
) - arg (z 2/3
)=0
r =4 r =1

\ S - K + G = 2 - ( - 16) + 1 = 19 æz + z z 2 1/ 3 ö
Þ arg ç ÷ =0 [by property]
è z2/ 3 ø
l Ex. 31 If z - 1 = 1, where z is a point on the argand
æ z ö
z -2 Þ arg çz 4 / 3 + 1/ 3 ÷ = 0
plane, show that = i tan (arg z ), where i = -1 . è z ø
z
z
Sol. Given, z -1 =1 Þ z -1
2
=1 Þ z 4/3 + 1/ 3
is purely real.
z
Þ ( z - 1) ( z - 1) = 1 Þ z z - z - z = 0 æ z ö
Þ Im çz 4 / 3 + 1/ 3 ÷ = 0
z è z ø
Þ (z + z ) = z z Þ +1=z
z
z æ 4/3 z ö æ z ö
Þ _ =z -1 …(i) çz + 1/ 3 ÷ - çz 4 / 3 + 1/ 3 ÷
z è z ø è z ø =0
æ Im (z ) ö Þ
Now, RHS = i tan (arg z ) = i ç ÷ 2i
è Re (z ) ø —
ì z -z ü z (z )
ïï ïï Þ z4/ 3 + = (z )4 / 3 +
æ z -z ö z 1/ 3 (z )1 / 3
= i í 2i ý = i ç ÷
z +z è i (z + z ) ø
ï ï (z ) (z )1 / 3 z (z )1 / 3
ïî 2 ïþ Þ z4/ 3 + 2/3
= (z )4 / 3 + 2/3
z z z
-1
z -z z ( z - 1) - 1 z - 2
= = = = [from Eq. (i)] [Qz 1 / 3 (z )1 / 3 = (z z )1 / 3 = z
2/3
]
z + z z + 1 ( z - 1) + 1 z
z 1
Þ z 4 / 3 - (z )4 / 3 - ((z )4 / 3 - (z )4 / 3 ) = 0
= LHS 2/3
z
Aliter
We have, z - 1 = 1 i.e.(z - 1) is unimodular, so we can take é 1 ù
Þ {z 4 / 3 - (z )4 / 3 } ê1 - ú =0
z - 1 = cos q + i sin q êë z
2 /3
úû
\ z - 2 = - 1 + cos q + i sin q 2/3
q q q \ z =1 [Qz ¹ z ]
= - 2 sin 2 + 2i sin cos
2 2 2 Therefore, z =1
2 2 q q q
= 2i sin + 2i sin cos
2 2 2 l Ex. 33 C is the complex numbers f : C ® R is defined by
q æ q qö f ( z ) = z 3 - z + 2 . Find the maximum value of f ( z ), if
or z - 2 = 2i sin ç cos + i sin ÷ …(i)
2 è 2 2ø z =1.
and z = 1 + cos q + i sin q Sol. Q z =1
q q q
= 2 cos 2 + 2i sin cos \ z = e iq
2 2 2
q æ q qö \ f ( e iq ) = e 3iq
- e iq + 2 = e 2 iq
( e iq - e - iq ) + 2
z = 2 cos ç cos + i sin ÷ …(ii)
2 è 2 2ø 2iq
= e × 2i sin q + 2
From Eqs. (i) and (ii), we get
z -2 q = (cos 2q + i sin 2q ) × 2i sin q + 2
= i tan
z 2 = (2 - 2 sin 2q sin q ) + 2i sin q cos 2q
z -2 = 2 (1 - sin 2q sin q ) + i sin q cos 2q
Therefore, = i tan (arg z ) [Q arg (z ) = q /2 from Eq. (ii)]
z = 2 (1 - sin 2q sin q )2 + (sin q cos 2q )2
56 Textbook of Algebra

= 2 (1 + sin 2 q - 2 sin q sin 2q ) II. Aliter


Here, OA = OB …(i)
= 2 1 + sin 2 q (1 - 4 cos q )
From Rotation theorem,
= 2 1 + (1 - cos q ) (1 + cos q ) (1 - 4 cos q ) z1 - 0 OA 2 pi / 3
= e
1 é 1ù z 2 - 0 OB
For maximum value, cosq = - êQ cos q ¹ - 1, 1, 4 ú
2 ë û z1 æ 2p 2p ö
Þ = ç cos + i sin ÷ [from Eq. (i)]
z2 è 3 3 ø
æ3ö æ1ö
\ Maximum value of f (z ) = 2× 1 + ç ÷ ç ÷ (3) = 13 1 i 3
è2ø è2ø Þ
z1
=- +
z2 2 2
l Ex. 34 Prove that the complex numbers z 1 and z 2 and æ z1 1 ö i 3
2p Þ ç + ÷= …(ii)
the origin form an isosceles triangle with vertical angle , if èz2 2ø 2
2 2 3
z 1 + z 2 + z 1 z 2 = 0. On squaring both sides in Eq. (ii), we get
2 2 2
Sol. Given, z1 + z 2 + z1z 2 = 0 Þ
z1
+
1 z1
+ =-
3
2
Þ (z1 - wz 2 ) (z1 - w2z 2 ) = 0 z2 4 z2 4
2
Þ z1 = wz 2 or z1 = w2z 2 z1 z1
Þ 2
+ +1=0
In the first case, z1 = wz 2 Þ z1 = w z 2 Þ z1 = z 2 z2 z2
2 2
Hence, two sides equal Þ z1 + z1z 2 + z 2 = 0
2 2
amp (z1 ) = amp (w) z 2 = amp (w) + amp (z 2 ) \ z1 + z 2 + z1z 2 = 0
2p
Þ amp (z1 ) = + amp (z 2 )
3 l Ex. 35 If a = e 2 pi / 7 , where i = -1 and
2p
Þ amp (z1 ) - amp (z 2 ) = 20
3 f (x ) = A 0 + å A k x k , then find the value of
2p k =1
So, the angle between two sides is .
3 f ( x ) + f (ax ) + f (a 2 x ) + ... + f (a 6 x ) independent of a.
Similarly, the other case Sol. Q a = e 2 pi / 7
I. Aliter
2 2 \ a 7 = e 2 pi = cos 2p + i sin 2p = 1 + 0 = 1 or a = (1)1/ 7
Given, z1 + z 2 + z1z 2 = 0
\ 1, a , a 2 , a 3 , a 4 , a 5 , a 6 are the seven, 7th roots of unity.
Þ (z1 - wz 2 ) (z1 - w2z 2 ) = 0
20 20
Þ z1 = wz 2 or z1 = w2z 2 Q f (x ) = A0 + å Ak x k = å Ak x k
k =1 k =0
In the first case, z1 = wz 2
é 2 pi ù Now, f ( x ) + f (ax ) + f (a 2 x ) + ... + f (a 6 x )
Þ ( z 1 - 0) = ( z 2 - 0) e 2 pi / 3 êQ w = e 3 ú 20
êë úû
¾® ¾®
= å Ak x k [( (1)k + (a )k + (a 2 )k + ... + (a 6 )k ]
k =0
Þ OA = OB e 2 pi / 3
¾® ¾® = A 0 x 0 (7 ) + A 7 x 7 (7 ) + A14 x 14 (7 )
2p
i.e., OA is obtained by rotating OB through angle of . = 7 ( A 0 + A 7 x 7 + A14 x 14 )
3
O é k k 2 k 6 k
ê Q(1) + (a ) + (a ) + ... + (a )
2π/3 ë
ì 7, k is multiple of 7 ù
=í ú
B(z2) A(z1)
î 0, k is not multiple of 7 û

2p
Þ OA = OB and ÐAOB = l Ex. 36 Show that all the roots of the equation
3
a 1 z 3 + a 2 z 2 + a 3 z + a 4 = 3, (where a i £ 1, i = 1, 2, 3, 4) lie
Thus, triangle formed by z1, z 2 and origin is isosceles with
2p outside the circle with centre at origin and radius 2 / 3.
vertical angle .
3 Sol. Given that, a1z 3 + a 2 z 2 + a 3z + a 4 = 3
Chap 01 Complex Numbers 57

We have, 3 = a1z 3 + a 2 z 2 + a 3z + a 4 On squaring both sides, we get


2
3 £ a1z 3 + a 2 z 2 + a 3z + a 4 (z 3 - z 2 )2 a æ a aö
= - 4 sin 2 ç cos + i sin ÷
( z 2 - z1 ) 2
2 è 2 2ø
Þ 3 £ a1 z 3 + a2 z 2 + a3 z + a4
2 a
3 2
= - 4 sin (cos a + i sin a )
Þ 3 £ a1 z + a2 z + a3 z + a4 2
3 2 [from De-Moivre’s theorem]
Þ 3£ z + z + z +1 [Q | ai | £ 1] 2
(z 3 - z 2 ) a æ z - z1 ö
Þ 3£1+ z + z
2
+ z
3
< 1+ z + z
2
+ z
3
+ ... = - 4 sin 2 ç 3 ÷ [from Eq. (ii)]
( z 2 - z1 ) 2 2 è z 2 - z1 ø
2 3
Þ 3<1+ z + z + z + ... Therefore, (z 2 - z 3 )2 = 4 sin 2 (a / 2) (z 3 - z1 ) (z1 - z 2 )
1 1
= Þ 3< Aliter
1 - |z | 1- z æp a ö
Q ÐABC = ç - ÷
è2 2ø
1 2
Þ 1- z < Þ - z <0 From Coni method, we have
3 3
æp aö
Þ z > 2 / 3 or z - 0 > 2 / 3 z1 - z 2 AB i çè 2 - ÷

Hence, all the roots lie in the exterior of circle, = e …(i)
z 3 - z 2 BC
z - 0 = 2 / 3.
æp a ö
and ÐACB = ç - ÷
l Ex. 37 If A, B and C represent the complex numbers è2 2ø
z 1 , z 2 and z 3 respectively on the complex plane and the From Coni method, we have
æp aö
1 z 2 - z 3 BC i çè 2 - 2 ÷ø
angles at B and C are each equal to ( p - a ), then prove = e …(ii)
2 z1 - z 3 AC
a
that ( z 2 - z 3 ) = 4 ( z 3 - z 1 ) ( z 1 - z 2 ) sin 2 æç ö÷ .
2 On dividing Eq. (ii) by Eq. (i), we get
è2ø (z 2 - z 3 )2 ( BC )2 æ BC ö
2
= =ç ÷ [Q AB = AC ]
Sol. It is given that, (z 3 - z1 ) (z1 - z 2 ) AB × AC è AB ø
2
p a æ ö
Ð ABC = ÐACB = - ç ÷
2 2 sin a
=ç ÷ [using sine rule]
Þ ÐA = a ç æp a ö÷
\ AC = AB …(i) ç sin çè - ÷ø ÷
è 2 2 ø
So, D ABC is an isosceles triangle. 2
Considering rotation of AB about A through angle a, we get æ æa ö æa ö ö
ç 2 sin ç ÷ cos ç ÷ ÷
è 2 ø è 2 ø÷ æa ö
A(z1) =ç = 4 sin 2 ç ÷
ç æa ö ÷ è2ø
ç cos ç ÷ ÷
a è è2ø ø
Therefore, (z 2 - z 3 )2 = 4 (z 3 - z1 ) (z1 - z 2 ) sin 2 a / 2

p– a p– a l Ex. 38 If z 1 and z 2 are two complex numbers such that


2 2 2 2
B(z2) C(z3) z1 - z 2 iz
= 1, then prove that 1 = k , where k is a real
z 3 - z1 z1 + z 2 z2
Þ =eia
z 2 - z1 number. Find the angle between the lines from the origin to
the points z 1 + z 2 and z 1 - z 2 in terms of k.
z 3 - z1
Þ = cos a + i sin a …(ii) z1 - z 2
z 2 - z1 Sol. (i) Given, =1
z1 + z 2
z 3 - z1
- 1 = cos a - 1 + i sin a z1
z 2 - z1 -1
z2
z3 - z2 a a a Þ =1
Þ = - 2 sin 2 + 2i sin cos z1
+1
z 2 - z1 2 2 2 z2
z3 - z2 a æ a aö
or = 2i sin ç cos + i sin ÷ …(iii) Þ
z1 z
-1 = 1 +1 …(i)
z 2 - z1 2 è 2 2ø z2 z2
58 Textbook of Algebra

On squaring Eq. (i) both sides, we have (ii) Now, let the angle between OB and OA be q, then from
z1
2
æz ö z
2
æz ö Coni method,
+ 1 - 2 Re ç 1 ÷ = 1 + 1 + 2 Re ç 1 ÷ z1 + z 2 - 0 OB iq
z2 èz2 ø z2 èz2 ø = e
z1 - z 2 - 0 OA
æ z1 ö
Þ 4 Re ç ÷ = 0 z 1 + z 2 iq
èz2 ø = e
z z1 - z 2
Þ 1 is purely imaginary number
z2 æ z1 + z 2 ö iq
Þ ç ÷ =e [from Eq. (i)]
z z è z1 - z 2 ø
Þ 1 can be written as i 1 = k , where k is a real number.
z2 z2
æ z1 ö
(ii) Let q be the angle between z1 - z 2 and z1 + z 2 , then ç + 1÷ - ki + 1
z ÷ = e iq Þ
Þ ç 2
= e iq [from Eq. (ii)]
Q(z1 + z2) ç z1 - 1 ÷ - ki - 1
ç ÷
è z2 ø
B(z1 + z2)

θ
O P(z1 – z2)

æ z1 ö θ
æ z1 + z 2 ö ç + 1÷ æ - ik + 1 ö
O A(z1 – z2)
z2
q = arg ç ÷ = arg ç ÷ = arg ç ÷ - 1 + ki
è z1 - z 2 ø ç 1 -1 ÷
z è - ik - 1 ø Þ = e iq
ç ÷ 1 + ki
è z2 ø
æ k 2 - 1 + 2ik ö ( - 1 + ki ) (1 - ki )
æ -1 + ik ö Þ = e iq
= arg ç ÷ = arg ç ÷ (1 + ki ) (1 - ki )
è 1 + ik ø è k 2 +1 ø
( k 2 - 1) 2ki
-1 æ 2k ö Þ + = e iq
Therefore, q = tan ç 2 ÷ 2
( k + 1) 1 + k2
èk - 1ø
k2 - 1
z1 - z 2 \ Re (e iq ) = cos q =
Aliter (i) Given, =1 …(i) k2 + 1
z1 + z 2
2k
z1 - z 2 cos a + i sin a and Im(e iq ) = sin q =
Let = k2 + 1
z1 + z 2 1
2k
(z1 - z 2 ) + (z1 + z 2 ) 1 + cos a + i sin a \ tan q =
Þ = [ by k2 - 1
(z1 + z 2 ) - (z1 - z 2 ) 1 - cos a - i sina
æ 2k ö
componendo and dividendo] Therefore, q = tan - 1 ç 2 ÷
èk -1ø
æa ö æa ö æa ö
2 cos 2 ç ÷ + 2i sin ç ÷ cos ç ÷
z1 è2ø è2ø è2ø
Þ = l Ex. 39 If z = x + iy is a complex number with rationals x
z2 a
æ ö a
æ ö æa ö
2 sin 2 ç ÷ - 2i sin ç ÷ cos ç ÷
è2ø è2ø è2ø and y and z =1, then show that z 2 n - 1 is a rational
æ
æa ö æa ö æa öö number for every n Î N .
ç cos ç ÷ + i sin ç ÷ ÷
2 cos ç ÷
è2ø
è è2ø è 2 øø
Þ = Sol. Since, z = 1, where z is unimodular
æa ö æ æa ö æa öö \ z = cos q + i sin q
-2i sin ç ÷ ç cos ç ÷ + i sin ç ÷ ÷
è2ø è è2ø è 2 øø As x and y are rational, cos q , sinq are rationals
æa ö æ 1ö
zn - z - n
n
cot ç ÷ \ z 2 n - 1 = z n çz n - n ÷ = z
z1 è2ø è z ø
Þ =-
z2 i = 1 2i sin nq
iz1 æa ö = 2 sin nq
Þ = - cot ç ÷ = k ( say) = real
z2 è2ø Since, sinnq is rational, therefore z 2 n - 1 is a rational
iz1
Hence, =k ...(ii) number.
z2
Chap 01 Complex Numbers 59

l Ex. 40 If a is a complex number such that a =1, then Therefore, z lies on the right bisector of the segment
connecting the points 0 + i × 0 and - 1 + 0 × i. Thus,
find the value of a, so that equation az 2 + z + 1 = 0 has one Re (z ) = - 1 / 2. Hence, roots are collinear and will have
purely imaginary root. their real parts equal to - 1 / 2. Hence, sum of the real parts
æ 1 ö
Sol. We have, az 2 + z + 1 = 0 …(i) of roots is ç - (n - 1)÷.
è 2 ø
On taking conjugate both sides, we get
Aliter
az 2 + z + 1 = 0
Q z n = (z + 1)n
_
n
Þ a(z )2 + z + 1 = 0 æz + 1ö z +1
Þ ç ÷ = 1 or = (1)1 / n
Þ a ( -z ) 2 + ( -z ) + 1 = 0 è z ø z
[since, z is purely imaginary, z = - z ] = (cos 0 + i sin 0)1 /n
or a z2 - z + 1 = 0 …(ii) = (cos 2r p + i sin 2r p)1/ n
Eliminating z from Eqs. (i) and (ii) by cross-multiplication 1 æ 2rp ö æ 2r p ö 2 r pi / n
rule, we get Þ 1+ = cos ç ÷ + i sin ç ÷=e
z è n ø è n ø
( a - a )2 + 2 (a + a ) = 0 r pi
1 æ pr ö
On dividing each by 4, we get or = (e 2r pi / n - 1) = e n × 2i sin ç ÷
2
z èn ø
æa - aö æa + a ö pr i
ç ÷ +ç ÷ =0 æ1ö 1 -
è 2 ø è 2 ø or z =-ç ÷i× ×e n
2
è2ø æ pr ö
æa - a ö æa + a ö sin ç ÷
Þ -ç ÷ +ç ÷ =0 èn ø
è 2i ø è 2 ø
æ pr pr ö
or - ( Im (a ))2 + Re (a ) = 0 …(iii) ç cos - i sin ÷
æi ö è n n ø
=-ç ÷×
Given, a =1 è2ø pr
sin
Let a = cos a + i sin a n
\ Re (a ) = cos a , Im (a ) = sin a 1
\ Re (z ) = - [ here r ¹ 0]
Then, from Eq. (iii), we get 2
- sin 2 a + cos a = 0 or cos 2 a + cos a - 1 = 0 where, r = 1, 2, 3, ... , n - 1
1 1 1
-1± 1+ 4 Sum of real parts of z = - - - - ... - (n - 1) times
\ cosa = 2 2 2
2
1
5 -1 = - (n - 1).
Only feasible value of cos a = 2
2
Hence, a = cos a + i sin a l Ex. 42 Prove that the angle between the line a z + a z = 0
æ 5 -1ö
where, a = cos - 1 ç ÷ and its reflection in the real axis is
è 2 ø ì 2 Re (a ) Im (a ) ü
q = tan -1 í

.
2
l Ex. 41 If n Î N >1, find the sum of real parts of the roots î {Im (a )} - {Re (a )} þ
Sol. Let z = x + iy , then equation a z + a z = 0 can be written as
of the equation z n = ( z + 1) n .
( a + a) x + i ( a - a) y = 0
Sol. The equation z n = (z + 1)n will have exactly n - 1 roots. æa + a ö æa - a ö
We have, Þ ç ÷x+ç ÷y =0
è 2 ø è 2i ø
n n
æz + 1ö æz + 1ö Þ {Re (a )} x + {Im (a )} y = 0
ç ÷ =1 Þ ç ÷ = 1
è z ø è z ø {Re (a )}
\ Slope of the given line (m ) = -
z +1 {Im (a )}
Þ =1 {Re (a )} {Re (a )}
|z | Then, tan (180°- f ) = - Þ tan f =
{Im (a )} {Im (a )}
Þ z +1 = z
Þ z - ( - 1) = z - 0 Hence, angle between the given line and its reflection in
real axis
60 Textbook of Algebra

æ3 4ö
\ z P = 20 (cos q + i sin q ) = 20 ç + i ÷

Imaginary axis
è5 5ø
\ z P = 12 + 16i
Inc ay
ide dr Similarly, zQ = - 12 + 16i
nt cte
az+ ra efle From the figure, E is the point with least modulus and D is
az= y R
0φ φ the point with maximum modulus.
Real axis ¾® ¾® ¾®
O φ Hence, z E = OE = OC - EC = 25i - 15i = 10i
¾® ¾® ¾®
and z D = OD = OC + CD = 25i + 15i = 40i
ì 2 tan f ü
= 2f = tan -1 {tan 2f } = tan -1 í 2 ý l Ex. 44 Two different non-parallel lines meet the circle
î 1 - tan f þ
ì | z | = r in the points a , b and c , d , respectively. Prove that these
{Re (a )} ü
ï 2 ï a -1 + b -1 - c -1 - d -1
ï {Im(a )} ï ì 2 Re (a ) Im (a ) ü lines meet in the point z given by z = ,
= tan -1 í 2ý
= tan -1 í 2 2 ý a -1b -1 - c -1d -1
ï 1 - {Re (a )} ï î {Im (a )} - {Re(a )} þ
ïî where a, b, c, d are complex constants.
{Im(a )} 2 ïþ
Sol. Let two non-parallel straight lines PQ , RS meet the circle
| z | = r in the points a, b and c , d , then
l Ex. 43 Among the complex numbers z which satisfies | a | = r , | b | = r , | c | = r and | d | = r
| z - 25i | £ 15, find the complex numbers z having
P a
(i) least positive argument. b S
(ii) maximum positive argument.
z
(iii) least modulus. d Q
R c
(iv) maximum modulus.
Sol. The complex numbers z satisfying the condition or |a |2 = |b |2 = |c |2 = |d |2 = r 2
| z - 25i | £ 15 …(i) Þ aa =bb =c c =d d =r2
r2 r2 r2 r2
D 40i \ a= ,b = ,c = and d =
Tangent from a b c d
origin For line PQ, points a, b and z are collinear, then
C 25i
z z 1
θ
Q P a a 1 =0
E b b 1
φ
θ Þ z ( a - b ) - z (a - b ) + (ab - ab ) = 0
O N
ær r2ö2 æ ar 2 br 2 ö
Þ z ç - ÷ - z (a - b ) + ç - ÷ =0
are represented by the points inside and on the circle of èa bø è b a ø
radius 15 and centre at the point C (0, 25). On dividing both sides by (b - a ), we get
The complex numbers having least positive argument and r2 r2
maximum positive arguments in this region are the points z +z - (a + b ) = 0
ab ab
of contact of tangents drawn from origin to the circle.
z z (a + b )
Here, q = Least positive argument Þ + - =0 …(i)
ab r 2 ab
and f = Maximum positive argument
Similarly, for line RS, we get
\ In DOCP , OP = (OC )2 - (CP )2 = (25)2 - (15)2 = 20 z z (c + d )
+ - =0 …(ii)
and sin q =
OP 20 4
= = cd r 2 cd
OC 25 5 On subtracting Eq. (ii) from Eq. (i), we get
4 æ4ö æ1 1 ö (a + b ) (c + d )
\ tan q = Þ q = tan -1 ç ÷ zç - ÷- + =0
3 è3ø è ab cd ø ab cd
Thus, complex number at P has modulus 20 and argument Þ z (a -1b -1 - c -1d -1 ) = a -1 + b -1 - c -1 - d -1
æ4ö
q = tan -1 ç ÷ a -1 + b -1 - c -1 - d -1
è3ø Therefore, z=
a -1b -1 - c -1d -1
Chap 01 Complex Numbers 61

l Ex. 45 If n is an odd integer but not a multiple of 3, then Sol. (i) Since, | z - 1 | + | z + 1 | = 4
prove that xy ( x + y ) ( x 2 + y 2 + xy ) is a factor of i.e., (distance of z from the point 1 + 0 × i ) + (distance
(x + y )n - x n - y n . of z from the point - 1 + 0 × i ) = 4 (constant)
i.e., The sum of the distances of z from two fixed
Sol. We have, xy ( x + y ) ( x 2 + y 2 + xy ) = xy ( x + y )
points 1 + 0 ×i and - 1 + 0 × i is constant, which is the
( x - w y ) ( x - w2y ) definition of an ellipse.
and let f ( x , y ) = ( x + y )n - x n - y n …(i) Therefore, locus of z satisfying the given condition
On putting x = 0 in Eq. (i), we get will be an ellipse with foci at 1 + 0 ×i and - 1 + 0 × i and
f (0, y ) = y n - 0 - y n = 0 centre at origin.
(ii) Given that,
\ x - 0 is a factor of Eq. (i). p
On putting y = 0 in Eq. (i), we get arg(z + i ) - arg(z - i ) =
2
f ( x , 0) = x n - x n = 0 æz + i ö p
\ y - 0 is a factor of Eq. (i).
or arg ç ÷= …(i)
èz - i ø 2
On putting x = - y in Eq. (i), we get
Let the points A and B have affixes i and - i and the
f ( - y , y ) = ( - y + y )n - ( - y )n - y n point P has affix z. Then, Eq. (i) can be written as
= 0 - ( - y )n - y n = y n - y n = 0 [because n is odd] p é æz + i öù
ÐBPA = êQ ÐBPA = arg ç ÷ú
\ x + y is a factor of Eq. (i). 2 ë èz - i øû
On putting x = wy in Eq. (i), we get
Thus, locus of P (z ) is such that the angle subtended at
f ( wy , y ) = ( wy + y )n - ( wy )n - y n p
P by the line joining points A and B is . This is the
= y n [( w + 1)n - wn - 1] 2
definition of a circle with diameter AB.
= y n [( - w2 )n - wn - 1)] [Q1 + w + w2 = 0]
Y
= - y n { w2 n + wn + 1} [because n is odd]
A(i)
Since, n is odd but not a multiple of 3, then n = 3k + 1 or
P(z)
n = 3k + 2 , where k is an integer. π/2
\ w2 n + wn + 1 = 0 [in both cases] …(ii)
X
\ f ( wy , y ) = 0 O
\ x - wy is a factor of Eq. (i).
On putting x = w2y in Eq. (i), we get
B(–i)
f ( w2y , y ) = ( w2y + y )n - ( w2y )n - y n
= y n {( w2 + 1)n - w2 n - 1} Therefore, locus of point z is a circle with diameter AB
n n 2n and centre at origin with radius 1.
= y {( - w) - w - 1}
(iii) We have, 1 < | z - 2 - 3i | < 4 represents a circle with
= - y n { wn + w2 n + 1} [because, n is odd]
centre at ( 2, 3) and radius r Î(1, 4 ).
=0 [from Eq. (ii)]
2
\ x - w y is a factor of Eq. (i).
Combining all the factors, we get
1
( x - 0) (y - 0) ( x + y ) ( x - wy ) ( x - w2y ) C 4

Therefore, xy ( x + y ) ( x 2 + xy + y 2 ) is a factor of (2, 3)

f ( x , y ) = ( x + y )n - x n - y n .
l Ex. 46 Interpret the following equations geometrically on
the Argand plane. Since, | z - 2 - 3i | > 1 represents the region in the
plane outside the circle.
p
(i) | z - 1 | + | z + 1 | = 4 (ii) arg ( z + i ) - arg ( z - i ) = \ | z - 2 - 3i | = 1 …(i)
2
p p and | z - 2 - 3i | < 4 represents the region inside circle.
(iii) 1 < | z - 2 - 3i | < 4 (iv) < arg ( z ) <
4 3 \ | z - 2 - 3i | = 4 …(ii)
ì | z - 1| + 4 ü Hence, 1 < | z - 2 - 3i | < 4 represent the angular space
(v) log cos p/ 3 í ý>1
î 3 | z - 1 | - 2þ between the two circles (i) and (ii).
62 Textbook of Algebra

p p On multiplying Eqs. (i) and (ii), we get


(iv) We have, < arg (z ) <
4 3 ( z1 - z 2 ) ( z 2 - z 3 ) = ( z1 - z 3 ) 2
æy ö
Let z = x + iy Þ arg (z ) = tan -1 ç ÷ 2 2 2
èx ø or z1 + z 2 + z 3 = z1z 2 + z 2 z 3 + z 3z1
Y Hence, the triangle whose vertices are z1, z 2 and z 3 is
equilateral.
Similarly, it can be shown that the triangle whose vertices
√3x

are z1 ¢ , z 2 ¢ and z 3 ¢ is also equilateral.


y=

x
l Ex. 48 Show that the triangle whose vertices are
y= z 1 , z 2 , z 3 and z 1 ¢ , z 2 ¢ , z 3 ¢ are directly similar, if
z1 z1¢ 1
π/3 π/4
O X
z 2 z 2 ¢ 1 = 0.
z3 z3 ¢ 1
The given inequality can be written as
p Sol. Let A , B, C be the points of affix z1, z 2 , z 3 and A ¢ , B ¢ , C ¢ be
æy ö p
< tan -1 ç ÷ < the points of affix z1 ¢ , z 2 ¢ , z 3 ¢.
4 èx ø 3
Since, the triangles ABC and A ¢ B ¢ C ¢ are similar, if
p y p ¾® ¾¾®
Þ tan < < tan BC = l B ¢ C ¢
4 x 3
i.e., (z 3 - z 2 ) = l (z 3 ¢ - z 2 ¢ ) …(i)
y
Þ 1< < 3 ¾® ¾¾®
x and CA = l C ¢ A ¢
Þ x <y < 3 x i.e., ( z1 - z 3 ) = l ( z1 ¢ - z 3 ¢ ) …(ii)
This inequality represents the region between the lines A ′ (z1′ )
y = x and y = 3x
A(z1)
ì |z - 1 | + 4 ü
(v) We have, log cos p/ 3 í ý>1
î 3 | z - 1 | - 2þ
ì |z - 1 | + 4 ü α α
or log1 / 2 í ý>1
î 3 | z - 1 | - 2þ B(z2) C(z3) ′
B (z2′) C ′ (z3′ )
|z - 1 | + 4 1 On dividing Eq. (i) by Eq. (ii), we get
Þ <
3 |z - 1 | - 2 2 z3 - z2 z3 ¢ - z2 ¢
=
or 2 |z - 1 | + 8 < 3 |z - 1 | - 2 z1 - z 3 z1 ¢ - z 3 ¢
Þ | z - 1 | > 10 Þ z 3 ( z1 ¢ - z 3 ¢ ) - z 2 ( z1 ¢ - z 3 ¢ )
Hence, the inequality represents exterior of a circle of = z1 ( z 3 ¢ - z 2 ¢ ) - z 3 ( z 3 ¢ - z 2 ¢ )
radius 10 with centre at (1, 0). Þ z1(z 2 ¢ - z 3 ¢ ) - z 2 (z1 ¢ - z 3 ¢ ) + z 3 (z1 ¢ - z 2 ¢ ) = 0
z1 z1 ¢ 1
l Ex. 47 Show that the triangles whose vertices are Hence, z2 z2 ¢ 1 = 0
z 1 , z 2 , z 3 and z 1¢, z 2 ¢ , z 3 ¢ are equilateral, if
z3 z3 ¢ 1
(z 1 - z 2 ) (z 1 ¢ - z 2 ¢ ) = (z 2 - z 3 ) (z 2 ¢ - z 3 ¢ )
Aliter
= (z 3 - z 1 ) (z 3 ¢ - z 1 ¢ )
Since, DABC and DA ¢ B ¢ C ¢ are similar.
Sol. From the first two relations, we have
AB BC
z1 - z 2 z - z3 If = and ÐABC = ÐA ¢ B ¢C ¢ = a [say]
= 2 A ¢B ¢ B ¢C ¢
z 2 ¢ - z 3 ¢ z1 ¢ - z 2 ¢
Then, from Coni method in D ABC and D A ¢ B ¢C ¢, we have
( z1 - z 2 ) + ( z 2 - z 3 ) z - z3
= = 1 z1 - z 2 AB ia
( z 2 ¢ - z 3 ¢ ) + ( z1 ¢ - z 2 ¢ ) z1 ¢ - z 3 ¢ = e …(i)
z 3 - z 2 BC
z1 - z 2 z - z3
\ = 1 …(i) z 1 ¢ - z 2 ¢ A ¢ B ¢ ia
z 2 ¢ - z 3 ¢ z1¢ - z 3¢ and = e …(ii)
z 3 ¢ - z 2 ¢ B ¢C ¢
Also, from the last two relations
( z 2 - z 3 ) ( z 2 ¢ - z 3 ¢ ) = ( z 3 - z1 ) ( z 3 ¢ - z1 ¢ ) …(ii)
Chap 01 Complex Numbers 63

AB BC AB A ¢ B ¢ Þ (a1z1 + a 3z 3 ) = - (a 2z 2 + a 4z 4 ) …(ii)
Since, = \ =
A ¢ B ¢ B ¢C ¢ BC B ¢ C ¢ On dividing Eq. (ii) by Eq. (i), we get
z - z 2 z1 ¢ - z 2 ¢ a1z1 + a 3z 3 a 2 z 2 + a 4 z 4
From Eqs. (i) and (ii), we get 1 = ' = …(iii)
z3 - z2 z3 ¢ - z2 ¢ a1 + a 3 a2 + a4
z1 z1 ¢ 1 Eq. (iii) implies that point O divides PR in the ratio a 3 : a1
On simplifying as in Ist method, we get z 2 z 2 ¢ 1 = 0 and O divides QS in the ratio a 4 : a 2 .
z3 z3 ¢ 1 Let OR = a1k , OP = a 3k , OQ = a 4l , OS = a 2 l
Now, in DOPQ,
l Ex. 49 If w is the nth root of unity and z 1 , z 2 are any two ( PQ )2 = (OP )2 + (OQ )2 - 2 (OP ) (OQ ) cos q
complex numbers, then prove that
n -1 Þ | z1 - z 2 | 2 = a 32k 2 + a 42l 2 - 2a 3a 4lk cos q
2 2 2
å z 1 + wk z 2 = n { z1 + z2 }, where n Î N . \ a1a 2 | z1 - z 2 | 2 = a1a 2 a 23k 2 + a1a 2 a 42l 2
k =0
-2a1a 2 a 3a 4lk cos q
Sol. If 1, w, w2 , w3 , ..., wn - 1 are the n, nth roots of unity, then
2
n -1 n -1 Similarly, a 3a 4 | z 3 - z 4 | = a 3a 4a12k 2
+ a 3a 4a 22l 2
S k
w = 0 and S ( w) = 0k
…(i) - 2a1a 2 a 3a 4lk cos q
k =0 k =0
n -1
From given condition, a1a 2 | z1 - z 2 | 2 = a 3a 4 | z 3 - z 4 | 2
S
2
LHS = z1 + wk z 2 \ a1a 2 a 32k 2 + a1a 2 a 42l 2 = a 3a 4a12k 2 + a 3a 4a 22 l 2
k =0
n -1 Þ k 2a 3a1 (a 2 a 3 - a1a 4 ) = l 2a 2 a 4 (a 2 a 3 - a1a 4 )
= S
k =0
k
( z1 + w z 2 ) ( z1 + ( w ) z 2 ) k
Þ (a1k ) (a 3k ) = (a 2 l ) (a 4 l )
n -1
Þ OP × OR = OQ × OS
= å {z1z1 + z1z 2 ( w)k + z1z 2 wk + z 2 z 2 ( wk ) ( w)k }
k =0 So, P , Q , R and S are concyclic.
n -1 n -1 n -1 n -1

= S |z1|2 + k S= 0 z1z 2 ( w)k + S z1z 2 wk + S | z 2 |2


l Ex. 51 If a and b are the roots of
k =0 k =0 k =0 1
n -1 n -1 n -1 n -1
z + = 2 (cos q + i sin q ), where 0 < q < p and i = -1 ,
z
= | z1| 2
S
k =0
1 + z1z 2 S
k =0
k
( w) + z1z 2 S
k =0
k
( w) + | z 2 | 2
S
k =0
1 show that | a - i | = | b - i |.
1
Sol. Since, z + = 2 (cos q + i sin q )
= n | z1| 2 + 0 + 0 + n | z | 2 [from Eq. (i)] z
= n {| z1 | 2 + | z 2 | 2 } = RHS 1
\ z+ = 2e iq Þ z 2 - 2e iqz + 1 = 0
z
4 4 2e iq ± ( 4e 2 i q - 4 )
l Ex. 50 Let S a i = 0 and S a j z j = 0, then prove that Þ z=
i =1 j =1 2
z 1 , z 2 , z 3 and z 4 are concyclic, if Þ z = e iq ± (e 2 iq - 1) Þ z = e i q ± e i q × 2i sin q
a 1a 2 | z 1 - z 2 | 2 = a 3 a 4 | z 3 - z 4 | 2 . Þ z - i = e i q - i ± e i q × 2i sin q
4
Sol. Q S ai = 0 = (e i q - e i p/ 2 ) ± e i (q + p/ 2 ) × 2sin q
i =1 P(z1) æ q pö æ q pö
iç + ÷ ç - ÷ q /2+ p /4
è 2 4ø
=e × 2i sin è 2 4ø
± e i( )
× 2sin q

θ ì æq p ö ü
= e i (q / 2 + p/ 4 )í2i sin ç - ÷ ± 2sin q ý
Q(z2)
O î è 2 4 ø þ
θ S(z4)
æq p ö
\ | z - i | = 1 × 4 sin 2 ç - ÷ + 2 sin q
è2 4ø
R(z3) æ p öö
æ
\ a1 + a 2 + a 3 + a 4 = 0 = 2 ç1 - cos çq - ÷ ÷ + 2 sin q
è è 2 øø
Þ (a1 + a 3 ) = - (a 2 + a 4 ) …(i)
4
= 2 - 2sin q + 2sin q = 2
and S aj z j = 0
j =1
Þ | a - i | = | b - i | = 2 [here, a ,b are two values of z - i ]
\ a1z1 + a 2 z 2 + a 3z 3 + a 4 z 4 = 0
#L Complex Numbers Exercise 1 :
Single Option Correct Type Questions
n This section contains 30 multiple choice questions. 8. Let a and b be two fixed non-zero complex numbers and
Each question has four choices (a), (b), (c) and (d) out of z is a variable complex number. If the lines
which ONLY ONE is correct a z + a z + 1 = 0 and b z + b z − 1 = 0 are mutually
1. If cos (1 − i ) = a + ib, where a, b ∈ R and i = − 1, then perpendicular, then
1 1 1 1 (a) ab + ab = 0 (b) ab − ab = 0
(a) a = e −  cos 1, b = e +  sin 1
2 e 2 e (c) ab − ab = 0 (d) ab + ab = 0
1 1 1 1
8π   8π 
9. If α = cos 
(b) a = e +  cos 1, b = e −  sin 1
2 e 2 e  + i sin   , where i = − 1, then
 11   11 
1 1 1  1
(c) a = e +  cos 1, b = e +  sin 1 Re (α + α + α + α + α 5 ) is
2 3 4
2 e 2  e
1 1 1 1
1  1 (a) (b) − (c) 0 (d) None of these
(d) a = e −  cos 1, b = e −  sin 1 2 2
2 e 2  e
10. The set of points in an Argand diagram which satisfy both
2. Number of roots of the equation z 10 − z 5 − 992 = 0, where π
real parts are negative, is | z | ≤ 4 and 0 ≤ arg (z ) ≤ , is
(a) 3 (b) 4 (c) 5 (d) 6 3
(a) a circle and a line (b) a radius of a circle
3. If z and z represent adjacent vertices of a regular (c) a sector of a circle (d) an infinite part line
polygon of n sides with centre at origin and if
Im (z ) 11. If f ( x ) = g ( x 3 ) + xh( x 3 ) is divisible by x 2 + x + 1, then
= 2 − 1, the value of n is equal to (a) g( x ) is divisible by ( x − 1 ) but not h( x )
Re (z )
(b) h( x ) is divisible by ( x − 1 ) but not g( x )
(a) 2 (b) 4 (c) 6 (d) 8
r
(c) both g ( x ) and h ( x ) are divisible by ( x − 1 )
4. If Π e ipθ = 1, where Π denotes the continued product
p =1
(d) None of the above
12. If the points represented by complex numbers
and i = − 1, the most general value of θ is z 1 = a + ib, z 2 = c + id and z 1 − z 2 are collinear, where
2nπ 2nπ i = − 1, then
(a) , n ∈I (b) , n ∈I
r (r − 1 ) r (r + 1 ) (a) ad + bc = 0 (b) ad − bc = 0
4nπ 4nπ (c) ab + cd = 0 (d) ab − cd = 0
(c) , n ∈I (d) , n ∈I
r (r − 1 ) r (r + 1 )
13. Let C denotes the set of complex numbers and R is the
(where, n is an integer)
set of real numbers. If the function f : C → R is defined
5. If (3 + i ) (z + z ) − (2 + i ) (z − z ) + 14 i = 0, where i = − 1, by f (z ) = | z |, then
then z z is equal to (a) f is injective but not surjective
(a) 10 (b) 8 (b) f is surjective but not injective
(c) − 9 (d) – 10 (c) f is neither injective nor surjective
(d) f is both injective and surjective
6. The centre of a square ABCD is at z = 0, A is z 1 . Then,
the centroid of ∆ABC is 14. Let α and β be two distinct complex numbers, such that
z1 | α | = | β |. If real part of α is positive and imaginary part
(a) z1 (cos π ± i sin π ) (b) (cos π ± i sin π )
3 of β is negative, then the complex number
 π π z1  π π (α + β ) / (α − β ) may be
(c) z1  cos ± i sin  (d)  cos ± i sin 
 2 2 3  2 2 (a) zero (b) real and negative
(where, i = − 1) (c) real and positive (d) purely imaginary
15. The complex number z, satisfies the condition
3 −i
7. If z = , where i = − 1, then (i 101 + z 101 )103 equals  z − 25  = 24. The maximum distance from the origin of
2
 z
to
(a) iz (b) z
coordinates to the point z, is
(a) 25 (b) 30
(c) z (d) None of these
(c) 32 (d) None of these
Chap 01 Complex Numbers 65

16. The points A, B and C represent the complex numbers  z 


z1 − 
z 1 , z 2 , (1 − i ) z 1 + iz 2 respectively, on the complex plane |z | π z 
23. If arg  = and − z 1 = 3, then | z 1 | equals to
 z  2 | z | 
(where, i = − 1 ). The ∆ABC, is  
 |z | 
(a) isosceles but not right angled
(b) right angled but not isosceles (a) 3 (b) 2 2 (c) 10 (d) 26
π
24. If | z − 2 − i | = | z |
sin  − arg z 
(c) isosceles and right angled
, where i = − 1, then
(d) None of the above
 4 
17. The system of equations | z + 1 − i | = 2 and | z | = 3 has locus of z, is
(where, i = − 1) (a) a pair of straight lines (b) circle
(c) parabola (d) ellipse
(a) no solution (b) one solution
(c) two solutions (d) None of these 25. If 1, z 1 , z 2 , z 3 , …, z n − 1 are the n, nth roots of unity, then
n −1
18. Dividing f (z ) by z − i, we obtain the remainder 1 − i 1
and dividing it by z + i, we get the remainder 1 + i.
the value of ∑ (3 − z r )
, is
r =1
Then, the remainder upon the division of f (z ) by z 2 + 1, n ⋅3n −1
1 n ⋅ 3n − 1
(a) + (b) −1
is 3n − 1 2 3n − 1
(a) i + z (b) 1 + z
n ⋅ 3n − 1
(c) 1 − z (d) None of these (c) +1 (d) None of these
3n − 1
19. The centre of the circle represented by | z + 1 | = 2 | z − 1 |
26. If z = (3 + 7i ) ( λ + iµ ), when λ , µ ∈ I ~ { 0 } and i = − 1,
on the complex plane, is
5 is purely imaginary then minimum value of | z | 2 is
(a) 0 (b)
3 (a) 0 (b) 58
1 3364
(c) (d) None of these (c) (d) 3364
3 3

20. If x = 9 1 / 3 ⋅ 9 1 / 9 ⋅ 9 1 / 27 … ∞, y = 4 1 / 3 ⋅ 4 − 1 / 9 ⋅ 4 1 / 27 … ∞ and 27. Given, z = f ( x ) + ig ( x ), where i = −1 and


∞ f , g : (0, 1) → (0, 1) are real-valued functions, which of the
z= ∑ (1 + i ) − r , where i = − 1, then arg ( x + yz ) is following hold good?
r =1 1  1  1  1 
(a) z = +i   (b) z = +i  
equal to 1 − ix  1 + ix  1 + ix  1 − ix 
 2
(a) 0 (b) − tan −1   1  1  1  1 
 3  (c) z = +i   (d) z = +i  
1 + ix  1 + ix  1 − ix  1 − ix 
 2   2
(c) − tan −1   (d) π − tan −1   28. If z 3 + (3 + 2i ) z + ( − 1 + ia ) = 0, where i = −1, has one
 3  3 
real root, the value of a lies in the interval (a ∈ R )
21. If centre of a regular hexagon is at origin and one of the (a) ( − 2, − 1 ) (b) ( − 1, 0 )
vertices on Argand diagram is 1 + 2i, where i = − 1, (c) ( 0, 1 ) (d) (1, 2 )
then its perimeter is 29. If m and n are the smallest positive integers satisfying
m n
(a) 2 5 (b) 4 5  π  π
(c) 6 5 (d) 8 5 the relation 2 CiS  =  4 CiS  , where i = − 1,
 6  4
 n  (m + n ) equals to
22. Let | z r − r | ≤ r , ∀ r = 1, 2, 3, …, n, then ∑ z r is less than (a) 60 (b) 72 (c) 96 (d) 120
r = 1 
30. Number of imaginary complex numbers satisfying the
(a) n (b) 2n
n(n + 1 ) equation, z 2 = z ⋅ 21 − | z | is
(c) n(n + 1 ) (d)
2 (a) 0 (b) 1 (c) 2 (d) 3
66 Textbook of Algebra

#L Complex Numbers Exercise 2 :


More than One Option Correct Type Questions
n This section contains 15 multiple choice questions. z1 − z 4
(a) is purely real
Each question has four choices (a), (b), (c) and (d) out of z2 − z3
which MORE THAN ONE may be correct. z − z3
(b) 1 is purely imaginary
z +1 z2 − z4
31. If is a purely imaginary number (where i = − 1),
z +i (c) | z1 − z 3 | ≠ | z 2 − z 4 |
then z lies on a  z − z4  z2 − z4 
(a) straight line (d) amp  1  ≠ amp  
z2 − z4  z3 − z4 
(b) circle
(c) circle with radius =
1 38. If | z − 3 | = min {| z − 1 |, | z − 5 | }, then Re(z ) is equal to
2 (a) 2 (b) 2.5 (c) 3.5 (d) 4
(d) circle passing through the origin
π 2π
39. If arg (z + a ) = and arg (z − a ) = (a ∈ R + ), then
32. If z satisfies | z − 1 | < | z + 3 |, then ω = 2 z + 3 − i (where, 6 3
i = − 1) satisfies (a) | z | = a (b) | z | = 2a
π π
(a) | ω − 5 − i | < | ω + 3 + i | (b) | ω − 5 | < | ω + 3 | (c) arg (z ) = (d) arg (z ) =
π 3 2
(c) Im (iω ) > 1 (d) | arg (ω − 1 )| <
2 40. If z = x + iy, where i = −1, then the equation
33. If the complex number is (1 + ri ) = λ (1 + i ), when  (2z − i )

3
 = m represents a circle, then m can be
i = − 1, for some real λ, the value of r can be  (z + i ) 
π 3π 1
(a) cos (b) cosec (a) (b) 1 (c) 2 (d) ∈(3, 2 3 )
5 2 2
π π
(c) cot (d) tan 41. Equation of tangent drawn to circle | z | = r at the point
12 12
A (z 0 ), is
34. If z ∈ C , which of the following relation(s) represents a z z
circle on an Argand diagram? (a) Re   = 1 (b) Im   = 1
z0 z0
(a) | z − 1 | + | z + 1 | = 3 (b) | z − 3 | = 2
z 
(c) | z − 2 + i | =
7
(d) (z − 3 + i ) ( z − 3 − i ) = 5 (c) Im  0  = 1 (d) z z 0 + z 0 z = 2 r 2
z
3
(where, i = −1 ) 42. z 1 and z 2 are the roots of the equation z 2 − az + b = 0,
35. If 1, z 1 , z 2 , z 3 , …, z n − 1 be the n, nth roots of unity and ω where | z 1 | = | z 2 | = 1 and a, b are non-zero complex
numbers, then
be a non-real complex cube root of unity, then
n −1 (a) | a | ≤ 1 (b) | a | ≤ 2
∏ (ω − z r ) can be equal to (c) arg (a ) = arg (b 2 ) (d) arg (a 2 ) = arg (b )
r =1
(a) 1 + ω (b) − 1 43. If α is a complex constant, such that αz 2 + z + α = 0 has
(c) 0 (d) 1 a real root, then
(a) α + α = 1
36. If z is a complex number which simultaneously satisfies
(b) α + α = 0
the equations
(c) α + α = −1
3 | z − 12 | = 5 | z − 8i | and | z − 4 | = | z − 8 |, where (d) the absolute value of real root is 1
i = − 1 , then Im (z ) can be
44. If the equation z 3 + (3 + i ) z 2 − 3z − (m + i ) = 0, where
(a) 8 (b) 17
(c) 7 (d) 15 i = −1 and m ∈ R, has atleast one real root, value of m is
(a) 1 (b) 2 (c) 3 (d) 5
37. If P (z 1 ), Q(z 2 ), R(z 3 ) and S(z 4 ) are four complex
numbers representing the vertices of a rhombus taken in 45. If z + (3 + 2i ) z + ( − 1 + ia ) = 0, where i = − 1, has one
3

order on the complex plane, which one of the following real root, the value of a lies in the interval (a ∈ R )
is hold good? (a) ( − 2, 1 ) (b) ( − 1, 0 ) (c) ( 0, 1 ) (d) (− 2, 3)
Chap 01 Complex Numbers 67

Complex Numbers Exercise 3 :


Passage Based Questions
n This section contains 4 passages. Based upon each of Passage III
the passage 3 multiple choice questions have to be (Q. Nos. 52 to 54)
answered. Each of these questions has four choices (a),
(b), (c) and (d) out of which ONLY ONE is correct. For any two complex numbers z1 and z 2 ,
| z | − | z 2 |
| z1 − z 2 | ≥  1
Passage I  | z 2 | − | z1 |
(Q. Nos. 46 to 48)
and equality holds iff origin z 1 and z 2 are collinear
π , if arg ( z ) < 0 and z 1 , z 2 lie on the same side of the origin.
arg ( z ) + arg ( − z ) =  , where
 − π , if arg ( z ) > 0 52. If z −  = 2 and sum of greatest and least values of | z |
1
− π < arg ( z ) ≤ π.  z
is λ, then λ 2 , is
46. If arg (z ) > 0, then arg ( − z ) − arg (z ) is equal to (a) 2 (b) 4 (c) 6 (d) 8
π
(a) − π (b) − 2
2 53. If z +  = 4 and sum of greatest and least values of | z |
π  z
(c) (d) π
2 is λ, then λ 2 , is
(a) 12 (b) 18 (c) 24 (d) 30
47. Let z 1 and z 2 be two non-zero complex numbers, such
54. If z −  = 6 and sum of greatest and least values of | z | is
3
that | z 1 | = | z 2 | and arg (z 1 z 2 ) = π, then z 1 is equal to
(a) z 2 (b) z 2  z
(c) − z 2 (d) − z 2 2λ, then λ 2 , is
(a) 12 (b) 18 (c) 24 (d) 30
 z1 
48. If arg ( 4 z 1 ) − arg (5 z 2 ) = π, then is equal to
z2  Passage IV
(a) 1 (b) 1.25 (Q. Nos. 55 to 57)
(c) 1.50 (d) 2.50
Consider the two complex numbers z and w, such that
Passage II z −1
w= = a + ib, where a, b ∈ R and i = − 1.
(Q. Nos. 49 to 51) z+2
Sum of four consecutive powers of i (iota) is zero.
55. If z = C iS θ, which of the following does hold good?
i.e., i n + i n + 1 + i n + 2 + i n + 3 = 0, ∀ n ∈ I . 9b
(a) sin θ =
25 1 − 4a
49. If Σ i n ! = a + ib, where i = −1, then a − b , is 1 − 5a
n =1 (b) cos θ =
1 + 4a
(a) prime number
(b) even number (c) (1 + 5a ) 2 + (3b ) 2 = (1 − 4a ) 2
(c) composite number (d) All of these
(d) perfect number b
95 50 56. Which of the following is the value of − , whenever it
Σ Σ
a
50. If ir + i r ! = a + ib, where i = −1, the unit place exists?
r=−2 r=0
 θ 1  θ
digit of a 2011
+b2012
, is (a) 3 tan   (b) tan  
 2 3  2
(a) 2 (b) 3
1 θ
(c) 5 (d) 6 (c) − cot θ (d) 3 cot
3 2
100 101
51. If Σ i r ! + rΠ= 1 i r
r=4
= a + ib, where i = −1, then a + 75b , is 57. Which of the following equals to | z | ?
(a) | w | (b) (a + 1 ) 2 + b 2
(a) 11 (b) 22
(c) 33 (d) 44 (c) a 2 + (b + 2 ) 2 (d) (a + 1 ) 2 + (b + 1 ) 2
68 Textbook of Algebra

#L Complex Numbers Exercise 4 :


Single Integer Answer Type Questions
n This section contains 10 questions. The answer to π 1 − π i π −i 
each question is a single digit integer, ranging from 0 63. If z = (1 + i ) 4  +  , where i = −1,
to 9 (both inclusive).
4  π +i 1 + π i
 |z | 
58. The number of values of z (real or complex) then   equals to
 amp (z )
simultaneously satisfying the system of equations
1 + z + z 2 + z 3 + … + z 17 = 0
64. Suppose A is a complex number and n ∈ N , such that
A n = ( A + 1)n = 1, then the least value of n is
and 1 + z + z 2 + z 3 + … + z 13 = 0 is
65. Let z r ; r = 1, 2, 3, …, 50 be the roots of the equation
59. Number of complex numbers z satisfying z 3 = z is 50 50
1
60. Let z = 9 + ai, where i = − 1 and a be non-zero real. ∑ ( z )r = 0. If ∑ (z = − 5λ, then λ equals to
r=0 r =1 r − 1)
p
If Im (z ) = Im (z ), sum of the digits of a is
2 3 2 32  10  2qπ 2qπ  
66. If P = Σ (3p + 2)  Σ sin − i cos  , where
61. Number of complex numbers z, such that | z | = 1 p =1 q = 1  11 11  

z z i = − 1 and if (1 + i ) P = n (n !), n ∈ N , then the value of n is
and +  = 1 is
z z 67. The least positive integer n for which
−1  1 + x 
n
62. If x = a + ib, where a, b ∈ R and i = − 1 and x = 3 + 4i, 2 1 + i  2 2
  = sin   , where x > 0 and i = −1 is
x 3 = 2 + 11i, the value of (a + b ) is 1 − i  π  2x 

#L Complex Numbers Exercise 5 :


Matching Type Questions
n
This section contains 4 questions. Questions 68 and 69 have three statements (A, B and C) given in Column I and
four statements (p, q, r and s) in Column II and questions 70 and 71 have four statements (A, B, C and D) given in
Column I and five statements (p, q, r, s and t) in Column II. Any given statement in Column I can have correct
matching with one or more statement(s) given in Column II.

68. Column I Column II

(A) 
If z −
1
= 2 and if greatest and least values of | z | are G and L respectively, then G − L , is (p) natural number
 z

Ifz +  = 4 and if greatest and least values of | z | are G and L respectively, then G − L , is
2
(B) (q) prime number
 z

If z −  = 6 and if greatest and least values of | z | are G and L respectively, then G − L , is
3
(C) (r) composite number
 z
(s) perfect number

69.
Column I Column II

(A) If (6 + 8 i ) + (− 6 + 8 i ) = z1 , z2 , z3 , z4 (where i = − 1), then | z1 | + | z2 | | + | z3 | + | z4 | is divisible by


2 2 2 2
(p) 7

(B) If (5 − 12i ) + (− 5 − 12i ) = z1 , z2 , z3 , z4 (where i = − 1), then | z1 | + | z2 | + | z3 | + | z4 | is divisible by


2 2 2 2
(q) 8

(C) If (8 + 15 i ) + (− 8 − 15 i ) = z1 , z2 , z3 , z4(where i = − 1), then | z1 |2 + | z2 |2 + | z3 |2 + | z4 |2 is divisible by (r) 13


(s) 17
Chap 01 Complex Numbers 69

70. 71. Column I Column II


Column I Column
II (A)
If z −  = 5 and maximum and
6 (p) λµ + µ λ = 8
 z
(A) If λ and µ are the unit’s place digits of (p) 2
minimum values of | z | are λ and µ
(143)861 and (5273)1358 respectively,
respectively, then
then λ + µ is divisible by
(B)
If z −  = 6 and maximum and
7 (q) λµ − µ λ = 7
(B) If λ and µ are the unit’s place digits (q) 3
 z
of (212)7820 and (1322)1594
minimum values of | z | are λ and µ
respectively, then λ + µ is divisible
respectively, then
by
(C)
If z −  = 7 and maximum and
8 (r) λµ + µ λ = 7
(C) If λ and µ are the unit’s place digits of (r) 4
 z
(136)786 and (7138)13491 respectively,
minimum values of | z | are λ and µ
then λ + µ is divisible by
respectively, then
(s) 5
(s) λµ − µ λ = 6
(t) 6
(t) λµ + µ λ = 9

#L Complex Numbers Exercise 6 :


Statement I and II Type Questions
n
Directions (Q. Nos. 72 to 78) are Assertion-Reason 75. Statement-1 Locus of z satisfying the equation
type questions. Each of these questions contains two | z − 1 | + | z − 8 | = 5 is an ellipse.
statements:
Statement-1 (Assertion) and Statement-2 (Reason) Statement-2 Sum of focal distances of any point on
Each of these questions also has four alternative ellipse is constant for an ellipse.
choices, only one of which is the correct answer. You 76. Let z 1 , z 2 and z 3 be three complex numbers in AP.
have to select the correct choice as given below.
(a) Statement-1 is true, Statement-2 is true; Statement-2 Statement-1 Points representing z 1 , z 2 and z 3 are
is a correct explanation for Statement-1 collinear.
(b) Statement-1 is true, Statement-2 is true; Statement-2 Statement-2 Three numbers a, b and c are in AP, if
is not a correct explanation for Statement-1 b − a = c − b.
(c) Statement1 is true, Statement-2 is false
77. Statement-1 If the principal argument of a complex
(d) Statement-1 is false, Statement-2 is true
number z is θ, the principal argument of z 2 is 2θ.
72. Statement-1 3 + 7i > 2 + 4i, where i = − 1.
Statement-2 arg (z 2 ) = 2 arg (z )
Statement-2 3 > 2 and 7 > 4
78. Consider the curves on the Argand plane as
73. Statement-1 (cos θ + i sin φ) 3 = cos 3 θ + i sin 3 φ, π
i = −1 C 1 : arg (z ) = ,
2 4
 π π 3π
Statement-2  cos + i sin  = i C 2 : arg (z ) =
 4 4 4
74. Statement-1 Let z 1 , z 2 and z 3 be three complex and C 3 : arg (z − 5 − 5i ) = π, where i = −1.
numbers, such that | 3z 1 + 1 | = | 3z 2 + 1 | = | 3z 3 + 1 | and Statement-1 Area of the region bounded by the curves
1 + z 1 + z 2 + z 3 = 0, then z 1 , z 2 , z 3 will represent vertices 25
of an equilateral triangle on the complex plane. C 1 , C 2 and C 3 is .
2
Statement-2 z 1 , z 2 and z 3 represent vertices of an Statement–2 The boundaries of C 1 , C 2 and C 3 constitute
equilateral triangle, if a right isosceles triangle.
z 12 + z 22 + z 32 + z 1z 2 + z 2 z 3 + z 3 z 1 = 0.
70 Textbook of Algebra

Complex Numbers Exercise 7 :


Subjective Type Questions
n In this section, there are 24 subjective questions. 88. Show that if a and b are real, then the principal value of
79. If z 1 , z 2 and z 3 are three complex numbers, then prove arg (a ) is 0 or π, according as a is positive or negative and
π π
that z 1 Im ( z 2 z 3 ) + z 2 Im ( z 3 z 1 ) + z 3 Im (z 1 z 2 ) = 0. that of b is or − , according as b is positive or negative.
2 2
80. The roots z 1 , z 2 and z 3 of the equation
89. Two different non-parallel lines meet the circle | z | = r .
x 3 + 3ax 2 + 3bx + c = 0 in which a, b and c are complex
One of them at points a and b and the other which is
numbers, correspond to the points A, B, C on the tangent to the circle at c. Show that the point of
Gaussian plane. Find the centroid of the ∆ ABC and 2c −1 − a −1 − b −1
show that it will be equilateral, if a 2 = b. intersection of two lines is .
c − 2 − a −1b −1
81. If 1, α 1 , α 2 , α 3 and α 4 are the roots of x 5
− 1 = 0, then
90. A, B and C are the points representing the complex
prove that
numbers z 1 , z 2 and z 3 respectively, on the complex
ω − α1 ω − α 2 ω − α 3 ω − α 4 plane and the circumcentre of ∆ABC lies at the origin. If
⋅ ⋅ ⋅ = ω, where ω is
ω 2 − α1 ω 2 − α 2 ω 2 − α 3 ω 2 − α 4 the altitude of the triangle through the vertex A meets
a non-real complex root of unity. the circumcircle again at P, prove that P represents the
 z z 
82. If z 1 and z 2 both satisfy the relation z + z = 2 z − 1 and complex number  − 2 3  .
 z1 
π
arg (z 1 − z 2 ) = , find the imaginary part of (z 1 + z 2 ). 91. If | z | ≤ 1 and | ω | ≤ 1, show that
4
83. If ax + cy + bz = X , cx + by + az = Y , bx + ay + cz = Z, | z − ω | 2 ≤ (| z | − | ω | ) 2 + {arg (z ) − arg (ω )} 2 .
show that 92. Let z, z 0 be two complex numbers. It is given that | z | = 1
(i) (a 2 + b 2 + c 2 − bc − ca − ab ) ( x 2 + y 2 and the numbers z, z 0 , z z 0 , 1 and 0 are represented in an
Argand diagram by the points P, P0 , Q, A and the origin,
+ z − yz − zx − xy ) = X
2 2
+Y 2
+ Z − YZ − ZX − XY
2
respectively. Show that ∆POP0 and ∆AOQ are
(ii) (a + b + c − 3abc ) ( x + y + z 3 − 3xyz )
3 3 3 3 3
congruent. Hence, or otherwise, prove that
= X 3 + Y 3 + Z 3 − 3XYZ . | z − z 0 | = | z z 0 − 1 |.
93. Suppose the points z 1 , z 2 , …, z n (z i ≠ 0) all lie on one
84. For every real number c ≥ 0, find all complex numbers z
side of a line drawn through the origin of the complex
which satisfy the equation | z | 2 − 2iz + 2c (1 + i ) = 0,
planes. Prove that the same is true of the points
where i = −1 . 1 1 1
, , …, . Moreover, show that
85. Find the equations of two lines making an angle of 45° z1 z 2 zn
with the line (2 − i ) z + (2 + i ) z + 3 = 0, where i = −1 1 1 1
z 1 + z 2 + … + z n ≠ 0 and + +…+ ≠ 0.
and passing through ( − 1, 4 ). z1 z 2 zn

86. For n ≥ 2, show that 94. If a, b and c are complex numbers and z satisfies
2  22 
 1 + i   1 + i    1 + i   az 2 + bz + c = 0, prove that | a | | b | = a (b ) 2 c and
+ +
  2     2     2  
1 1 1 +
     | a | = | c | ⇔ | z | = 1.

 2 
n 95. Let z 1 , z 2 and z 3 be three non-zero complex numbers
1 + i    1 
… 1 +   = (1 + i ) 1 − n  , where i = −1. | z1 | | z 2 | | z 3 |
  2    22  and z 1 ≠ z 2 . If | z 2 | | z 3 | | z 1 | = 0, prove that
 
| z 3 | | z1 | | z 2 |
87. Find the point of intersection of the curves
arg (z − 3i ) = 3π / 4 and arg (2z + 1 − 2i ) = π , where
(i) z 1 , z 2 , z 3 lie on a circle with the centre at origin.
4 2
z  z − z1 
i = − 1. (ii) arg  3  = arg  3  .
z 2  z 2 − z1 
Chap 01 Complex Numbers 71

96. Prove that, if z 1 and z 2 are two complex numbers and π 3π 5π 1


(iii) sin sin sin =
 1 14 14 14 8
c > 0, then | z 1 + z 2 | 2 ≤ (1 + c ) | z 1 | 2 + 1 +  | z 2 | 2 . π 3π 5π 1
 c (iv) tan tan tan =
14 14 14 7
97. Find the circumcentre of the triangle whose vertices are
given by the complex numbers z 1 , z 2 and z 3 . Also, show that
98. Find the orthocentre of the triangle whose vertices are  π
(1 + y ) 7 + (1 − y ) 7 = 14 y 2 + tan 2 
given by the complex numbers z 1 , z 2 and z 3 .  14 
 2 2 3π   2 2 5π 
99. Prove that the roots of the equation y + tan  y + tan 
π 3π 5π  14   14 
8x 3 − 4 x 2 − 4 x + 1 = 0 are cos , cos and cos .
7 7 7 and then deduce that
Hence, obtain the equations whose roots are  π  3π   5π 
tan 2   + tan 2   + tan 2   = 5
π 3π 5π  14   14   14 
(i) sec 2 , sec 2 , sec 2
7 7 7
101. If the complex number z is to satisfy
2 π 2 3π 2 5π
(ii) tan , tan , tan | z | = 3, | z − {a (1 + i ) − i }| ≤ 3 and| z + 2a − (a + 1) i | > 3, where
7 7 7 i = − 1 simultaneously for atleast one z, then find all
π 3π 5π
(iii) Evaluate sec + sec + sec a ∈ R.
7 7 7
100. Solve the equation z 7 + 1 = 0 and deduce that 102. Write equations whose roots are equal to numbers
π 2π 3π nπ
π 3π 5π 1 (i) sin 2 , sin 2 , sin 2 , ..., sin 2 .
(i) cos cos cos =− 2n + 1 2n + 1 2n + 1 2n + 1
7 7 7 8
π 3π 5π π 2π 3π nπ
(ii) cos cos cos =
7 (ii) cot 2 , cot 2 , cot 2 , ..., cot 2 .
14 14 14 8 2n + 1 2n + 1 2n + 1 2n + 1

#L Complex Numbers Exercise 8 :


Questions Asked in Previous 13 Years’ Exams
n
This section contains questions asked in IIT-JEE, π
(a) | z − 1 | > 2; | arg (z − 1 )| <
AIEEE, JEE Main & JEE Advanced from year 2005 4
to year 2017. π
(b) | z − 1 | > 2; | arg (z − 1 )| <
2
103. If ω is a cube root of unity but not equal to 1, then π
(c) | z + 1 | > 2; | arg (z + 1 )| <
minimum value of | a + bω + cω 2 |, (where a, b and c are 4
integers but not all equal), is π
(d) | z + 1 | > 2; | arg (z + 1 )| <
[IIT-JEE 2005, 3M] 3
3
(a) 0 (b) (c) 1 (d) 2 105. If one of the vertices of the square circumscribing the
2 circle | z − 1 | = 2 is 2 + 3i, where i = − 1. Find the
104. PQ and PR are two infinite rays. QAR is an arc. Point other vertices of the square. [IIT-JEE 2005, 4M]
lying in the shaded region excluding the boundary
106. If z 1 and z 2 are two non-zero complex numbers, such
satisfies [IIT-JEE 2005, 3M]
that | z 1 + z 2 | = | z 1 | + | z 2 |, then arg (z 1 ) − arg (z 2 ) is
Y
(–1+√2, √2i) equal to [AIEEE 2005, 3M]
Q (a) − π (b) − π / 2
0 ) (1,0) (c) π / 2 (d) 0
(–1,
P A
X′
O
X 107. If 1, ω, ω 2 are the cube roots of unity, then the roots of
the equation ( x − 1) 3 + 8 = 0 are [AIEEE 2005, 3M]
R (a) − 1, 1 + 2 ω, 1 + 2 ω 2 (b) − 1, 1 − 2 ω, 1 − 2 ω 2
(–1+√2, –√2 i )
Y′ (c) − 1, − 1, − 1 (d) None of these
72 Textbook of Algebra

108. If ω =
z
and | ω | = 1, where i = − 1, then z lies on 115. The number of elements in the set A ∩ B ∩ C , is
1
z− i (a) 0 (b) 1
3 [AIEEE 2005, 3M] (c) 2 (d) ∞
(a) a straight line (b) a parabola 116. Let z be any point in A ∩ B ∩ C . Then,
(c) an ellipse (d) a circle
| z + 1 − i | 2 + | z − 5 − i | 2 lies between
109. If ω = α + iβ, where β ≠ 0, i = − 1 and z ≠ 1, satisfies the
(a) 25 and 29 (b) 30 and 34
 ω − ωz 
condition that   is purely real, the set of values (c) 35 and 39 (d) 40 and 44
 1 −z 
117. Let z be any point in A ∩ B ∩ C and ω be any point
of z is [IIT-JEE 2006, 3M]
satisfying | ω − 2 − i | < 3. Then, | z | − | ω | + 3 lies between
(a) {z : | z | = 1 } (b) {z : z = z }
(a) − 6 and 3 (b) − 3 and 6
(c) {z : z ≠ 1 } (d) {z : | z | = 1, z ≠ 1 }
(c) − 6 and 6 (d) − 3 and 9
2kπ 2kπ 
110. The value of Σ sin
10
+ i cos  (where i = − 1) 118. A particle P starts from the point z 0 = 1 + 2i, i = −1. It
k =1  11 11 
moves first horizontally away from origin by 5 units and
is [AIEEE 2006, 3M]
then vertically away from origin by 3 units to reach a
(a) i (b) 1
point z 1 . From z 1 , the particle moves 2 units in the
(c) − 1 (d) − i
direction of the vector $i + $j and then it moves through
111. If z 2 + z + 1 = 0, where z is a complex number, the value of
π
2 2 2 2 an angle in anti-clockwise direction on a circle with
 1  2 1  3 1  6 1
z +  + z + 2  + z + 3  + … + z + 6  2
 z  z   z   z  centre at origin, to reach a point z 2 , then the point z 2 is
is [AIEEE 2006, 6M] given by [IIT-JEE 2008, 3M]
(a) 18 (b) 54 (a) 6 + 7i (b) − 7 + 6i
(c) 6 (d) 12 (c) 7 + 6i (d) − 6 + 7i
112. A man walks a distance of 3 units from the origin 1
119. If the conjugate of a complex numbers is , where
towards the North-East (N 45° E) direction. From there, i −1
he walks a distance of 4 units towards the North-West i = − 1. Then, the complex number is [AIEEE 2008, 3M]
(N 45° W) direction to reach a point P. Then, the position
−1 1
of P in the Argand plane, is [IIT-JEE 2007, 3M] (a) (b)
i −1 i +1
(a) 3 e iπ / 4
+ 4i (b) (3 − 4i ) e iπ / 4
−1 1
(c) ( 4 + 3i ) e iπ / 4 (d) (3 + 4i ) e iπ / 4 (c) (d)
i +1 i −1
(where i = −1)
z 120. Let z = x + iy be a complex number, where x and y are
113. If | z | = 1 and z ≠ ± 1, then all the values of lie on integers and i = − 1. Then, the area of the rectangle
1−z2
[IIT-JEE 2007, 3M] whose vertices are the roots of the equation
(a) a line not passing through the origin z z 3 + z z 3 = 350, is [IIT-JEE 2009, 3M]
(b) | z | = 2 (a) 48 (b) 32
(c) the X -axis (c) 40 (d) 80
(d) the Y -axis 121. Let z = cos θ + i sin θ, where i = − 1. Then the value of
114. If | z + 4 | ≤ 3, the maximum value of | z + 1 | is 15

[AIEEE 2007, 3M]


Σ Im (z 2 m − 1 ) at θ = 2° is
m =1 [IIT-JEE 2009, 3M]
(a) 4 (b) 10 1 1
(c) 6 (d) 0 (a) (b)
sin 2 ° 3 sin 2 °
Passage (Q. Nos. 115 to 117) (c)
1
(d)
1
Let A , B and C be three sets of complex numbers as defined 2 sin 2 ° 4 sin 2 °
A = {z : Im ( z ) ≥ 1}
122. If  z −  = 2, the maximum value of | z | is equal to
below: 4
B = {z : | z − 2 − i | = 3}  z [AIEEE 2009, 4M]
C = {z : Re ((1 − i ) z ) = 2}, where i = − 1 (a) 2 + 2 (b) 3 + 1
[IIT-JEE 2008, 4+4+4M] (c) 5 + 1 (d) 2
Chap 01 Complex Numbers 73

123. Let z 1 and z 2 be two distinct complex numbers and 128. If z is any complex number satisfying | z − 3 − 2i | ≤ 2 ,
z = (1 − t ) z 1 + iz 2 , for some real number t with 0 < t < 1 where i = − 1 , then the minimum value of | 2z − 6 + 5i |,
and i = − 1 . If arg (w ) denotes the principal argument is [IIT-JEE 2011, 4M]
of a non-zero complex number w, then [IIT-JEE 2010, 3M] 129. The set
(a) | z − z1| + | z − z 2| = | z1 − z 2 |   2iz  
(b) arg (z − z1 ) = arg (z − z 2 )  Re   : z is a complex number | z | = 1, z ≠ ± 1 is
z − z1 z − z1   1 − z 
2

(c) =0
z 2 − z1 z 2 − z1 [IIT-JEE 2011, 2M]
(d) arg (z − z1 ) = arg (z 2 − z1 ) (a) ( − ∞, − 1 ] ∩ [1, ∞ ) (b) ( − ∞, 0 ) ∪ ( 0, ∞ )
2π 2π (c) ( − ∞, − 1 ) ∪ (1, ∞ ) (d) [2, ∞ )
124. Let ω be the complex number cos + i sin , where
3 3  1 
130. The maximum value of arg   for | z | = 1, z ≠ 1, is
i = − 1, then the number of distinct complex numbers z 1 − z 
z +1 ω ω2 given by [IIT-JEE 2011, 2M]
π π π 2π
satisfying ω z + ω2 1 = 0, is equal to (a) (b) (c) (d)
6 3 2 3
ω2 1 z +ω
[IIT-JEE 2010, 3M] 131. Let w = e iπ / 3 , where i = − 1 and a, b, c , x , y and z be
(a) 0 (b) 1
non-zero complex numbers such that
(c) 2 (d) 3
a +b +c = x
125. Match the statements in Column I with those in a + bw + cw 2 = y
Column II.
a + bw 2 + cw = z.
[Note Here, z takes values in the complex plane and Im
(z ) and Re (z ) denote respectively, the imaginary part | x |2 + | y |2 + | z |2
The value of , is
and the real part of z.] [IIT- JEE 2010, 8M] | a |2 + | b |2 + | c |2 [IIT-JEE 2011, 4M]

Column I Column II 132. Let α and β be real and z be a complex number. If


(A) The set of points z satisfying (p) an ellipse with z 2 + α z + β = 0 has two distinct roots on the line
| z − i | z || = | z + i | z ||, where eccentricity 4/5
Re (z ) = 1, then it is necessary that [AIEEE 2011, 4M]
i = − 1, is contained in or equal to
(a) β ∈ ( − 1, 0 ) (b) | β | = 1
(B) The set of points z satisfying (q) the set of points z (c) β ∈ (1, ∞ ) (d) β ∈( 0, 1 )
| z + 4 | + | z − 4 | = 10 is contained in satisfying
or equal to Im (z) = 0
133. If ω ( ≠ 1) is a cube root of unity and (1 + ω ) 7 = A + Bω,
1 (r) the set of points z then ( A, B ) equals to [AIEEE 2011, 4M]
(C) If | w | = 2, the set of points z = w − (a) (1, 1) (b) (1, 0)
w satisfying
is contained in or equal to |Im (z)| ≤ 1 (c) (−1, 1) (d) (0, 1)
1 (s) the set of points 134. Let z be a complex number such that the imaginary part
(D) If | w | = 1, the set of points z = w +
w satisfying of z is non-zero and a = z 2 + z + 1 is real. Then, a cannot
is contained in or equal to |Re (z) | ≤ 2
take the value [IIT-JEE 2012, 3M]
(t) the set of points z 1 1 3
satisfying | z | ≤ 3 (a) −1 (b) (c) (d)
3 2 4
126. If α and β are the roots of the equation x 2 − x + 1 = 0, z2
135. If z ≠ 1 and is real, the point represented by the
α 2009 + β 2009 is equal to [AIEEE 2010, 4M] z −1
(a) – 1 (b) 1 complex number z lies [AIEEE 2012, 4M]
(c) 2 (d) – 2 (a) on a circle with centre at the origin
127. The number of complex numbers z, such that (b) either on the real axis or on a circle not passing through
the origin
| z − 1 | = | z + 1 | = | z − i |, where i = − 1, equals to
(c) on the imaginary axis
[AIEEE 2010, 4M]
(d) either on the real axis or on a circle passing through the
(a) 1 (b) 2
origin
(c) ∞ (d) 0
74 Textbook of Algebra

136. If z is a complex number of unit modulus and argument 5


(a) is strictly greater than
1 + z 2
θ, then arg   equals to 5
1 + z  [JEE Main 2013, 4M]
(b) is equal to
2
π 3 5
(a) −θ (b) θ (c) is strictly greater than but less than
2 2 2
(c) π − θ (d) −θ (d) lies in the interval (1, 2)

137. Let complex numbers α and


1  2k π   2k π 
lie on circles 142. Let z k = cos   + i sin   ; k = 1, 2,K, 9. Then,
α  10   10 
( x − x 0 ) 2 + (y − y 0 ) 2 = r 2 and match the column.
( x − x 0 ) 2 + (y − y 0 ) 2 = 4 r 2 , respectively. If
Column I Column II
z 0 = x 0 + i y 0 satisfies the equation 2 | z 0 | 2 = r 2 + 2 ,
(A) For each zk there exists a z j such that (1) True
then | α | equals to [JEE Advanced 2013, 2M] zk ⋅ z j = 1
1 1 1 1
(a) (b) (c) (d)
2 2 7 3 (B) There exists a k ∈{1, 2, K , 9} such that (2) False
z1 ⋅ z = zk has no solution z in the set of
3 +i complex numbers
138. Let w = and P = {w n : n = 1, 2, 3, K }. Further,
2 (C) |1 − z1 ||1 − z2 | K |1 − z9 | (3) 1
 1   1  equals to
H 1 = z ∈ C : Re(z ) >  and H 2 = z ∈ C : Re(z ) <  −   , 10
 2   2 
 2kπ 
9
where C is the set of all complex numbers. If (D) 1− ∑ cos  10  equals to (4) 2
z 1 ∈ P ∩ H 1 , z 2 ∈ P ∩ H 2 and O represents the origin, k=1

then ∠z 1O z 2 equals to
[JEE Advanced 2013, 3M] [JEE Advanced 2014, 3M]
π π Codes
(a) (b)
2 6 A B C D A B C D
2π 5π (a) 1 2 4 3 (b) 2 1 3 4
(c) (d)
3 6 (c) 1 2 3 4 (d) 2 1 4 3

Passage (Q. Nos. 139 to 140) 143. A complex number z is said to be unimodular if | z | = 1.
Let S = S 1 ∩ S 2 ∩ S 3 , where Suppose z 1 and z 2 are complex numbers such that
z 1 − 2z 2
S 1 = {z ∈ C : | z | < 4}, is unimodular and z 2 is not unimodular. Then
2 − z 1z 2
  z − 1 + 3i 
S 2 =  z ∈ C : Im   > 0 the point z 1 lies on a [JEE Main 2015, 4M]
  1 − 3i   (a) circle of radius z
(b) circle of radius 2
and S 3 = {z ∈ C : Re z > 0}. [JEE Advanced 2013, 3+3M]
(c) straight line parallel to X -axis
139. min | 1 − 3i − z | equals to (d) straight line parallel to Y -axis
z ∈S
2− 3 2+ 3 144. Let ω ≠ 1 be a complex cube root of unity.
(a) (b)
2 2 If (3 − 3 ω + 2 ω 2 ) 4n + 3 + (2 + 3 ω − 3 ω 2 ) 4n + 3
3− 3 3+ 3 +( −3 + 2 ω + 3 ω 2 ) 4n + 3 = 0, then possible value(s) of n is
(c) (d)
2 2 (are) [JEE Advanced 2015, 2M]
140. Area of S equals to (a) 1 (b) 2
10 π 20 π (c) 3 (d) 4
(a) (b)
kπ   kπ 
3
16 π
3
32 π
145. For any integer k, let α k = cos   + i sin   , where
(c) (d) 7 7
12
3 3
141. If z is a complex number such that | z | ≥ 2, then the
∑ | αk +1 − αk |
k =1
i = −1. The value of the expression
  1  3
minimum value ofz +   , is
  2  ∑ | α 4k −1 − α 4k − 2 |
[JEE Main 2014, 4M] k =1
is [JEE Advanced 2015, 4M]
Chap 01 Complex Numbers 75

2 + 3i sin θ 1 1 
146. A value of θ for which is purely imaginary, is (a) the circle with radius and centre  , 0 for a > 0, b ≠ 0
 2a 
1 − 2i sin θ 2a
[JEE Main 2016, 4M]
 3
1  1 
π (b) the circle with radius − and centre  − , 0 for
(a) (b) sin −1   2a  2a 
6  4
a < 0, b ≠ 0
 1  π
(c) sin −1   (d) (c) the X -axis for a ≠ 0, b = 0
 3 3
(d) the Y -axis for a = 0, b ≠ 0
147. Let a, b ∈ R and a 2 + b 2 ≠ 0. 148. Let ω be a complex number such that 2ω + 1 = z, when
 1  1 1 1
Suppose S = z ∈ C : z = , t ∈ R, t ≠ 0 , where
 a + ibt  z = −3 if 1 − ω − 1 ω 2 = 3k, then k is equal to
2

i = −1. If z = x + iy and z ∈ S, then ( x , y ) lies on 1 ω2 ω7 [JEE Main 2017, 4M]


[JEE Advanced 2016 4M] (a) 1 (b) − z (c) z (d) − 1

Answers
Exercise for Session 1 71. A → (r); B → (p, s); C → (q, t)
72. (d) 73. (d) 74. (c) 75. (d) 76. (a) 77. (d)
1. (d) 2. (c) 3. (b) 4. (b) 5. (c) 6. (b)
78. (d)
7. (d) 8. (a)
82. 2

Exercise for Session 2 84. z = c + i (−1 ± (1 − c2 − 2c)) for 0 ≤ c ≤ 2 − 1 and no solution for
1. (b) 2. (b) 3. (b) 4. (b) 5. (b) 6. (b) c> 2 −1
7. (d) 8. (c) 9. (b) 10. (b) 11. (a) 12. (c) 85. (1 − 3i ) z + (1 + 3i ) z − 22 = 0 and (3 + i ) z + (3 − i ) z + 14 = 0
13. (c) 14. (a)
87. No solution 97.
∑ | z1 |2 ( z2 − z3 )
Exercise for Session 3 ∑ z1 ( z2 − z3 )
1. (a) 2. (b) 3. (d) 4. (a) 5. (b) 6. (a) 98.
∑ z12 ( z 2 − z 3 ) + ∑ | z1 | 2 ( z2 − z3 )
7. (c) 8. (b) 9. (c) 10. (d) 11. (c) 12. (a) ∑( z1 z2 − z2 z1 )
13. (b) 14. (b) 15. (a)
99. (i) x3 − 24x2 + 80x − 64 = 0
Exercise for Session 4 (ii) x3 − 21x2 + 35x − 7 = 0
1. (a) 2. (d) 3. (c) 4. (d) 5. (b) 6. (b) (iii) 4
7. (a) 8. (d) 9. (d) 10. (b) 11. (b) 12. (a) -
– –3 –5
13. (b) 14. (c) 100. Roots of z 7 + 1 = 0 are −1, α , α3, α 5, α , α , α , where
π π
α = cos + i sin
Chapter Exercises 7 7
1. (b) 2. (c) 3. (d) 4. (d) 5. (a) 6. (d)  1 − 71 −1 − 4 11   −1 + 4 11 1 + 71 
7. (b) 8. (d) 9. (b) 10. (c) 11. (c) 12. (b) 101. a ∈  ,  ∪ , 
 2 5   5 2 
13. (c) 14. (d) 15. (a) 16. (c) 17. (a) 18. (c)
2n+ 1
19. (b) 20. (b) 21. (c) 22. (c) 23. (c) 24. (c) 102. (i) C 1 (1 − x)n − 2n+ 1C 3 (1 − x)n− 1 x + ... + (−1)n xn = 0
2n+ 1
25. (d) 26. (d) 27. (b) 28. (b) 29. (b) 30. (c) (ii) C 1xn − 2n+ 1 C 3xn− 1 + 2n+ 1 C 5xn− 2 −... = 0
31. (b,c,d) 32. (b,c,d) 33. (b,c,d) 34. (b,c,d) 35. (a,c,d) 36. (a,b) 103. (c) 104. (c) 105. (1 − 3 ) + i, −i 3, ( 3 + 1) − i 106. (d)
37. (a,b,c) 38. (a,d) 39. (a,c) 40. (a,b,d) 41. (a,d)
107. (b) 108. (a) 109. (d) 110. (d) 111. (d) 112. (d)
42. (b,d) 43. (a,c,d) 44. (a,d) 45. (a,b,d)
113. (d) 114. (c) 115. (b) 116. (c) 117. (d) 118. (d)
46. (a) 47. (d) 48. (b) 49. (a) 50. (c) 51. (b)
119. (c) 120. (a) 121. (d) 122. (c) 123. (a, c, d) 124. (b)
52. (d) 53. (c) 54. (a) 55. (c) 56. (d) 57. (b) 125. A → (q, r); B → (p); C → (p, s); D → (q, r, s, t) 126. (b) 127. (a)
58. (1) 59. (5) 60. (9) 61. (8) 62. (3) 63. (4) 128. (5) 129. (a) 130. (c) 131. (3) 132. (c) 133. (a)
64. (6) 65. (5) 66. (4) 67. (4) 134. (d) 135. (d) 136. (b) 137. (c) 138. (c) 139. (c)
68. A → (p, q); B → (p, r); C → (p, r, s) 140. (b) 141. (d) 142. (c) 143. (a) 144. (a, b, d)
69. A → (q); B → (q, r); C → (q, s) 145. (4) 146. (c) 147. (a,c,d) 148. (b)
70. A → (p, q, r, t); B → (p, s); C → (p, r)
Solutions
r
4. We have, Π e ipθ = 1
p =1

⇒ e i θ ⋅ e 2i θ ⋅ e 3i θ … e ri θ = 1
 r (r + 1) 
iθ  
⇒ e iθ(1 + 2 + 3 + … + r ) = 1 ⇒ e  2 
=1
1. We have, r (r + 1 )  r (r + 1 ) 
or cos  θ  + i sin  θ = 1 + i ⋅ 0
a + ib = cos (1 − i ) = cos 1 cos i + sin 1 sin i  2   2 
= cos1 cosh 1 + sin 1 i sinh 1 On comparing, we get
[Q cos i = cosh 1, sin i ⋅ 1 = i sinh 1 ]
r (r + 1 )  r (r + 1 ) 
 e + e −1   e − e −1  cos  θ  = 1 and sin  θ = 0
= cos 1   + i sin 1    2   2 
 2   2  r (r + 1 ) r (r + 1 )
⇒ θ = 2m π and θ = m1π
1 1 1 1 2 2
= e +  cos 1 + i ⋅ e −  sin 1
2 e 2 e ⇒ θ=
4m π
and θ =
2m1π
1 1 r (r + 1 ) r (r + 1 )
∴ a = e +  cos1
2 e where, m, m1 ∈ I
1 1 4nπ
and b= e −  sin 1 Hence, θ = , n ∈I.
2 e r (r + 1 )

2. Given that, z 10 − z 5 − 992 = 0 5. Let z = x + iy , then


(3 + i ) (z + z ) − (2 + i ) (z − z ) + 14i = 0 reduces to
Let t = z5
(3 + i ) 2 x − (2 + i ) (2iy ) + 14i = 0
⇒ t 2 − t − 992 = 0 ⇒ 6 x + 2y + i (2 x − 4y + 14 ) = 0
1 ± 1 + 3968 1 ± 63 On comparing real and imaginary parts, we get
⇒ t= = = 32, − 31
2 2 6 x + 2y = 0
⇒ 3x + y = 0 …(i)
∴ z 5 = 32
and 2 x − 4y + 14 = 0
and z 5 = − 31 ⇒ x − 2y + 7 = 0 …(ii)
But the real part is negative, therefore z 5 = 32 does not hold. On solving Eqs. (i) and (ii), we get
∴ Number of solutions is 5. x = − 1 and y = 3
3. From Coni method, ∴ z = − 1 + 3i
O ∴ zz = | z | 2 = | − 1 + 3i | 2 = ( −1 ) 2 + (3 ) 2 = 10
6. Since, affix of A is z1.

n C B

π
-
An(z) A1(z) O π/2
z −0 z
= e 2π i /n or = e 2π i /n … (i)
z −0 z D A(z1)
Im(z ) → → → →
But given = 2 −1
Re(z ) ∴ OA = z1 and OB and OC are obtained by rotating OA
z −z z  π → →
− 1 through and π. Therefore, OB = iz1 and OC = -z1.
2i 1 z 2
⇒ = 2 −1 ⇒   = 2 −1 z + iz1 + ( −z1 )
z +z i z + 1 Hence, centroid of ∆ABC = 1
2 z  3
e 2 π i /n
− 1 i z1  π π
⇒  2 π i /n  = i ( 2 − 1) = z1 =  cos + i sin 
[from Eq. (i)] 3 3  2 2
e + 1
 π If A, B and C are taken in clockwise, then centroid of ∆ABC
⇒ i tan   = i ( 2 − 1 ) 1  π π
 n = z1  cos − i sin 
 π  π 3  2 2
⇒ tan   = tan   z1  π π
 n  8 ∴ Centroid of ∆ABC =  cos ± i sin 
∴ n =8 3  2 2
Chap 01 Complex Numbers 77

3 −i  −1 − i 3  z1 z1 1
7. Given that, z = = i  = iω 2
2  2  ⇒ z2 z2 1 =0
∴ z 101 = (iω 2 )101 = i 101 ω 202 = iω z1 − z 2 z1 − z 2 1
Now, i 101
+ z 101 = i + iω = i ( − ω 2 ) z1 z1 1

∴ (i 101 + z 101 )103 = − i 103 ω 206 = − i 3 ω 2 = iω 2 = z


M
a Applying R3 → R3 − R1 + R2, then z 2 z2 1 =0
8. The complex slope of the line az + a z + 1 = 0 is α = −
a M
b
and the complex slope of the line bz + b z − 1 = 0 is β = − 0… 0 …1
b Expand w.r.t. R3, then
Since, both lines are mutually perpendicular, then z1z 2 − z1 z 2 = 0
∴ α+β=0 ⇒ z1z 2 − (z1z 2 ) = 0
a b
⇒ − − =0 ⇒ Im (z1 z 2 ) = 0
a b
⇒ Im ((a + ib ) (c + id )) = 0
⇒ ab + a b = 0
⇒ Im ((a + ib ) (c − id )) = 0
 8π   8π 
9. We have, α = cos   + i sin   ⇒ bc − ad = 0 ⇒ ad − bc = 0
 11   11 
Now, Re (α + α + α + α + α 5 )
2 3 4 13. Let z = a + ib

α + α2 + α3 + α4 + α5 + α + α2 + α3 + α4 + α 5 ∴ f (a + ib ) = (a 2 + b 2 )
=
2 ⇒ f (z ) = f (z ) = f ( −z ) = f ( −z ) = (a 2 + b 2 )
− 1 + (1 + α + α 2 + α 3 + α 4 + α 5 + α + α 2 + α 3 + α 4 + α 5
= ∴ f is not injective (i.e., it is many-one).
2
−1 + 0 but | z | > 0 i. e. f (z ) > 0 ⇒ f (z ) ∈ R + (Range)
= [sum of 11, 11th roots of unity]
2 ⇒ R+ ⊂ R
1
=− ∴ f is not surjective (i.e., into).
2
Hence, f is neither injective nor surjective.
10. |z | ≤ 4 …(i)
14. Let α = re iθ , β = re iφ [Q| α | = | β |, given]
π
and 0 ≤ arg(z ) ≤ …(ii)  π π
3 where, θ ∈  − ,  and φ ∈ ( − π , 0 )
 2 2
P(z) θ+ φ
i
 2 

 θ − φ
e ⋅ 2 cos  
π/3 α + β re iθ + reiφ  2 
∴ = =
Real axis α − β re iθ − reiφ θ+ φ
O i
 2 

 θ − φ
e ⋅ 2i sin  
 2 
 θ − φ
= − i cot   = Purely imaginary
which implies the set of points in an argand plane, is a sector  2 
of a circle. 25 25 25 25
15. We have, | z | = z + − ≤ z+ +
11. Since, x 2 + x + 1 = ( x − ω ) ( x − ω 2 ), where ω is the cube root z z z −z
of unity and f ( x ) = g ( x 3 ) + x h ( x 3 ) is divisible by x 2 + x + 1. ⇒ | z | ≤ 24 +
25
Therefore, ω and ω 2 are the roots of f ( x ) = 0. |z |
⇒ f (ω ) = 0 and f (ω 2 ) = 0 ⇒ | z | − 24 | z | − 25 ≤ 0 ⇒ (| z | − 25 ) (| z | + 1 ) ≤ 0
2

∴ | z | − 25 ≤ 0 [Q | z | + 1 > 0 ]
⇒ g (ω 3 ) + ω h (ω 3 ) = 0
⇒ | z | ≤ 25 or | z − 0 | ≤ 25
and g ((ω 2 ) 3 ) + ω 2h (ω 2 ) 3 = 0 Hence, the maximum distance from the origin of coordinates
⇒ g (1 ) + ω h (1 ) = 0 to the point z is 25.
and g (1 ) + ω 2h (1 ) = 0 16. Q A ≡ z1, B ≡ z 2, C ≡ (1 − i ) z1 + iz 2
⇒ g (1 ) = h (1 ) = 0 ∴ AB = | z1 − z 2 |
Hence, g ( x ) and h ( x ) both are divisible by ( x − 1 ). BC = | z 2 − (1 − i ) z1 − iz 2| = | (1 − i ) (z 2 − z1 )|
12. Since, z1, z 2 and z1 − z 2 are collinear. = 2 | z1 − z 2 |
z1 z1 1 and CA = | (1 − i ) z1 + iz 2 − z1| = | − i (z1 − z 2)|
= | − i || z1 − z 2 | = | z1 − z 2 |
∴ z2 z2 1 =0
It is clear that, AB = CA and ( AB ) 2 + (CA ) 2 = ( BC ) 2
z1 − z 2 z1 − z 2 1
∴ ∆ ABC is isosceles and right angled.
78 Textbook of Algebra

17. Centre and radius of circle | z | = 3 …(i) y = 41 /3 ⋅ 4 − 1 /9 ⋅ 41 /27…∞ = 41 /3 − 1 /9 + 1 /27 … ∞


are C1 ≡ 0, r1 = 3 1 /3

and centre and radius of circle = 41 + 1 /3 = 41 /4 = 2



|z + 1 −i | = 2 …(ii) 1 1 1
and C 2 = − 1 + i , r2 = 2
and z = ∑ (1 + i )− r = (1 + i ) + (1 + i )2 + (1 + i )3 + …∞
r =1

Q | C1C 2 | = | − 1 + i | = 2 1
and | C1C 2 | < r1 − r2 (1 + i ) 1
= = = −i
1 i
1−
(1 + i )
|z|=3
Now, x + yz = 3 − i 2
 2
O′ |z+1–i|=√2 ∴ arg ( x + yz ) = arg (3 − i 2 ) = − tan −1  
 3 
O
21. Q A1 ≡ 1 + 2i
A2 A1(1+2i )

π/3
A3 A6
Hence, circle (ii) completely inside circle (i) O
∴ Number of solutions = 0
18. We have, f (z ) = g(z ) (z 2 + 1) + h(z )
A4 A5
where, degree of h (z ) < degree of (z 2 + 1 ) iπ / 3
∴ A2 = (1 + 2i ) e
⇒ h(z ) = az + b ; a, b ∈ C
1 i 3 1 i 3
∴ f (z ) = g (z ) (z 2 + 1 ) + az + b ; a, b ∈ C = (1 + 2i )  + = + +i − 3
2 2  2 2
⇒ f (z ) = g (z ) (z − i ) (z + i ) + az + b ; a, b ∈ C …(i)
Now, f (i ) = 1 − i 1   3 
[given] =  − 3 + i  + 1
⇒ ai + b = 1 − i [from Eq. (i)] …(ii) 2   2 
and f ( −i ) = 1 + i [given] 1   3 
∴ | A1A2 | = 1 + 2i −  − 3 − i  + 1
⇒ a ( −i ) + b = 1 + i [from Eq. (i)] …(iii) 2   2 
On solving Eqs. (ii) and (iii) for a and b, we get
1  3
a = − 1 and b = 1 = + 3 + i 1 − 
2  2 
∴ Required remainder, h (z ) = az + b = − z + 1 = 1 − z
2
19. We have, | z + 1| = 2 | z − 1| 1 
2
 3
=  + 3 + 1 −  = 5
Put z = x + iy , we get 2   2 
( x + 1 ) 2 + y 2 = 4 [( x − 1 ) 2 + y 2 ] ∴ Perimeter = 6 | A1A2 | = 6 5
⇒ 3 x + 3y − 10 x + 3 = 0
2 2
22. We have,
10 n n n
⇒ x 2 + y2 − x + 1 = 0
∑ zr = ∑ ∑ (| zr − r | + | r | )
…(i) (zr − r ) + r ≤
3
r =1 r =1 r =1
On comparing Eq. (i) with the standard equation
n n n n
x 2 + y 2 + 2 gx + 2 fy + c = 0 = ∑ | zr − r | + ∑ | r | ≤ ∑ r + ∑ | r |
10 5 r =1 r =1 r =1 r =1
⇒ g=− = − and f = 0
6 3 n(n + 1 ) n(n + 1 )
= + = n(n + 1 )
5  2 2
∴ Required centre of circle ≡ ( − g, − f ) ≡  , 0 n
3  ∴ ∑ zr ≤ n (n + 1 )
5 5 r =1
i.e. + 0⋅i =
3 3  z 
 z1 − 
20. Q x = 9 1 /3
⋅9
1 /9
⋅9
1 / 27
…∞ |z |  π z
23. We have, arg  = and − z1 = 3
1 /3  z  2 |z |
 
= 91 /3 + 1 /9 + 1 /27 + … ∞ = 91 − 1/3 = 91 /2 = 3  |z | 
Chap 01 Complex Numbers 79

which implies the following diagram λ 7


⇒ =
Y µ 3
z Q λ, µ ∈ I − { 0}
|z| For minimum value λ = 7, µ = 3
3
1
90° ∴ | z | 2 = | (3 + 7i ) ( λ + iµ ) | 2
z1
X′
O
X = | 3 + 7i | 2 | λ + iµ| 2 = 58 ( λ2 + µ 2 )
= 58 (7 2 + 3 2 ) = (58 ) 2 = 3364
27. We have,
z = f ( x ) + i g( x )
Y′ where, i = −1 and f , g : ( 0,1 ) → ( 0,1 ) are real-valued
z functions.
⇒ − z1 = 3 ⇒ | z1| = 9 + 1 = 10
|z | 1  1 
(a) z = + i 
24. Let z = x + iy = r (cos θ + i sin θ ) 1 − ix  1 + ix 
∴ | z | = r , arg (z ) = θ 1 + ix x+i 1+x (1 + x )
= + = +i
π  1+x 2
1+x 2
1+x 2
1 + x2
Given, | z − 2 − i | = | z | sin  − arg (z )
4  1+x 1+x
⇒ f (x ) = and g( x ) =
π  1 + x2 1 + x2
⇒ | x + iy − 2 − i | = r sin  − θ
4  But for x = 0.5, f ( 0.5 ) > 1 and g( 0.5 ) > 1, which is out of
range.
1
⇒ | ( x − 2 ) + i (y − 1 ) | = r (cos θ − sin θ ) Hence, (a) is not a correct option.
2
1  1 
1 (b) z = + i 
⇒ ( x − 2 ) 2 + (y − 1 ) 2 = x −y 1 + ix  1 − ix 
2
On squaring both sides, we get 1 − ix (i − x )  1 − x   1−x 
= + =  + i 
2 ( x 2 + y 2 − 4 x − 2y + 5 ) = x 2 + y 2 − 2 xy 1+x 2
1+x 2
1 + x 
2
1 + x 2
⇒ ( x + y ) 2 = 2( 4 x + 2y − 5 ) 1−x 1−x
⇒ f (x ) = and g( x ) =
1+x 2
1+ x2
which is a parabola.
25. Since, 1, z1, z 2, z 3, …, zn − 1 are the n, nth roots of unity. Clearly, f ( x ), g( x ) ∈ ( 0, 1 ), if x ∈( 0,1 )
Hence, (b) is the correct option.
∴ (z n − 1 ) = (z − 1 ) (z − z1 ) (z − z 2 ) (z − z 3 ) … (z − zn − 1 ) 1 − ix i (1 − ix ) (1 + x ) i (1 − x )
(c) z = + = +
n −1 1 + x2 1 + x 2 (1 + x 2 ) (1 + x 2 )
= (z − 1 ) ∏ (z − zr )
r =1 Hence, (c) is not a correct option.
Taking log on both sides, we get 1  1  1 + ix i (1 + ix )
n −1
(d) z = + i = +
1 − ix  1 − ix  1 + x 2 (1 + x 2 )
loge (z n − 1 ) = loge (z − 1 ) + ∑ loge (z − zr )
(1 − x ) i (1 + x )
r =1 = +
(1 + x 2 ) (1 + x 2 )
On differentiating both sides w.r.t.z, we get
n −1 Hence, (d) is not a correct option.
nz n − 1 1 1
− =∑ 28. Let z = α be a real roots of equation.
(z n − 1 ) (z − 1 ) r = 1 (z − zr )
z 3 + (3 + 2i )z + ( −1 + ia ) = 0

Putting z = 3, we get ⇒ α 3 + (3 + 2i ) α + ( −1 + ia ) = 0
n −1 ⇒ (α 3 + 3 α − 1 ) + i (a + 2 α ) = 0
1 n ⋅ 3n − 1 1
∑ = n
(3 − zr ) (3 − 1 ) 2
− On comparing the real and imaginary parts, we get
r =1
α 3 + 3 α − 1 = 0 and a + 2 α = 0
26. We have, a
z = (3 + 7i ) ( λ + iµ ) ⇒ α=−
2
= (3 λ − 7µ ) + i (7 λ + 3 µ ) a 3 3a
Since, z is purely imaginary. ⇒ − − −1 = 0
8 2
∴ 3 λ − 7µ = 0
⇒ a 3 + 12a + 8 = 0
80 Textbook of Algebra

Let f (a ) = a 3 + 12a + 8 which is a circle and passing through the origin


⇒ f ( −1 ) < 0 and f ( 0 ) > 0 1+i
2
1+i 1
and radius = −0= =
∴ a ∈ ( −1, 0 ) 2 2 2
π π π
29. CiS = cos + i sin 32. Given, |z − 1 | < |z + 3 |
6 6 6
⇒ |z − 1 |2 < |z + 3 |2
 3 + i  1  −1 + i 3  ω
=  =   = = − iω ⇒ | z | 2 + 1 − 2 Re (z ) < | z | 2 + 9 + 2 Re (3z )
 2  i  2  i
m ⇒ 2 Re ( 4z ) > − 8
 π
∴ 2 CiS  = ( −2iω ) = (( −2iω ) )
m 3 m /3
= (8i ) m /3
⇒ Re ( 4z ) > − 4
 6 4z + 4z
n n ⇒ >−4
 π   π π 2
and  4 CiS  =  4  cos + i sin   = (2 2 (1 + i )n
 4   4 4 ∴ z + z > −2
and ω = 2z + 3 − i
= (8(1 + i ) 2 )n /2 = (16i )n /2
∴ ω + ω = 2z + 3 − i + 2z + 3 + i
Thus, (8i ) m/3
= (16i )n / 2 = 2(z + z ) + 6 > − 4 + 6
which is satisfy only when m = 48 and n = 24 ⇒ ω + ω >2
∴ m + n = 72 Option (a) | ω − 5 − i | < | ω + 3 + i |
30. We have, z = z ⋅ 21 − | z|
2
⇒ | 2z + 3 − i − 5 − i | < | 2z + 3 − i + 3 + i |
Taking modulus on both sides, we get ⇒ | 2z − 2 − 2i | < | 2z + 6 |
| z | 2 = | z | ⋅ 21 − | z| ⇒ |z −1 −i | <|z + 3|
which is false.
⇒ | z | (| z | − 21 − | z | ) = 0 …(i)
1 − | z|
Option (b) | ω − 5 | < | ω + 3 |
and arg (z ) = arg (z ⋅ 2
2
)
⇒ | 2z + 3 − i − 5 | < | 2z + 3 − i + 3 |
⇒ 2 arg (z ) = arg (z ) = − arg (z ) ⇒ | 2z − 2 − i | < | 2z + 6 − i |
⇒ 3 arg (z ) = 0 i i
⇒ z −1 − < z + 3 −
∴ arg (z ) = 0 2 2
Then, y =0 [Q z = x + iy ] ⇒ |z −1| <|z + 3|
From Eq. (i), | z | = 0 ⇒ x = 0 [Q y = 0 ] which is true.
One solution is z = 0 + i ⋅ 0 = 0. Option (c) Im (iω ) > 1
Also, from Eq. (i), iω − iω
⇒ >1
–x
y=2
Y 2i
iω + i ω
/2 ⇒ >1
(0, 1) y = |x| 2i
⇒ ω + ω >2
X′ o X which is true.
π
Option (d) | arg (ω − 1 ) | <
2
π
⇒ | arg (2z + 3 − i − 1 ) | <
Y′ 2
| z | = 21 − | z| ⇒ | x | = 21 − x π
⇒ | arg (2z + 2 − i ) | <
|x| 2
⇒ = 2 − x = y (say)
2  Im(2z + 2 − i )  π
⇒ tan −1   <
Hence, total number of solutions = 2  Re(2z + 2 − i )  2
z +1
31. Q is a purely imaginary number. ∴ Re(2z + 2 − i ) > 0
z +i (2z + 2 − i ) + (2z + 2 + i )
 z + 1  z + 1 z +1  z + 1 ⇒ >0
∴   =−  ⇒ =−  2
z + i z + i z −i z + i ⇒ z +z +2>0
⇒ (z + 1 ) (z + i ) + (z − i ) (z + 1 ) = 0 ⇒ z + z > −2
⇒ 2zz + z (1 + i ) + z (1 − i ) = 0 which is true.
1 + i 1 − i 33. ∴ (1 + ri ) 3 = λ (1 + i )
⇒ zz +  z + z =0
 2   2  ⇒ 1 + (ri ) 3 + 3(1 ) 2ri + 3(1 ) (ri ) 2 = λ (1 + i )
Chap 01 Complex Numbers 81

⇒ 1 − r 3i + 3ri − 3r 2 = λ + i λ ⇒ 2( x 2 + y 2 ) + 27 x − 50y + 38 = 0 …(i)


On comparing real and imaginary parts, we get and | z − 4 | = | z − 8 | ⇒| z − 4 | = | z − 8 |
2 2

1 − 3r = λ
2
⇒ | z | + 16 − 2 Re ( 4z ) = | z | 2 + 64 − 2 Re(8z )
2

and − r 3 + 3r = λ ⇒ 8 Re (z ) = 48
∴ Re (z ) = 6
Then, − r + 3r = 1 − 3r
3 2
⇒ x =6 …(ii)
⇒ r − 3r − 3r + 1 = 0
3 2
From Eqs. (i) and (ii), we get
⇒ (r 3 + 1 ) − 3r (r + 1 ) = 0 2 (36 + y 2 ) + 162 − 50y + 38 = 0
⇒ (r + 1 ) (r 2 − r + 1 − 3r ) = 0 ⇒ y 2 − 25y + 136 = 0
⇒ (r + 1 ) (r 2 − 4r + 1 ) = 0 ⇒ (y − 17 ) (y − 8 ) = 0
∴ r = − 1, 2 ± 3 ⇒ y = 17,8
∴ Im (z ) = 17, 8
3π π π
⇒ r = cosec , tan , cot 37.
2 12 12 Q(z2)
P(z1)
34. Option (a) | z − 1 | + | z + 1 | = 3
Here, | 1 − ( −1 ) | < 3 90°
i. e. 2 < 3, which is an ellipse.
Option (b) | z − 3 | = 2
It is a circle with centre 3 and radius 2.
7
Option (c) | z − 2 + i | = (z3)R
S(z4)
3
7
It is a circle with centre (2 − i ) and radius . Option (a)Q PS || QR
3
z − z4 
Option (d) (z − 3 + i ) (z − 3 − i ) = 5 ∴ arg  1  =0
z2 − z3
⇒ (z − 3 + i ) (z − 3 + i ) = 5
z1 − z 4
⇒ | z − 3 + i |2 = 5 ⇒ is purely real.
z2 − z3
⇒ |z −3 + i | = 5 Option (b) Q Diagonals of rhombus are perpendicular.
It is a circle with centre at (3 − i ) and radius 5.  z − z3  π
Then, arg  1  =
35. Since, 1, z1, z 2, z 3, …, zn − 1 are the n, nth roots of unity. z2 − z4  2
Therefore, z1 − z 3
⇒ is purely imaginary.
z − 1 = (z − 1 ) (z − z1 ) (z − z 2 ) … (z − zn − 1 )
n z2 − z4
Option (c) Q PR ≠ QS
zn − 1
⇒ = (z − z1 ) (z − z 2 ) … (z − zn − 1 ) ∴ | z1 − z 3 | ≠ | z 2 − z 4 |
z −1
n −1
Option (d) Q ∠QSP = ∠RSQ
= ∏ (z − zr )  z − z4  z − z4 
r =1 ∴ amp  2  = amp  3 
 z1 − z4  z2 − z4 
Now, putting z = ω, we get
n −1  z − z4  z − z4 
ωn − 1 ⇒ − amp  1  = − amp  2 
∏ (ω − zr ) = z2 − z4  z3 − z4 
r =1 ω −1
 0, if n = 3r, r ∈ Z  z − z4  z − z4 
⇒ amp  1  = amp  2 
 z2 − z4  z3 − z4 
=  1, if n = 3r + 1 , r ∈ Z
1 + ω, if n = 3r + 2, r ∈ Z 38. Q | z − 3 | = min {| z − 1 |, | z − 5 | }

36. Q 3 | z − 12 | = 5 | z − 8i | Case I If | z − 3 | = | z − 1 |
∴ 9 | z − 12 | = 25 | z − 8i |
2 2 On squaring both sides, we get
| z − 3 |2 = | z − 1 |2
⇒ 9 (z − 12 ) (z − 12 ) = 25(z − 8i ) (z + 8i )
⇒ | z | 2 + 9 − 2 Re (3z ) = | z | 2 + 1 − 2 Re (z )
⇒ 9 (zz − 12(z + z ) + 144 ) = 25(zz + 8i (z − z ) + 64 )
⇒ 16zz + 108 (z + z ) + 200 (z − z ) i + 304 = 0 ⇒ 4 Re (z ) = 8
⇒ 16( x 2 + y 2 ) + 216 x − 400y + 304 = 0 ⇒ Re (z ) = 2
82 Textbook of Algebra

Case II If | z − 3 | = | z − 5 | z z 
+ 
On squaring both sides, we get z0 z0
⇒ =1
| z − 3 |2 = | z − 5 |2 2
⇒ | z | 2 + 9 − 2 Re (3z ) = | z | 2 + 25 − 2 Re (5z ) z
∴ Re   = 1
⇒ 4 Re (z ) = 16 ⇒ Re (z ) = 4 z0
39. C(z) 42. Q z1 + z 2 = a, z1z 2 = b
and given | z1| = | z 2 | = 1
π/6 π/3
a Let z1 = ei θ and z 2 = ei φ
2π/3
π/6 π/3 π/3 Q | a | = | z 1 + z 2 | ≤ | z 1| + | z 2 | = 1 + 1 = 2
a O a
A(– a,0) B(a,0) ∴ |a | ≤2
θ+φ
Also, arg (a ) = arg (z1 + z 2 ) = arg (ei θ + ei φ ) =
2
and arg (b ) = arg (z1z 2 ) = arg (ei θ + φ ) ) = θ + φ
From figure, it is clear that z lies on the point of intersection of ∴ 2 arg (a ) = arg (b ) ⇒ arg (a 2 ) = arg (b )
the rays from A and B.
43. Q αz2 + z + α = 0 …(i)
Q ∠ACB = 90 ° and OBC is an equilateral triangle.
Hence, OC = a Then, αz2 + z + α = 0
⇒ | z − 0 | = a or | z | = a ⇒ α (z ) 2 + z + α = 0
π
and arg (z ) = arg (z − 0 ) = ⇒ αz 2 + z + α = 0 [Q z = z ] …(ii)
3
On subtracting Eq. (ii) from Eq. (i), we get
2z − i
40. Q =m (α − α ) z 2 − (α − α ) = 0
z +i
⇒ α − α = 0 and z 2 = 1
z − i/2 m
⇒ = ∴ α = α and z = ± 1
z +i 2
Put z = ± 1 in Eq. (i), we get
m
For circle, ≠1 α+α =±1
2
and absolute value of real root = 1
⇒ m ≠ 2 and m > 0
i.e., |z | =| ± 1| =1
41. ∴ A(z 0 ) lie on | z | = r
44. Let z = α be a real root of equation
⇒ | z 0| = r ⇒ | z 0| 2 = r 2 ⇒ z 0 z 0 = r 2
z 3 + (3 + i ) z 2 − 3z − (m + i ) = 0
⇒ α 3 + (3 + i ) α 2 − 3 α − (m + i ) = 0
A(z0)
r ⇒ (α 3 + 3 α 2 − 3 α − m ) + i (α 2 − 1 ) = 0

O On comparing real and imaginary parts, we get


α 3 + 3α 2 − 3α − m = 0
P(z)
and α2 − 1 = 0 ⇒ α = ± 1

Let P (z ) be any point on tangent, then For α = 1, we get


π 1 + 3 −3 −m = 0 ⇒ m =1
∴ ∠PAO =
2 For α = − 1, we get
Complex slope of AP + Complex slope of OA = 0 −1 + 3 + 3 −m = 0 ⇒ m =5
z − z0 z0 − 0 45. Let z = α be a real root of equation
⇒ + =0
z − z 0 z0 − 0
z 3 + (3 + 2i ) z + ( −1 + ia ) = 0
⇒ z z 0 + z 0 z = 2z 0 z 0
⇒ α 3 + (3 + 2i ) α + ( − 1 + ia ) = 0
⇒ zz 0 + z 0z = 2r 2
⇒ (α 3 + 3 α − 1 ) + i (a + 2α ) = 0
⇒ zz 0 = zz 0
zz 0 z 0 z On comparing real and imaginary parts, we get
Also, + 2 =2 α 3 + 3α − 1 = 0
r2 r
zz 0 z 0z and a + 2α = 0
⇒ + =2
z 0z 0 z 0z 0
Chap 01 Complex Numbers 83

⇒ α=−
a 98  50 
2 = ∑ i r − 3 + i 0! + i 1! + i 2! + i 3! + ∑ i r ! 
r =1  r=4 
a 3 3a
⇒ − − − 1 = 0 ⇒ a 3 + 12a + 8 = 0
8 2  47 
= (i −2 + i −1 + 0 ) + i 1 + i 1 + i 2 + i 6 + ∑ i (r + 3)! 
Let f (a ) = a 3 + 12a + 8  
 r =1 
∴ f ( −1 ) < 0, f ( 0 ) > 0, f ( −2 ) < 0
= ( − 1 − i ) + (i + i − 1 − 1
f (1 ) > 0 and f (3 ) > 0
⇒ a ∈ ( −2, 1 ) or a ∈ ( −1, 0 ) or a ∈ ( −2,3 ) + (i 0 + i 0 + i 0 + … 47 times))
Sol. (Q. Nos. 46 to 48) = ( − 1 − i ) + (2i − 2 + 47 )
46. Q arg (z ) > 0 = 44 + i = a + ib [given]
∴ arg (z ) + arg ( −z ) = − π ∴ a = 44, b = 1
⇒ − arg (z ) + arg ( −z ) = − π Unit place digit of a 2011 = ( 44 ) 2011
⇒ arg ( −z ) − arg (z ) = − π = ( 44 ) (( 44 ) 2 )1005 = ( 44 ) (1936 )1005
47. Q arg (z1z 2) = π = (Unit place of 44)
⇒ arg (z1 ) + arg (z 2 ) = π × (Unit place digit of (1936 )1005)
⇒ arg (z1 ) − arg (z 2 ) = π = Unit place of ( 4 × 6 ) = 4
Given, | z 1| = | z 2 | and unit place digit of b 2012 = (1 ) 2012 = 1
∴ | z 1| = | z 2 | = | z 2 | Hence, the unit place digit of a 2011 + b 2012 = 4 + 1 = 5.
Then, z1 + z 2 = 0 100 101
⇒ z1 = − z 2 51. Q ∑ i r ! + ∏ i r
r=4 r =1
48. arg ( 4z1 ) − arg (5z 2 ) = π 97
is possible only when | 4z1| = | 5z 2 | = ∑ i (r + 3)! + i 1 ⋅ i 2 ⋅ i 3 … i 101
r =1
z1 5
⇒ = = 1 . 25 = (i 0 + i 0 + i 0 + … 97 times) + i 1 + 2 + 3 + … + 101
z2 4
= 97 + i 5151 = 97 + i 3 = 97 − i
and also 4z1 + 5z 2 = 0
∴ a = 97 and b = − 1
z1 5
⇒ =− Hence, a + 75b = 97 − 75 = 22
z2 4
Sol. (Q. Nos. 52 to 54)
z1 5
∴ = = 1 . 25 If z ±
a
= b, where a, b > 0
z2 4 z
Sol. (Q. Nos. 49 to 51) a a
∴ z± ≤|z | +
49. Q n ! is divisible by 4, ∀ n ≥ 4. z |z |
25 22 a
⇒ b ≤|z | +
∴ ∑ i n ! = ∑ i (n + 3)! |z |
n=4 n =1
⇒ | z |2 − b | z | + a ≥ 0
= i 0 + i 0 + i 0 + … (22 times) = 22 …(i)
b − b 2 − 4a
25 25 ∴ |z | ≤
∴ ∑ i n ! = i 1! + i 2! + i 3! + ∑ in! 2
n =1 n=4 b + b 2 − 4a
and |z | ≥ …(i)
= i + i 2 + i 6 + 22 [from Eq. (i)] 2
= i − 1 − 1 + 22 = 20 + i a a
Also, z± ≥ |z | −
z |z |
∴ a = 20, b = 1
∴ a − b = 20 − 1 = 19 ⇒
a
b ≥ |z | −
which is a prime number. |z |
a
95 50
⇒ −b ≤|z | − ≤b
50. ∴ ∑ ir + ∑ ir ! |z |
r = −2 r=0
⇒ −b | z | ≤ | z | 2 − a ≤ b | z |
84 Textbook of Algebra

Case I −b | z | ≤ | z | 2 − a On squaring and adding Eqs. (i) and (ii), we get


⇒ |z | + b |z | −a ≥ 0
2 (1 − a ) 2 + b 2 = (2a + 1 ) 2 + (2b ) 2

− b − b 2 + 4a ⇒ 3a 2 + 3b 2 + 6a = 0
∴ |z | ≤
2 ⇒ a 2 + b 2 + 2a = 0
− b + b 2 − 4a From option (c),
and |z | ≥
2 (1 + 5a ) 2 + (3b ) 2 = (1 − 4a ) 2
Case II | z | 2 − a ≤ b | z | ⇒ 9a 2 + 9b 2 + 18a = 0
⇒ | z |2 − b | z | − a ≤ 0 ∴ a 2 + b 2 + 2a = 0

b − b 2 + 4a b + b 2 + 4a 56. From Eq. (i), we get


∴ ≤|z | ≤  1 − tan 2 θ / 2   2 tan θ / 2 
2 2 (1 − a )   +b  = 2a + 1
From Case I and Case II, we get  1 + tan θ / 2
2
 1 + tan 2 θ / 2
− b + b 2 + 4a b + b 2 + 4a θ θ
≤|z | ≤ …(ii) ⇒ (1 − a ) − (1 − a ) tan 2 + 2b tan
2 2 2 2
θ
From Eqs. (i) and (ii), we get = (2a + 1 ) + (2a + 1 ) tan 2
2
− b + b 2 + 4a b + (b 2 + 4a ) θ θ
≤|z | ≤ ⇒ (2 + a ) tan 2 − 2b tan + 3a = 0
2 2 2 2
b + b 2 + 4a θ 2b ± 4b 2 − 12a (2 + a )
∴ The greatest value of | z | is ∴ tan =
2 2 2(2 + a )
− b + b 2 + 4a
and the least value of | z | is . 2b ± 4b 2 − 12( −b 2 )
2 = [Q a 2 + b 2 + 2a = 0 ]
− 2b 2 / a
52. Here, a = 1 and b = 2
(2b ± 4b ) a 6ba − 2ab 3a a
λ = Sum of the greatest and least values of | z | = = or =− or
− 2b 2 − 2b 2 − 2b 2 b b
= b 2 + 4a = 4 + 4 = 8 θ b b b θ θ
∴ cot = − or or − = 3 cot or − cot
∴ λ2 = 8 2 3a a a 2 2
53. Here, a = 2 and b = 4 57. ∴ a 2 + b 2 + 2a = 0 ⇒ (a + 1) 2 + b 2 = 1
λ = Sum of the greatest and least value of | z |. Now, | z | = 1 = (a + 1 ) 2 + b 2

= b 2 + 4a = 16 + 8 = 24 58. Q 1 + z + z 2 + z 3 + … + z 17 = 0

∴ λ2 = 24 1 ⋅ (1 − z 18 )
∴ =0
54. Here, a = 3 and b = 6 (1 − z )

λ = Sum of the greatest and least value of | z | ⇒ 1 − z 18 = 0, 1 − z ≠ 0

= b 2 + 4a = 36 + 12 = 48 = 4 3 ∴ z 18 = 1, z ≠ 1 …(i)

⇒ λ =2 3 and 1 + z + z + z + … + z
2 3 13
=0

∴ λ2 = 12 1 ⋅ (1 − z )
14
∴ =0
(1 − z )
Sol. (Q. Nos. 55 to 57)
z −1 ⇒ 1 − z 14 = 0, 1 − z ≠ 0
Q W = = a + ib
z +2 ∴ z 14 = 1, z ≠ 1 …(ii)

55. Q z = CiS θ = e From Eqs. (i) and (ii), we get

e −1 z 14 ⋅ z 4 = 1 ⇒ 1 ⋅ z 4 = 1
∴ = a + ib
e iθ + 2 ∴ z4 = 1
⇒(cosθ + i sin θ − 1 ) = (a + ib ) (cosθ + i sin θ + 2 ) Then, z = 1, − 1, i , − i
On comparing real and imaginary parts, we get Q z ≠1
cosθ − 1 = a cosθ + 2a − b sin θ Q z = − 1, i , − i
⇒ (1 − a ) cosθ + b sin θ = 2a + 1 …(i) Hence, only z = − 1 satisfy both Eqs. (i) and (ii).
and sin θ = a sin θ + b cosθ + 2b ∴ Number of values of z is 1.
(1 − a ) sin θ − b cosθ = 2b …(ii)
Chap 01 Complex Numbers 85

59. We have, z3 = z …(i) 3 1


For x = ,y = ± [from Eq. (i)]
⇒ |z | =|z | =|z |
3 2 2
3 1
⇒ | z | (| z | − 1 ) = 0
2
For x = − ,y = ± [from Eq. (i)]
2 2
⇒ | z | = 0 and | z | 2 = 1
1 i 3 1 i 3 3 i 3 i
Now, | z | 2 = 1 ∴ Solutions are ± ,− ± , ± ,− ±
2 2 2 2 2 2 2 2
1 Hence, number of solutions is 8.
⇒ zz = 1 ⇒ z =
z 62. We have, x = a + ib
On putting this value in Eq. (i), we get
⇒ x 2 = (a 2 − b 2 ) + 2iab = 3 + 4i [given]
1
z3 =
z ∴ a − b = 3 and ab = 2
2 2
…(i)
⇒ z4 = 1 …(ii) and x = x ⋅ x = (a + ib ) [(a − b ) + 2iab ]
3 2 2 2

Clearly, Eq. (ii) has 4 solutions. = (a 3 − ab 2 − 2ab 2 ) + i [2a 2b + b(a 2 − b 2 )]


Therefore, the required number of solutions is 5. = (a 3 − 3ab 2 ) + i (3a 2b − b 3 ) = 2 + 11i [given]
60. We have, z = 9 + ai
∴ a − 3ab = 2
3 2

⇒ z 2 = (81 − a 2 ) + 18ai
and 3a 2b − b 3 = 11 …(ii)
z = (729 − 27a ) + (243a − a ) i
3 2 3
From Eq. (i), we get
According to the question, we have a 2 + b 2 = (a 2 − b 2 ) 2 + 4a 2 b 2 = 5
Im (z 2 ) = Im (z 3 )
Then, 2a 2 = 8, 2b 2 = 2
⇒ 18a = 243a − a 3 ⇒ a (a 2 − 225 ) = 0
∴ a 2 = 4, b 2 = 1
⇒ a = 0 or a = 225
2
⇒ a = 2, b = 1
But a≠0 and a = − 2, b = − 1 [Q ab = 2 ]
∴ a = 225
2 Finally, a = 2, b = 1 satisfies Eq. (ii).
∴ The sum of digits of a 2 = 2 + 2 + 5 = 9 Hence, a + b =2 + 1 =3
63. Q (1 + i ) 4 = [(1 + i ) 2 ]2
61. Let z = x + iy
= (1 + i 2 + 2i ) 2 = (1 − 1 + 2i ) 2
Q |z | =1 …( i )
∴ x + y2 =1
2 = 4i 2 = − 4 …(i)
z z 1− πi π −i
and + =1 and +
z z π +i 1+ πi
x + iy x − iy (1 − π i ) ( π − i ) ( π − i ) (1 − π i )
⇒ + =1 = +
x − iy x + iy π+1 1+π

( x + iy ) 2 + ( x − iy ) 2 π − i − πi − π + π − πi − i − π
⇒ =1 =
x2 + y 2 π +1
− 2 πi − 2i
2 (x 2 − y 2 ) = = − 2i …(ii)
⇒ =1 [from Eq. (i)] π+1
1
π 1 − π i π −i 
1 Given, z = (1 + i ) 4  + 
⇒ x2 − y 2 = ± …(ii) 4  π +i 1 + π i
2
π
From Eqs. (i) and (ii), we get =
( − 4 ) ( −2i ) = 2πi [from Eqs. (i) and (ii)]
4
1 1 3
2x 2 = 1 ± = ,  |z |  2π
2 2 2 Now,   = =4
 amp (z) π / 2
1 3 1 3
⇒ x2 = , ⇒ x = ± , ±
4 4 2 2 64. Q An = 1
1
For x = , y = ±
3
[from Eq. (i)] ⇒ A = (1 )1/n = e 2π r i /n , r = 0, 1, 2, …, n − 1
2 2 ∴ A = 1, e 2 π i /n , e 4 π i /n , e 6 πi /n , …, e 2 π (n − 1) i /n
1 3
For x = − , y = ± [from Eq. (i)] and ( A + 1 )n = 1 ⇒ A + 1 = (1 )1/n = e 2 π pi /n
2 2
86 Textbook of Algebra

 πp   10  2q π 2q π  
⇒ A = e 2 π pi /n − 1 = e p π i /n ⋅ 2i sin   , = − i  ∑  cos + i sin  − 1
 n  q = 0  11 11  
p = 0, 1, 2, …, n − 1 = − i {(sum of 11, 11th roots of unity) − 1}
 π  2π 
∴ A = 0, e π i /n ⋅ 2i sin   , e 2 π i /n 2i sin   ,..., = − i (0 − 1) = i
 n n p
32  10  2qπ 2qπ  
 π (n − 1 ) Q P = ∑ (3 p + 2 )  ∑ sin − i cos 
e π i (n − 1)/n ⋅ 2i sin   p =1  q = 1 11 11  
 n 
For n = 6, 32
4 π i /n = ∑ (3 p + 2 ) (i ) p
e = e 4 π i / 6 = e 2π i / 3 p =1

2π 2π 1 i 3 32 32
= cos + i sin =− + = 3 ∑ p(i ) p + 2 ∑ (i ) p
3 3 2 2 p =1 p =1
 π  π
and e π i /n ⋅ 2i sin   = e π i / 6 ⋅ 2i sin   32
 n  6 = 3 ∑ p(i ) p + 0 = 3S (say)
p =1
 π π
=  cos + i sin  ⋅ i 32
 6 6 where, S = ∑ p(i ) p
p =1
 3 i 1 i 3
= + i =− +
 2 2 2 2 S = 1 ⋅ i + 2 ⋅ i 2 + 3 ⋅ i 3 + … + 31 ⋅ i 31 + 32 ⋅ i 32
Hence, the least value of n is 6. iS = 1 ⋅ i 2 + 2 ⋅ i 3 + … + 31 ⋅ i 32 + 32i 33
65. Given, z1, z 2, z 3, … , z 50 are the roots of the equation (1 − i ) S = (i + i 2 + i 3 + … + i 32 ) − 32i 33
50
= ( 0 ) − 32i
∑ (z )r = 0, then
r=0 32i ⋅ (1 + i )
∴ S =−
50 50 (1 − i ) ⋅ (1 + i )
∑ (z ) r
= (z − z1 ) (z − z 2 ) (z − z 3 ) … (z − z 50 ) = Π (z − zr )
r =1 = − 16 (i − 1 ) = 16 (1 − i )
r=0

Taking log on both sides on base e, we get ∴ P = 3S = 48 (1 − i )


Given, (1 + i ) P = n(n !) ⇒ (1 + i ) ⋅ 48 (1 − i ) = n(n !)
 50  50
loge  ∑ (z )r  = ∑ loge (z − zr ) ⇒ 96 = n(n !) ⇒ 4( 4 !) = n(n !)
 
r = 0  r =1
∴ n=4
On differentiating both sides w.r.t. z, we get
1+i (1 + i ) 2 1 + i 2 + 2i
50 67. Q = = =i
1 − i (1 − i ) (1 + i )
∑ r (z )r − 1 50
2
r=0 1 n
1 + x2
50
= ∑ (z − zr ) Given,
1 + i

2
 = sin
−1
 
1 − i π
∑ (z ) r r =1  2x 
r=0
2 1 + x2
On putting z = 1 in both sides, we get ⇒ in = sin − 1  
π  2x 
50

∑r 50 ⇒
1 + x2 π n
sin − 1   = (i )
r=0 1
50
= ∑ (1 − zr )
 2x  2
∑1 r =1

1 + x2 π 
= sin  (i )n  …(i)
r=0
2x 2 
(1 + 2 + 3 + … + 50 ) 1 50
⇒ =−∑ Now, AM ≥ GM
51 r =1
(z r − 1) 1
x+
x ≥1 ⇒ x + 1 ≥1
2
= − ( − 5λ ) [given]
50 2 2x
× 51  π 
⇒ 2 = 5λ ⇒ sin  (i )n  ≥ 1 [Q −1 ≤ sin θ ≤ 1]
51 2 
⇒ λ =5 π 
∴ sin  (i )n  = 1
10
 2q π 2q π  2 
66. Q ∑ sin − i cos 
q =1 11 11  ⇒ n = 4, 8, 12, 16, …
 102q π 2q π  ∴ Least positive integer, n = 4
= − i ∑  cos + i sin 
q = 1 11 11 
Chap 01 Complex Numbers 87

68. (A) → ( p,q ), (B) → ( p,r ),(C) → ( p,r ,s )  13 + 5 13 − 5 


(B) 5 − 12i = ±  −i  = ± (3 − 2i )
a  2 2 
If z ± = b, where a > 0 and b > 0, then
z  13 − 5 13 + 5 
and − 5 − 12i = ±  −i  = ± (2 − 3i )
− b + b 2 + 4a b + b 2 + 4a  2 2 
≤ |z | ≤
2 2 ∴ z = 5 − 12i + −5 − 12i = ± (3 − 2i ) ± (2 − 3i )
(A) Here, a = 1 and b = 2
= 5 − 5i , − 1 − i , − 5 + 5i , 1 + i
Then, − 1 + 2 ≤ | z | ≤ 1 + 2
∴ z1 = 5 − 5i , z 2 = − 1 − i ,
∴ G =1 + 2 z 3 = − 5 + 5i and z 4 = 1 + i
and L = −1 + 2
∴ | z1| 2 + | z 2 | 2 + | z 3| 2 + | z 4 | 2 = 50 + 2 + 50 + 2
⇒ G − L =2 [natural number and prime number]
= 104 = 8 × 13
(B) Here, a = 2 and b = 4
 17 + 8 17 − 8 
Then, − 2 + 6 ≤ | z | ≤ 2 + 6 (C) 8 + 15i = ±  +i 
 2 2 
∴ G =2 + 6
 5 3i  1
and L = −2 + 6 =± + =± (5 + 3i )
 2 2 2
⇒ G − L = 4 [natural number and composite number]
 17 − 8 17 + 8
(C) Here, a = 3 and b = 6 and − 8 − 15i = ±  −i 
 2 2 
Then, − 3 + 2 3 ≤ | z | ≤ 3 + 2 3
 3 5  1
∴ G =3 + 2 3 =± − i = ± (3 − 5i )
 2 2  2
and L = −3 + 2 3
∴ z = 8 + 15i + − 8 − 15i
⇒ G − L =6
1 1
[natural number, composite number and perfect number] =± (5 + 3i ) ± (3 − 5i )
2 2
69. (A) → (q), B → (q, r), C → (q, s)
1 1
We know that, z= (8 − 2i ), ( − 2 − 8i ),
2 2
 | z | − Re (z ) | z | − Re (z )  1 1
z =± +i  ( − 8 + 2i ), (2 + 8i )
 2 2  2 2
 | z | + Re (z ) | z | − Re (z )  ∴ z1 = 2 ( 4 − i ), z 2 = 2 ( − 1 − 4i )
If Im(z ) > 0 = ±  −i 
 2 2  z 3 = 2 ( − 4 + i ) and z 4 = 2 (1 + 4i )
If Im (z ) < 0
∴ | z1| + | z 2 | 2 + | z 3| 2 + | z 4 | 2 = 34 + 34 + 34 + 34
2
 10 + 6 10 − 6  = 136 = 17 × 8
(A) 6 + 8i = ±  +i 
 2 2 
70. (A) → (p, q, r, t);(B) → (p,s);(C) → (p, r)
= ± (2 2 + i 2 ) (A) Here, the last digit of 143 is 3. The remainder when 861 is
= ± 2 (2 + i ) divided by 4 is 1. Then, press switch number 1 and we get
3. Hence, the digit in the units place of (143 ) 861 is 3.
 10 − 6 10 + 6 
and − 6 + 8i = ±  +i  ∴ λ =3
 2 2 
Next, the last digit of 5273 is 3. The remainder when 1358
= ± ( 2 + i 2 2 ) = ± 2 (1 + 2i ) is divided by 4 is 2. Then, press switch number 2 and we
∴ z = 6 + 8i + − 6 + 8i get 9. Hence, the digit in the units place of (5273 )1358 is 9.
∴ µ =9
= ± 2 (2 + i ) ± 2 (1 + 2i )
Hence, λ + µ = 3 + 9 = 12
= 3 2 (1 + i ), 2 (1 − i ), − 3 2 (1 + i ), 2 ( − 1 + i ) which is divisible by 2, 3, 4 and 6.
∴ z1 = 3 2 (1 + i ), z 2 = 2 (1 − i ), (B) Here, the last digit of 212 is 2. The remainder when 7820
is divided by 4 is 0. Then, press switch number 0 and we
z 3 = − 3 2 (1 + i ) get 6. Hence, the digit in the unit’s place of (212 ) 7820 is 6.
and z 4 = 2 (− 1 + i ) ∴ λ =6
∴ | z 1 | + | z 2 | + | z 3| + | z 4 |
2 2 2 2 Next, the last digit of 1322 is 2. The remainder when 1594
is divided by 4 is 2. Then, press switch number 2 and we
= 36 + 4 + 36 + 4 = 80 which is divisible by 8. get 4.
88 Textbook of Algebra

Hence, the digit in the unit’s place of (1322 )1594 is 4. 1 + z 1 + z2 + z3


⇒ =0
∴ µ=4 3
z 1+ z2 + z3 1
Hence, λ + µ = 6 + 4 = 10, which is divisible by 2 and 5. ⇒ =−
3 3
(C) Here, the last digit of 136 is 6. Therefore, the unit’s place
 1 
of (136 ) 786 is 6. ∴ Centroid of the triangle is  − , 0 .
 3 
∴ λ =6
Next, the last digit of 7138 is 8. The remainder when So, the circumcentre and centroid of the triangle coincide.
Hence, required triangle is an equilateral triangle.
13491 is divided by 4 is 3. Then, press switch number 3
and we get 2. Hence, unit’s place of (7138 )13491 is 2. Therefore, Statement-1 is true. Also, z1, z 2 and z 3 represent
vertices of an equilateral triangle, if
∴ µ =2
z12 + z 22 + z 32 − (z1z 2 + z 2 z 3 + z 3z1 ) = 0.
Hence, λ + µ =6 + 2 =8
Therefore, Statement-2 is false.
which is divisible by 2 and 4.
75. We have,
71. (A) → (r); ( B) → (p,s); ( C) → (q, t)
|z − 1| + |z − 8| = 5 …(i)
a Here, z1 = 1, z 2 = 8 and 2a = 5
If z − = b, where a > 0 and b > 0, then
z Now, | z1 − z 2 | = | 1 − 8 | = | − 7 | = 7
− b + b 2 + 4a b + b 2 + 4a ∴ 2a = 5 < 7
≤ |z | ≤
2 2 Therefore, locus of Eq. (i) does not represent an ellipse. Hence,
Statement-1 is false. Statement-2 is true by the property of
b + b 2 + 4a − b + b 2 + 4a ellipse.
∴ λ= and µ =
2 2 76. Since, z1, z 2 and z 3 are in AP.
(A) Here, a = 6 and b = 5
∴ 2z 2 = z1 + z 3
∴ λ = 6 and µ = 1
z + z3
⇒ λµ + µ λ = 61 + 1 6 = 7 ⇒ z2 = 1
2
and λµ − µ λ = 61 − 1 6 = 5 It is clear that, z 2 is the mid-point of z1 and z 3.
(B) Here, a = 7 and b = 6 ∴ z1, z 2 and z 3 are collinear.
∴ λ = 7 and µ = 1 Statement-1 is true, Statement-2 is true; Statement-2 is a
∴ λµ + µ λ = 71 + 1 7 = 8 correct explanation of Statement-1.
and λµ − µ λ = 71 − 1 7 = 6 77. Principal argument of a complex number depend upon
quadrant and principal argument lies in ( − π , π ].
(C) Here, a = 8 and b = 7 Hence, Statement-1 is always not true and Statement-2 is
∴ λ = 8 and µ = 1 obviously true.
⇒ λµ + µ λ = 81 + 1 8 = 9 π
78. We have, C1 : arg (z ) =
and λµ − µ λ = 81 − 1 8 = 7 4
 y  π
72. Statement-1 is false because 3 + 7i > 2 + 4i is meaningless in ⇒ tan − 1   = [ let z = x + iy ]
 x 4
the set of complex number as set of complex number does not
hold ordering. But Statement-2 is true. y π
⇒ = tan = 1
73. Statement-1 is false as x 4
⇒ y =x
(cos θ + i sin φ )n ≠ cos nθ + i sin nφ
2 ∴ C1 : y = x …(i)
 π π π π
Now,  cos + i sin  = cos + i sin 3π
 4 4 2 2 C 2 : arg(z ) =
4
=i [by De-Moivre’s theorem] −1 y  3π
⇒ tan   = [ let z = x + iy ]
∴ Statement-2 is true.  x 4
74. We have, y 3π
⇒ = tan = −1
| 3z1 + 1 | = | 3z 2 + 1 | = | 3z 3 + 1 | x 4
 1  ⇒ y =−x
∴ z1, z 2 and z 3 are equidistant from  − , 0 and circumcentre
 3  ∴ C2 : y = − x …(ii)
 1  and C 3 : arg (z − 5 − 5i ) = π
of triangle is  − , 0 .
 3   y − 5
⇒ tan − 1   =π [ let z = x + iy ]
Also, 1 + z1 + z 2 + z 3 = 0  x − 5
Chap 01 Complex Numbers 89

y −5 On putting x = ω in Eq. (i), we get


⇒ = tan π = 0 ⇒ y = 5
x −5 ω5 − 1
= (ω − α 1 ) (ω − α 2 ) (ω − α 3 ) (ω − α 4 )
∴ C3 : y = 5 …(iii) ω −1
We get the following figure. ω2 − 1
⇒ = (ω − α 1 ) (ω − α 2 )(ω − α 3 ) (ω − α 4 ) ...(ii)
Y ω −1

x
=
and putting x = ω 2 in Eq. (i), we get

:y
1
C
B C3 : y=5 ω10 − 1
= (ω 2 − α 1 ) (ω 2 − α 2 ) (ω 2 − α 3 )(ω 2 − α 4 )
(–5, 5) A(5, 5) ω2 − 1
C2 : y= – x ω −1
X ⇒ = (ω 2 − α 1 ) (ω 2 − α 2 ) (ω 2 − α 3 )(ω 2 − α 4 ) ...(iii)
O(0, 0) ω 2 −1
On dividing Eq. (ii) by Eq. (iii), we get
ω − α1 ω − α 2 ω − α 3 ω − α 4 (ω 2 − 1 ) 2
⋅ 2 ⋅ 2 ⋅ 2 =
∴ Area of the region bounded by C1, C 2 and C 3 ω − α1 ω − α 2 ω − α 3 ω − α 4
2
(ω − 1 ) 2
1 5−0 5−0 ω 4 + 1 − 2ω 2 ω + 1 − 2 ω 2
= = 25 = =
2 −5 − 0 5 − 0 ω 2 + 1 − 2ω ω 2 + 1 − 2 ω
∴ Statement-1 is false. − ω 2 − 2ω 2 − 3ω 2
= = =ω
Now, OA = 5 2, OB = 5 2 and AB = 10 − ω − 2ω − 3ω
(OA ) 2 + (OB ) 2 = ( AB ) 2 and OA = OB z +z
Q 82. Let z = x + iy , then =x
2
Therefore, the boundary of C1, C 2 and C 3 constitutes right
∴ From given relation, we get
isosceles triangle.
Hence, Statement-2 is true. ⇒ x = x + iy − 1
⇒ x = ( x − 1 ) + iy
z 2 z 3 − (z 2 z 3 ) 1
79. Since, Im ( z 2 z 3 ) = = {z 2 z 3 − z 2 z 3 } ⇒ x 2 = (x − 1)2 + y 2 ⇒ 2x = 1 + y 2
2i 2i
1 If z1 = x1 + iy1 and z 2 = x 2 + iy 2
z1 Im ( z 2 z 3 ) = {z1z 2 z 3 − z1z 2 z 3} ...(i)
2i Then, 2 x1 = 1 + y12 ...(i)
1
Similarly, z 2 Im ( z 3 z1 ) = {z 2 z 3 z1 − z 2 z1 z 3} ...(ii) and 2x 2 = 1 + y 2 2
...(ii)
2i On subtracting Eq. (ii) from Eq. (i), we get
1
and z 3 Im ( z1 z 2 ) = {z 3 z1 z 2 − z 3z1z 2} ...(iii) 2 ( x1 − x 2 ) = y12 − y 22
2i
2 ( x1 − x 2 ) = (y1 + y 2 ) (y1 − y 2 ) ...(iii)
On adding Eqs. (i), (ii) and (iii), we get
But, given that arg (z1 − z 2 ) = π / 4
z1 Im ( z 2 z 3 ) + z 2 Im ( z 3 z1 ) + z 3 Im ( z1 z 2 ) = 0
y − y 2 π y − y2
Therefore, this is proved. Then, tan − 1  1  = ⇒ 1 =1
 x1 − x 2  4 x1 − x 2
80. Since, z1, z 2 and z 3 are the roots of
∴ y1 − y 2 = x1 − x 2 ...(iv)
x 3 + 3ax 2 + 3bx + c = 0,
From Eqs. (iii) and (iv), we get
we get z1 + z 2 + z 3 = − 3a
y1 + y 2 = 2 [Q y1 − y 2 ≠ 0]
z1 + z 2 + z 3
⇒ = −a ∴ Im (z1 + z 2 ) = 2
3
Hence, the imaginary part (z1 + z 2 ) is 2.
and z1z 2 + z 2 z 3 + z 3z1 = 3b
83. (i) LHS = (a 2 + b 2 + c 2 − bc − ca − ab )
Hence, the centroid of the ∆ABC is the point of affix ( − a ).
( x 2 + y 2 + z 2 − yz − zx − xy )
Now, the triangle will be equilateral, if
= (a + bω + c ω 2 ) (a + bω 2 + cω )
z12 + z 22 + z 32 = z1z 2 + z 2 z 3 + z 3z1
( x + yω + zω 2 ) ( x + yω 2 + zω )
⇒ (z1 + z 2 + z 3 ) 2 = 3(z1z 2 + z 2 z 3 + z 3z1 )
= {(a + bω + cω ) ( x + yω + zω 2 )}
2

⇒ ( − 3a ) = 3(3b )
2
{(a + bω 2 + cω ) ( x + yω 2 + zω )}
Therefore, the condition is a = b. 2
= {ax + cy + bz + ω (bx + ay + cz )
81. Q x 5 − 1 = 0 has roots 1, α 1, α 2, α 3, α 4 . + ω 2 (cx + by + az )} × {ax + cy + bz + ω 2
(bx + ay + cz ) + ω (cx + by + az )}
∴ (x 5 −1) = (x − 1) (x − α1 ) (x − α 2 ) (x − α 3 ) (x − α 4 ) = ( X + ωΖ + ω 2 Y ) ( X + ω 2 Z + ω Y )
x 5 −1 = RHS
⇒ = (x − α1 ) (x − α 2 ) (x − α 3 ) (x − α 4 ) ...(i)
x −1
90 Textbook of Algebra

(ii) LHS = (a 3+ b 3 + c 3 − 3abc ) ( x 3+ y 3 + z 3 − 3 xyz ) Using Eqs. (i) and (ii), then equations of lines are
= (a + b + c ) (a 2 + b 2 + c 2 − ab − bc − ca ) × z + z 3 (z − z )
( x + y + z )( x 2+ y 2 + z 2 − xy − yz − zx ) + − 11 = 0
2 2i
= (a + b + c ) ( x + y + z ) 3 (z + z ) (z − z )
(a 2 + b 2 + c 2 − ab − bc − ca ) × and − +7=0
2 2i
( x 2 + y 2 + z 2 − xy − yz − zx ) [using (i) part] i.e., (1 − 3i ) z + (1 + 3i ) z − 22 = 0
= (ax + ay + az + bx + by + bz + cx + cy + cz ) and (3 + i ) z + (3 − i ) z + 14 = 0
( X 2 + Y 2 + Z 2 − YZ − ZX − XY ) 1+i
86. Putting = x in LHS, we get
= {(ax + cy + bz ) + (cx + by + az ) + (bx + ay + cz )} 2
( X 2 + Y 2 + Z 2 − YZ − ZX − XY ) 2
LHS = (1 + x ) (1 + x 2 ) (1 + x 2 ) … (1 + x 2 )
n

= ( X + Y + Z ) ( X 2 + Y 2 + Z 2 − YZ − ZX − XY ) 2 n
(1 − x ) (1 + x ) (1 + x 2 ) (1 + x 2 ) … (1 + x 2 )
=X 3
+ Y + Z − 3 XYZ = RHS
3 3 =
(1 − x )
84. Let z = x + iy 2 n
(1 − x 2 ) (1 + x 2 ) (1 + x 2 ) … (1 + x 2 )
∴ |z | = x + y
2 2 2 =
(1 − x )
∴ x 2 + y 2 − 2i ( x + iy ) + 2c (1 + i ) = 0 2 2 n
(1 − x 2 ) (1 + x 2 ) … (1 + x 2 )
=
( x + y + 2y + 2c ) + i ( − 2 x + 2c ) = 0
2 2
(1 − x )
On comparing the real and imaginary parts, we get n n n
(1 − x 2 ) (1 + x 2 ) 1 − ( x 2 ) 2
x 2 + y 2 + 2y + 2c = 0 …(i) = =
(1 − x ) (1 − x )
and − 2 x + 2c = 0 …(ii) 2n
i
From Eqs. (i) and (ii), we get 1− 
 2  1 + i
y 2 + 2y + c 2 + 2c = 0 =
1 + i Q x = 2 
−2 ± 4 − 4(c 2 + 2c ) 1− 
⇒ y = = − 1 ± (1 − c 2 − 2c )  2 
2 1
1− (1 )
Q x and y are real.
22
n
(1 + i )  1 
∴ 1 − c 2 − 2c ≥ 0 or c 2 + 2c + 1 ≤ 2 = ⋅ = (1 + i ) 1 − n  = RHS
 1 − i  (1 + i )  22 
 
(c + 1 ) 2 ≤ ( 2 ) 2 ⇒ − 2 − 1 ≤ c ≤ 2 − 1  2 
∴ 0 ≤c ≤ 2 −1 [Q given c ≥ 0] 87. Since, arg (z − 3i ) = 3π / 4 is a ray which is start from 3i and
makes an angle 3 π / 4 with positive real axis as shown in the
Hence, the solution is z = x + iy = c + i ( − 1 ± 1 − c 2 − 2c ) figure.
for 0 ≤ c ≤ 2 − 1 x+y=3 Y
and z = x + iy ≡ no solution for c > 2 − 1
85. Let z = x + iy (0, 3) 3
z +z y=x+ –
2
∴ Re (z ) = x = …(i) 2
2
z −z 3/2
and Im (z ) = y = …(ii)
2i 1
The equation (2 − i ) z + (2 + i ) z + 3 = 0 can be written as X′ X
1/2 O
2(z + z ) − i (z − z ) + 3 = 0
or 4 x + 2y + 3 = 0 Y′
∴ Slope of the given line, m = − 2 ∴ Equation of ray in cartesian form is
Let slope of the required line be m1, then y − 3 = tan (3 π / 4 ) ( x − 0 )
m −m   ⇒ 1 =  m1 + 2 ⇒ ± 1 = m1 + 2 or y − 3 = − x or x + y = 3
tan 45 ° =  1
1 + m1 
m  1 − 2m 1 1 − 2m1 and arg (2z + 1 − 2i ) = π / 4
1   1 
∴ m1 = − , 3 ⇒ arg 2 z + − i   = π / 4
3   2 
∴ Equation of straight lines through ( − 1, 4 ) and having slopes  1 
or arg(2 ) + arg z + − i  = π / 4
1 1
− and 3 are y − 4 = − ( x + 1 ) and y − 4 = 3 ( x + 1 )  2 
3 3  1 
⇒ x + 3y − 11 = 0 and 3 x − y + 7 = 0 or 0 + arg z + − i  = π / 4
 2 
Chap 01 Complex Numbers 91

  1  On subtracting Eq. (i) from Eq. (ii), we get


or arg z −  − + i   = π / 4
  2  z (c −2 − a −1b −1 ) = 2c −1 − a −1 − b −1
1
which is a ray that start from point − + i and makes an angle 2c −1 − a −1 − b −1
2 Hence, z=
π / 4 with positive real axis as shown in the figure. c −2 − a −1b −1
∴Equation of ray in cartesian form is which is a required point.
y − 1 = 1 [ x − ( − 1 / 2 )] ⇒ y = x + 3 / 2 90. Q AD ⊥ BC
From the figure, it is clear that the system of equations has no ∴ AP is also perpendicular to BC.
solution. A(z1)
88. Let a = r cos α and 0 = r sin α …(i)
So that, a 2 + 02 = r 2
∴ r = |a | O
Then, a = | a | cos α [from Eq. (i)]
C(z3)
∴ cos α = ± 1 (z2)B D
Then, cos α = 1 or − 1 according as a is + ve or – ve and
sin α = 0. P(z)
Hence, α = 0 or π according as a is + ve and – ve.  z −z  π
Then, arg  1  =
Again, let 0 = r1 cos β or b = r1 sin β …(ii) z3 − z2 2
So that, 0 2 + b 2 = r12  z −z 
∴ Re  1  =0
∴ r1 = | b | z3 − z2
From Eq. (ii), we get b = | b | sin β z1 − z z −z
+ 1
∴ sin β = ± 1 z3 − z2 z 3 − z 2
⇒ =0
Then, sin β = 1 or − 1 according as b is + ve or – ve and cosβ = 0. 2
π π z1 − z z1 − z
Hence, β = or − according as b is + ve or −ve. ⇒ + =0 …(i)
2 2 z3 − z2 z 3 − z 2
89. Let two non-parallel straight lines PQ, RS meet the circle But O is the circumcentre of ∆ABC, then
| z | = r in the points a, b and c. OP = OA = OB = OC
P a | z | = | z1 | = | z 2 | = | z 3 |
b S On squaring the above relation, we get
O
z | z | 2 = | z 1| 2 = | z 2 | 2 = | z 3 | 2
Q
⇒ zz = z1z1 = z 2z 2 = z 3z 3
c z z
R
From first two relations 1 = …(ii)
z z1
Then, | a | = r , | b | = r and | c | = r or | a | = | b | = | c | = r
2 2 2 2
z z
From first and third relation 2 = …(iii)
∴ a a = b b = c c = r 2, z z2
r2 r2 r2 z z
then a = , b = and c = and from first and fourth relation 3 = …(iv)
a b c z z3
1 z1
z z −1
z1 − z
Points a, b and z are collinear, then a a 1 =0 From Eq. (i), we get + z =0 …(v)
z3 − z2 z3 − z2
b b 1
z z
∴ z ( a − b ) − z (a − b ) + ab − a b = 0 From Eqs. (ii), (iii), (iv) and (v), we get
r 2 r 2 r 2a r 2b z
⇒ z  −  − z (a − b ) + − =0 −1
a b b a z1 − z z1
+ =0
z3 − z2 z

z
On dividing both sides by r 2 (b − a ), we get
z3 z2
z z
+ = a −1 + b −1 …(i)  z1 − z   z z   z1 − z 
ab r 2 ⇒   1 + 2 3  = 0 Q z − z ≠ 0 
z3 − z2  zz1   3 2 
For RS, replace a = b = c in Eq. (i), then
z z z 2z3 z z
+ = 2c −1 …(ii) ⇒ 1+ =0 ⇒z =− 2 3
c2 r2 z z1 z1
92 Textbook of Algebra

91. From the figure, 92. Given, OA = 1 and | z | = 1


α = ( arg (z ) − arg (ω )) …(i)
2
Y P0(z0)
α α
and for every α, sin 2 ≤  …(ii)
2  2

A(z)
P(z)
Imaginary axis

A(1)
|z| |z – ω | O
X

B(ω)
α Q(zz0)
|ω |
O Real axis ∴ = |z − 0 | = |z | = 1
OP
∴ = OA
OP
In ∆OAB, from cosine rule = |z 0 − 0 | = |z 0 |
OP0
( AB ) 2 = (OA ) 2 + (OB ) 2 − 2OA ⋅ OB cos α and = | zz 0 − 0 | = | zz 0| = | z | | z 0| = 1 | z 0| = | z 0|
OQ
⇒ | z − ω | 2 = | z | 2 + | ω | 2 − 2 | z | | ω | cos α ∴ = OQ
OP0
⇒ | z − ω | 2 = (| z | − | ω | ) 2 + 2 | z | | ω | (1 − cos α )  z − 0 z  z z 
Also, ∠P0OP = arg  0  = arg  0  = arg  0 
 z − 0 z zz 
α
⇒ | z − ω | 2 = (| z | − | ω | ) 2 + 4 | z | | ω |sin 2
2 z z  z z 
= arg  20  = arg  0  = − arg (z z 0 )
2  | |   1 
α z
⇒ | z − ω | 2 ≤ (| z | − | ω | ) 2 + 4 | z | | ω |   [from Eq. (ii)]
 2  1 
= − arg (z z 0 ) = arg  
⇒ | z − ω | ≤ (| z | − | ω | ) + α
2 2 2
[Q | z | ≤ 1, | ω | ≤ 1] z z0
| z − ω | ≤ (| z | − | ω | ) + ( arg (z ) − arg (ω )) 2
2 2
[from Eq. (i)]  1−0 
= arg   = ∠AOQ
 z z 0 − 0
I. Aliter
Let z = r (cosθ + i sin θ ) and ω = r1 (cos θ1 + i sin θ1 ) , Thus, the triangles POP0 and AOQ are congruent.
then | z | = r and | ω | = r1 ∴ PP0 = AQ
Also, arg (z ) = θ and arg (ω ) = θ1 | z − z 0| = | z z 0 − 1|
and r ≤ 1 and r1 ≤ 1 [Q given | z | ≤ 1, | ω | ≤ 1 ] 93. Let the equation of line passing through the origin be
We have, z − ω = (r cos θ − r1 cos θ1 ) + i (r sin θ − r1 sin θ1 ) a z + az = 0 …(i)
∴ | z − ω | 2 = (r cos θ − r1 cos θ1 ) 2 + (r sin θ − r1 sin θ1 ) 2 According to the question, z1, z 2, …, zn all lie on one side of
⇒ | z − ω | 2 = r 2 + r12 − 2rr1 cos (θ − θ1 ) line (i)
∴ a zi + azi > 0 or < 0 for all i = 1, 2, 3, …, n …(ii)
= (r − r1 ) 2 + 2rr1 − 2rr1 cos (θ − θ1 ) n n
= (r − r1 ) 2 + 2rr1 (1 − cos (θ − θ1 )) ⇒ a ∑ zi + a ∑ zi > 0 or < 0 …(iii)
i =1 i =1
 θ − θ1   n
= (r − r1 ) 2 + 4rr1 sin 2   ⇒
n
∑ zi ≠ 0
n
 2  If ∑ zi = 0, then ∑ zi = 0,
i =1  i =1 i =1
2
 θ − θ1  n n 
≤ (r − r1 ) + 4rr1 
2
 [Q|sin θ | ≤ | θ | ] hence a ∑ zi + a ∑ zi = 0 
 2  i =1 i =1 
= (r − r1 ) 2 + rr1(θ − θ1 ) 2 From Eq. (ii), we get
≤ (r − r1 ) 2 + (θ − θ1 ) 2 [Qr , r1 ≤ 1] a zi + azi > 0 or < 0 for all i = 1, 2, 3, …, n
⇒ | z − ω | 2 ≤ (| z | − | ω | ) 2 + ( arg z − arg ω ) 2 a zi zi azi zi
⇒ + > 0 or < 0
zi zi
II. Aliter
a a
Let z = r cos θ ⇒ | zi | 2  +  > 0 or < 0
and ω = r1 cos θ1 zi zi 
a a
∴ r 2 + r12 − 2rr1 cos (θ − θ1 ) ≤ r 2 + r12 − 2rr1 + (θ − θ1 ) 2 ⇒ + > 0 or < 0 for all i = 1, 2, 3, …, n
zi zi
 θ − θ1   θ − θ1 
2
Q r , r1 ≤ 1 
⇒ rr1 sin 2   ≤   2
1 1 1
 2   2  ⇒ , , …, lie on one side of the line a z + az = 0
and sin x ≤ x 
2
z1 z 2 zn
Chap 01 Complex Numbers 93

n 1 n 1
or a ∑ +a ∑ > 0 or < 0 z2
i = 1 zi i = 1 zi z1
r
r
n 1  n 1 n 1 o
Therefore, ∑ ≠ 0 If ∑ = 0, then ∑ =0
i = 1 zi  i = 1 zi i = 1 zi r
n 1 n 1 
⇒ a ∑ +a ∑ = 0 z3
i = 1 zi i = 1 zi  From the given condition,
94. Given, | a | | b | = ab 2c ; | a | = | c |; az 2 + bz + c = 0, then we r1 r2 r3
have to prove that | z | = 1 r2 r3 r1 = 0
On squaring, we get r3 r1 r2
| a | 2 | b | 2 = a b 2c and | a | 2 = | c | 2
⇒ r13 + r23 + r33 − 3r1r2 r3 = 0
⇒ a a b b = a b 2c and a a =c c
1
⇒ a b =b c and a a = c c …(i) ⇒ (r1 + r2 + r3 ) {(r1 − r2 ) 2 + (r2 − r3 ) 2 + (r3 − r1 ) 2 } = 0
2
If z1 and z 2 are the roots of az 2 + bz + c = 0 Since, r1 + r2 + r3 ≠ 0,
Then, z1 and z 2 are the roots of a ( z ) 2 + b z + c = 0 …(A) then (r1 − r2 ) 2 + (r2 − r3 ) 2 + (r3 − r1 ) 2 = 0
b c
∴ z1 + z 2 = − , z1z 2 = It is possible only when
a a
 …(ii) r1 − r2 = r2 − r3 = r3 − r1 = 0
b c
and z1 + z 2 = − , z1z 2 =  ∴ r1 = r2 = r3
a a 
1 1 z1 + z 2 − b / a b b and | z1 | = | z 2 | = | z 3 | = r [say]
Q + = = = − = − = z1 + z 2 Hence, z1, z 2, z 3 lie on a circle with the centre at the origin.
z1 z 2 z1 z 2 c /a c a
[from Eqs. (i) and (ii)]
1 1 z1 + z 2 −b / a (ii) Again, in ∆ oz 2 z 3 by Coni method
and + = =
z1 z 2 z1 z 2 c /a  z − 0 z 
arg  3  = ∠z 2 oz 3 ⇒ arg  3  = ∠z 2 oz 3 …(i)
b a bc b  z 2 − 0 z2
=− =− = − = z1 + z 2
c ca a a In ∆ z 2z1z 3 by Coni method
[from Eqs. (i) and (ii)]  z − z1  1
1 1 arg  3  = ∠z 2 z1 z 3 = ∠z 2 oz 3 [property of circle]
Now, it is clear that z1 = and z 2 =  2
z − z 1 2
z1 z2
1 z 
= arg  3  [from Eq. (i)]
Then, | z1 | 2 = 1 and | z 2 | 2 = 1 2  z1 
Hence, |z | = 1 z   z − z1 
∴ arg  3  = 2 arg  3 
Conversely For az 2 + bz + c = 0, we have to prove  z1   z 2 − z1 
| z | = 1 ⇒| a | | b | = a b 2c 2
z   z 3 − z1 
and |a | = |c | Hence, arg  3  = arg  
 z1   z 2 − z1 
1
| z | = 1 ⇒ | z | 2 = 1 ⇒ z z = 1 ⇒z =
z 96. We know that,
From Eq. (A), we get Re (z1z 2 ) ≤ | z1z 2 |
 1
2
 1 ∴ | z1 | 2 + | z 2 | 2 + 2 Re (z1 z 2 ) ≤ | z1 | 2 + | z 2 | 2 + 2 | z1 z 2|
a   + b   + c = 0 or cz 2 + b z + a = 0
z z ⇒ | z1 + z 2 | 2 ≤ | z1 | 2 + | z 2 | 2 + 2 | z1 | | z 2 | …(i)
Also, az + bz + c = 0, on comparing
2
Also, AM ≥ GM
2
c b a  1 
= = ( c | z1 | ) 2 +  | z | 1 /2
a b c  c 2   1 
∴ ≥  c ⋅| z1 | 2 ⋅ | z 2 | 2  [Qc > 0]
∴ a a = c c and a b = b cs 2  c 
⇒ | a | = | c | and | a | | b | = a b 2c 1
⇒ c | z 1| + | z 2 | ≥ 2 | z 1 | | z 2 |
2 2

95. (i) Let z1 = r1 (cos α + i sin α ), c


1
z 2 = r2 (cos β + i sin β ) and z 3 = r3 (cos γ + i sin γ ) ∴ | z1 | 2 + | z 2 | 2 + 2 | z1 | | z 2 | ≤ | z1 | 2 + | z 2 | 2 + c | z1 | 2 + | z 2 | 2
c
∴ | z1 | = r1, | z 2 | = r2, | z 3 | = r3 −1
⇒ | z1| + | z 2 | + 2 | z1 | | z 2 | ≤ (1 + c ) | z1 | + (1 + c ) (| z 2 | 2 )
2 2 2
and arg (z1 ) = α , arg (z 2 ) = β, arg (z 3 ) = γ
…(ii)
94 Textbook of Algebra

From Eqs. (i) and (ii), we get 98. Let z be the complex number corresponding to the orthocentre
| z1 + z 2 | 2 ≤ (1 + c ) | z1 | 2 + (1 + c −1) | z 2 | 2 O, since AD ⊥ BC , we get
A(z 1)
Aliter
Here, (1 + c ) | z1 | 2 + (1 + c −1 ) | z 2 | 2 − | z1 + z 2 | 2
F
 1 E
= (1 + c ) z1z1 + 1 +  z 2 z 2 − (z1 + z 2 ) ( z1 + z 2 )
 c O(z)

 1
= (1 + c ) z1z1 + 1 +  z 2 z 2 − z1z1 − z1z 2 − z 2 z1 − z 2 z 2
 c
1 B(z 2 ) D C(z 3 )
= c z1 z1 + z 2 z 2 − z1z 2 − z 2 z1
c
1 2  z − z1  π
= {c z1z1 + z 2 z 2 − cz1z 2 − cz 2 z1 } arg   =
c z2 − z3 2
1 z − z1
= {cz1 (cz1 − z 2 ) − z 2 (cz1 − z 2 )} i. e. is purely imaginary.
c z2 − z3
1 1
= (cz1 − z 2 ) (cz1 − z 2 ) = (cz1 − z 2 ) (cz1 − z 2 )  z − z1  z − z1 z − z1
c c i. e. Re   = 0 or + =0 ...(i)
1 z2 − z3 z2 − z3 z2 − z3
= | cz1 − z 2 | ≥ 0 as c > 0
2
c z − z2 z − z2
Similarly, + =0 [Q BE ⊥ CA ] ...(ii)
 1 z 3 − z1 z 3 − z1
∴ (1 + c ) | z1| 2 + 1 +  | z 2 | 2 − | z1 + z 2 | 2 ≥ 0
 c
From Eq. (i), we get
 1
Hence, | z1 + z 2 | 2 ≤ (1 + c ) | z1| 2 + 1 +  | z 2 | 2 (z − z 2 ) ( z 2 − z 3 )
 c z = z1 − ...(iii)
(z 2 − z 3 )
97. If z be the complex number corresponding to the circumcentre
O, then we have From Eq. (ii), we get
OA = OB = OC (z − z 2 ) ( z 3 − z1 )
z = z2 − ...(iv)
A(z 1) (z 3 − z1 )
Eliminating z from Eqs. (iii) and (iv), we get
( z − z1 ) (z − z 2 ) ( z 3 − z1 )
z1 − z 2 = (z 2 − z 3 ) −
(z) (z 2 − z 3 ) (z 3 − z1 )
O
or (z − z1 ) ( z 2 − z 3 ) ( z 3 − z1 ) − (z − z 2 ) (z 3 − z1 ) (z 2 − z 3 )
C(z 3)
B(z 2) = ( z1 − z 2 ) (z 2 − z 3 ) (z 3 − z1 )
⇒ | z − z1 | = | z − z 2 | = | z − z 3 | or z {( z 2 − z 3 ) (z 3 − z1 ) − (z 3 − z1 ) (z 2 − z 3 )}
⇒ | z − z1 | 2 = | z − z 2 | 2 = | z − z 3 | 2 = ( z1 − z 2 ) (z 2 − z 3 ) (z 3 − z1 ) + z1 (z 2 − z 3 ) ( z 3 − z1 )
− z 2 ( z 3 − z1 ) (z 2 − z 3 )
⇒ (z − z1 ) ( z − z1 ) = (z − z 2 ) ( z − z 2 )
⇒ z [ z 2 z 3 − z 2 z1 − z 3z 3 + z 3z1 − z 3z 2 + z 3z 3 + z1z 2 − z1 z 3 ]
= (z − z 3 ) ( z − z 3 ) …(i)
From first two members of Eq. (i), we get = ( z1 − z 2 ) {z 2 z 3 − z 2z1 − z 32 + z 3z1 }

z (z 2 − z1 ) = z1 (z − z1 ) − z 2 (z − z 2 ) …(ii) + ( z 2 − z 3 ) (z 3z1 − z12 ) + ( z 3 − z1 ) (z 2 z 3 − z 22 )


and from last two members of Eq. (i), we get = − {z12 ( z 2 − z 3 ) + z 22 ( z 3 − z1 ) + z 32 ( z1 − z 2 )}
z (z 3 − z 2 ) = z 2 (z − z 2 ) − z 3 (z − z 3 ) …(iii) + {z1z 2 z 3 − z 2 z1z1 + z 3z1z1 + z 2 z1z 3
Eliminating z from Eqs. (ii) and (iii), we get − z1z 3z 3 + z 2z 3z 3 − z1z 2z 3 } − z ∑ (z1z 2 − z 2 z1 )
(z 2 − z1 ) [ z 2 (z − z 2 ) − z 3 (z − z 3 )] = (z 3 − z 2 ) = − ∑ z12 ( z 2 − z 3 ) − Σ z1z1 (z 2 − z 3 )
[ z1 (z − z1 ) − z 2 (z − z 2 )] 2
∑ z12 (z 2 − z 3 ) + ∑ z1 (z 2 − z 3 )
Hence, z =
or z [z 2 (z 2 − z1 ) − z 3 (z 2 − z1 ) − z1 (z 3 − z 2 ) + z 2 (z 3 − z 2 )] ∑ (z1z 2 − z 2 z1 )
= z 2 z 2 (z 2 − z1 ) − z 3z 3 (z 2 − z1 ) − z1z1(z 3 − z 2 ) + z 2 z 2 (z 3 − z 2 ) 1
99. Let θ = (2n + 1) π , where n = 0, 1, 2, 3, ... , 6
or z ∑ z1 (z 2 − z 3 ) = ∑ z1 z1 (z 2 − z 3 ) 7
∴ 7θ = (2n + 1 ) π or 4 θ = (2n + 1 ) π − 3 θ
∑ | z1 | 2 (z 2 − z 3 ) or cos 4 θ = − cos3 θ
or z=
∑ z1 (z 2 − z 3 ) or 2 cos2 2 θ − 1 = − ( 4 cos3 θ − 3 cos θ )
Chap 01 Complex Numbers 95

or 2 (2 cos2 θ − 1 ) 2 − 1 = − ( 4 cos3 θ − 3 cos θ ) (z 7 + 1 )


⇒ = (z − α ) (z − α ) (z − α 3 ) (z − α 3 ) (z − α 5 ) (z − α 5 )
or 8 cos4 θ + 4 cos3 θ − 8 cos2 θ − 3 cos θ + 1 = 0 (z + 1 )
Now, if cos θ = x, then we have ⇒ z6 − z5 + z4 − z3 + z2 − z + 1
8x 4 + 4x 3 − 8x 2 − 3x + 1 = 0  π  3π 
= z 2 + 1 − 2z cos  z 2 + 1 − 2z cos 
or ( x + 1 ) (8 x − 4 x − 4 x + 1 ) = 0
3 2  7  7
 2 5π 
x+1≠0 [Q θ ≠ π] z + 1 − 2z cos  ...(A)
 7
∴ 8x − 4x − 4x + 1 = 0
3 2
...(i) Dividing by z 3 on both sides, we get
Hence, the roots of this equation are  3 1  2 1  1
z + 3  − z + 2  + z +  − 1
π 3π 5π  z   z   z
cos , cos , cos .
7 7 7  1 π  1 3π   1 5π 
9π 5π 11 π = z + − 2 cos  z + − 2 cos  z + − 2 cos 
[ since cos = cos , cos  z 7  z 7 z 7
7 7 7 1
3π 13π π On putting z + = 2 x, we get
= cos , cos = cos and Eq. (i) is cubic] z
7 7 7
1 1 (8 x 3 − 6 x ) − ( 4 x 2 − 2 ) + 2 x − 1
(i) On putting 2 = y or x = in Eq. (i), then Eq. (i) becomes
x y  π  3π   5π 
= 8  x − cos   x − cos   x − cos 

8
− −
4 4
+1=0  7  7  7
y y y y π

2 or 8 x 3 − 4 x 2 − 4 x + 1 = 8  x − cos 
 4
2
 4  2   7
⇒ 1 −  =  1 −   π π
 y    3   5 
 y y  x − cos   x − cos  …(i)
 7  7
16 8 16  4 4
or 1+ 2 − = 1 + 2 − 
y y y  y y So, 8 x 3 − 4 x 2 − 4 x + 1 = 0 and this equation has roots
π 3π 5π
or y 3 − 24y 2 + 80y − 64 = 0 ...(ii) cos , cos , cos
7 7 7
1 1
where y = 2 = = sec 2 θ π 3π 5π Constant term
x cos2 θ ∴ cos cos cos =−
7 7 7 Coefficient of x 3
Thus, the roots of x 3 − 24 x 2 + 80 x − 61 = 0
π 3π 5π π 3π 5π 1
are sec 2 , sec 2 , sec 2 cos cos cos =− [ proved (i) part]
7 7 7 7 7 7 8
(ii) Again, putting y = 1 + z i. e. z = y − 1 On putting x = 1 in Eq. (i), we get
 π  3π   5π 
= sec 2 θ − 1 = tan 2 θ, Eq. (ii) reduces to 1 = 8 1 − cos  1 − cos  1 − cos 
 7   7   7
(1 + z ) 3 − 24 (1 + z ) 2 + 80 (1 + z ) − 64 = 0
 π 3π 5π 
z 3 − 21z 2 + 35z − 7 = 0 or 1 = 8 8 sin 2 sin 2 sin 2 
or …(iii)  14 14 14 
π 3π 5π Since, sinθ > 0 for 0 < θ < π / 2 , we get
Hence, tan 2 , tan 2 , tan 2 are the roots of
7 7 7 π 3π 5π 1
x 3 − 21 x 2 + 35 x − 7 = 0 ∴ sin sin sin = ...(ii) [ proved (iii) part]
14 14 14 8
1
(iii) Putting x = in Eq. (i), then Eq. (i) reduces to Again, putting x = − 1 in Eq. (i), we get
u
u 3 − 4u 2 − 4u + 8 = 0 whose roots are  π  3π   5π 
− 7 = − 8 1 + cos  1 + cos  1 + cos 
π 3π 5π  7  7 7
sec , sec , sec .
7 7 7  π 3π 5π 
7 = 8 8 cos2 cos2 cos2 
Therefore, sum of the roots is  14 14 14 
π 3π 5π
sec + sec + sec =4 Since, cosθ > 0 for 0 < θ < π / 2 , we get
7 7 7
100. Let roots of z 7 + 1 = 0 are − 1, α , α 3, α 5, α , α 3, α 5, π 3π 5π 7
cos cos cos = ...(iii) [ proved (ii) part]
π π 14 14 14 8
where α = cos + i sin
7 7 On dividing Eq. (ii) by Eq. (iii), we get
∴ (z 7 + 1 ) = (z + 1 ) (z − α ) (z − α ) (z − α 3 ) π 3π 5π 1
tan tan tan = [ proved (iv) part]
(z − α 3 ) (z − α 5 ) (z − α 5) 14 14 14 7
96 Textbook of Algebra

(1 + y ) Again, circles (i) and (iii) should not cut or touch, then distance
On putting z = in Eq. (A), we get
(1 − y ) between their centres > sum of their radii
π 3π 5π
2 6 cos2 cos2 cos2 ( − 2a − 0 ) 2 + (a + 1 − 0 ) 2 > 3 + 3
(1 + y ) 7 + (1 − y ) 7 14 14 14
=
2 (1 − y ) 6 (1 − y ) 6 or 5a 2 + 2a + 1 > 6
 2 2 π 2 2 3π   2 2 5π  ⇒ 5a 2 + 2a + 1 > 36
y + tan  y + tan  y + tan 
 14   14   14  or 5a 2 + 2a − 35 > 0
7  2 2 π 2a
∴ (1 + y ) 7 + (1 − y ) 7 = 2 7 ⋅ y + tan  ⇒ a2 + −7 > 0
64  14 5
 2 2 3π   2 2 5π   − 1 − 4 11   − 1 + 4 11 
y + tan  y + tan  or a −  a −  >0
 14   14 
Using result (ii), we get  5   5 
 π
(1 + y ) 7 + (1 − y ) 7 = 14 y 2 + tan 2   − 1 − 4 11   − 1 + 4 11 
 14 ∴ a ∈  − ∞,  ∪ , ∞ ...(v)
 2 2 3π   2 2 5π 
 5   5 
y + tan  y + tan 
 14   14  Hence, the common values of a satisfying Eqs. (iv) and (v) are
Equating the coefficient of y 4 on both sides, we get  1 − 71 − 1 − 4 11   − 1 + 4 11 1 + 71 
a ∈ ,  ∪ , 
 π 3π 5π   2 5   5 2 
C 4 + 7C 4 = 14 tan 2 + tan 2
7
+ tan 2 
 14 14 14  102. (i) From De-moivre’s theorem, we know that
π 3 π 5 π sin (2n + 1 ) α = 2n + 1
C1 (1 − sin 2 α )n
Therefore, tan 2 + tan 2 + tan 2 =5
14 14 14
sin α − 2n + 1C 3 (1 − sin 2 α )n − 1 sin 3 α
101. Equation z = 3 represents boundary of a circle and equation
+ ... + ( − 1 )n sin 2n + 1 α
z − {a (1 + i ) − i } ≤ 3 represents the interior and the
It follows that the numbers
boundary of a circle and equation z + 2a − (a + 1 ) i > 3
represents the exterior of a circle. Then, any point which π 2π nπ
sin , sin , ..., sin
satisfies all the three conditions will lie on first circle, on or 2n + 1 2n + 1 2n + 1
inside the second circle and outside the third circle. are the roots of the equation.
2n + 1
C1(1 − x 2 )n x − 2n + 1 C 3(1 − x 2 )n − 1 x 3 + ... + ( − 1 )n x 2n + 1
= 0 of the (2n + 1 ) th degree
C B
II Consequently, the numbers
A π 2π nπ
sin 2 , sin 2 , ... , sin 2 are the roots of the
III 2n + 1 2n + 1 2n + 1
I equation
2n + 1
For the existence of such a point first two circles must cut or C1 (1 − x )n − 2n + 1C 3 (1 − x )n − 1 x + ... + ( − 1 )n xn = 0 of
atleast touch each other and first and third circles must not the nth degree
intersect each other. The arcABC of first circle lying inside the (ii) From De-moivre’s theorem, we know that
second but outside the third circle, represents all such possible sin(2n + 1 ) α = 2n + 1C1(cosα ) 2n sin α
points.
2n + 1
Let z = x + iy , then equation of circles are − C 3(cosα ) 2n − 2 sin 3 α + ... + ( −1 )n sin 2n + 1 α
x2 + y 2 = 9 ...(i) or sin (2n + 1 ) α = sin 2n + 1 α
( x − a ) + (y − a + 1 ) 2 = 9
2
...(ii) { 2n + 1C1 cot 2n α − 2n + 1 C 3 cot 2n − 2 α + 2n +1C 5 cot 2n − 4 α − ...}
and ( x + 2a ) + (y − a − 1 ) = 9
2 2
...(iii) π 2π 3π nπ
It follows that α = , , , ... ,
Circles (i) and (ii) should cut or touch, then distance between 2n + 1 2n + 1 2n + 1 2n + 1
their centres ≤ sum of their radii
Therefore, equality holds
⇒ (a − 0 ) 2 + (a − 1 − 0 ) 2 ≤ 3 + 3 2n +1
C1 cot 2n α − 2n +1C 3 cot 2n − 2 α + 2n +1C 5 cot 2n − 4 α−... = 0
⇒ a + (a − 1 ) ≤ 36
2 2
It follows that the numbers
⇒ 2a 2 − 2a − 35 ≤ 0 π 2π nπ
cot 2 , cot 2 , ... , cot 2 are the roots of the
2n + 1 2n + 1 2n + 1
35  1 + 71   1 − 71 
⇒ a2 − a − ≤ 0 or a −  a −  ≤0 equation
2  2   2 
2n + 1 2n + 1
C1xn − C 3x n − 1 + 2n + 1
C 5xn − 2 − ... = 0
1 − 71 1 + 71
∴ ≤a≤ ...(iv)
2 2 of the nth degree.
Chap 01 Complex Numbers 97

103. Let y = | a + bω + cω 2 |. For y to be minimum, y 2 must be


z i
minimum. 108. = 1 ⇒ |z | = z −
i 3
∴ y 2 = | a + bω + cω 2 | 2 = (a + bω + cω 2 ) (a + bω + cω 2 ) z−
3
= (a + bω + cω 2 ) (a + b ω + c ω 2 )
Clearly, locus of z is perpendicular bisector of line joining
y = (a + bω + cω 2 ) (a + bω 2 + cω )3
2
i
points having complex number 0 + i 0 and 0 + .
= (a 2 + b 2 + c 2 − ab − bc − ca ) 3
Hence, z lies on a straight line.
1
= [(a − b ) 2 + (b − c ) 2 + (c − a ) 2 ]  ω − ωz 
2 109. Given,   is purely real ⇒ z ≠ 1
 1 −z 
Since, a, b and c are not equal at a time, so minimum value of
y 2 occurs when any two are same and third is differ by 1.  ω − ω z   ω − ω z  ω − ωz
∴   = =
⇒ Minimum of y = 1 (as a, b, c are integers)  1 −z   1 −z  1 −z
π ⇒ ( ω − ω z ) ( 1 − z ) = (1 − z ) ( ω − ω z )
104. Equation of ray PQ is arg (z + 1) =
4 ⇒ ( zz − 1 ) ( ω − ω) = 0
π ⇒ (| z | 2 − 1 ) (2iβ ) = 0 [Q ω = α + iβ ]
Equation of ray PR is arg (z + 1 ) = −
4 ∴ |z | − 1 = 0
2
π π π
Shaded region is − < arg (z + 1 ) < ⇒ | arg (z + 1 )| < ⇒ | z | = 1 and z ≠ 1 [Q β ≠ 0 ]
4 4 4 10
2kπ   2kπ 
∴ | PQ | = ( 2 ) 2 + ( 2 ) 2 = 2 110. ∑ sin   + i cos  
k =1
 11   11 
So, arc QAR is of a circle of radius 2 units with centre at −2k π i
 2kπ   2kπ  
10 10
P ( − 1, 0 ). All the points in the shaded region are exterior to  11
= i ∑ cos   − i sin   =i ∑ e
this circle | z + 1 | = 2. k =1 
 11   
11  k =1
π
i.e. | z + 1 | > 2 and | arg (z + 1 )| < .  10 −2k π i 
4  11 
= i  ∑e − 1 = i ( 0 − 1 ) [Qsum of 11, 11th roots of unity = 0]
zB − 1 k = 0 
105. In ∆AOB from Coni method, = e iπ / 2 = i  
zA − 1
= −i
B(zB) A(2 + √3i ) = zA 111. Q z 2+ z + 1 = 0
π/2 √2 ∴ z = ω, ω 2
√2
(1) 1 1
√2 √2 Q z+
= ω + = ω + ω2 = − 1
z ω
D (zD) 1 1
(zC)C ⇒ z 2 + 2 = ω2 + 2 = ω2 + ω = − 1
z ω
1 1
z B − 1 = (z A − 1 ) i ∴ z + 3 =ω + 3 =1 + 1 =2
3 3

∴ z B = 1 + (2 + 3i − 1 ) i = 1 + (1 + i 3 ) i z ω
=1 + i − 3 =1 − 3 + i 1 1 1
z + 4 =ω + 4 =ω + = −1
4 4
z ω ω
zC = 2 − z A = 2 − (2 + 3i ) = − 3 i
1 1
and z D = 2 − z B = 2 − (1 − 3 + i ) = 1 + 3 − i z 5 + 5 = ω5 + 5 = ω2 + ω = − 1
z ω
Hence, other vertices are (1 − 3 ) + i , − 3i , (1 + 3 ) − i. 1 1
and z + 6 = ω + 6 =2
6 6

106. Let z1 = r1 (cos θ1 + i sin θ1 ) and z 2 = r2 (cos θ 2 + i sin θ 2 ) z ω


∴ Required sum = ( −1 ) 2+ ( −1 ) 2 + (2 ) 2 + ( −1 ) 2 + ( −1 ) 2 + (2 ) 2 = 12
∴| z1 + z 2 | = [(r1 cos θ1 + r2 cos θ 2 ) 2 + (r1 sin θ1 + r2 sin θ 2 ) 2 ]1/2
112. Let OA = 3, so that the complex number associated with A is
= [| r12 + r22 + 2r1r2 cos ( θ1 − θ 2 )]1/2 = [(r1 + r2 ) 2 ]1/2
3e iπ /4 . If z is the complex number associated with P, then
∴ | z1 + z 2 | = | z1 | + | z 2 |
N (North)
Therefore, cos ( θ1 − θ 2 ) = 1
z
⇒ θ1 − θ 2 = 0
⇒ θ1 = θ 2 4

Thus, arg ( z1 ) – arg ( z 2 ) = 0 π/2 A 3ei π/4


107. ( x − 1) 3 = − 8 ⇒ x − 1 = ( − 8)1/3 3
⇒ x − 1 = − 2, − 2ω, − 2ω 2 π/4
W E
O (East)
⇒ x = − 1, 1 − 2ω, 1 − 2ω 2
S
98 Textbook of Algebra

z − 3e iπ /4 4 − π /2 4i 120. Q z z (z 2 + z 2 ) = 350
iπ / 4
= e =−
0 − 3e 3 3 Put z = x + iy
⇒ 3z − 9e iπ /4 = 12 ie iπ /4 ⇒ z = (3 + 4i ) e iπ /4 ⇒ ( x 2 + y 2 ) ⋅ 2( x 2 − y 2 ) = 350
113. Let z = cos θ + i sin θ ⇒ ( x 2 + y 2 ) ( x 2 − y 2 ) = 175 = 25 × 7
z cos θ + i sin θ ⇒ x 2 + y 2 = 25, x 2 − y 2 = 7
⇒ =
1 −z 2
1 − (cos 2 θ + i sin 2 θ ) ⇒ x 2 = 16 , y 2 = 9
cos θ + i sin θ ∴ x = ± 4, y = ± 3; x , y ∈ I
=
2 sin 2 θ − 2 i sin θ cos θ Area of rectangle = 8 × 6 = 48 sq units
15 15 15
cos θ + i sin θ i
= =
− 2 i sin θ (cos θ + i sin θ ) 2 sin θ
121. ∑ Im (z 2m − 1 ) = ∑ Im [e ( 2m − 1) iθ ] = ∑ sin (2m − 1) θ
m =1 m =1 m =1
z 15
2 sin (2m − 1 ) θ sin θ
lies on the imaginary axis i.e. x = 0 or on Y-axis.
Hence,
1 − z2 = ∑ 2 sin θ
m =1
Aliter
z z 1 1 1
15
cos (2m − 2 ) θ − cos 2mθ
Let E = =
1 − z2 zz − z2 z − z
= =−
z −z
=−
z − z 
= ∑ 2 sin θ
  2i m =1
 2i 
i cos 0 ° − cos 30 θ 1 − cos 60 °
= which is imaginary. = = (Qθ = 2 ° )
2 Im | z | 2 sin θ 2 sin 2 °
114. | z + 4 | ≤ 3 1
1−
2 = 1
⇒z lies inside or on the circle of radius 3 and centre at ( − 4, 0 ). =
2 sin 2 ° 4 sin 2 °
∴ Maximum value of | z + 1 | is 6.
Y 4 4 4
122. z − ≥ |z | − ⇒ 2 ≥ |z | −
z |z | |z |
4
⇒ − 2 ≤ |z | − ≤ 2 ⇒ − 2 |z | ≤ |z |2 − 4 ≤ 2 |z |
X′ O
X |z |
(–4, 0) A (0, 0)
(–1,0) ⇒ |z |2 + 2 |z | − 4 ≥ 0
and 12 − 2 |z | − 4 ≤ 0
Y′
⇒ (| z | + 1 ) 2 ≥ 5 and (| z | − 1 ) 2 ≤ 5
115. Let A = set of points on and above the line y = 1 in the argand − 5 ≤ | z | − 1 ≤ 5 and | z | + 1 ≥ 5
plane.
⇒ 5 − 1 ≤ |z | ≤ 5 + 1
B = set of points on the circle ( x − 2 ) 2 + (y − 1 ) 2 = 3 2
123. As z = (1 − t ) z1 + tz 2
C = Re (1 − i ) z = Re [(1 − i ) ( x + iy )] = x + y
z
⇒ x+y = 2
z1 t 1–t z2
Hence, ( A ∩ B ∩ C ) has only one point of intersection.
⇒ z1, z and z 2 are collinear.
116. The points ( − 1 + i ) and (5 + i ) are the extremities of diameter
Thus, options (a) and (d) are correct.
of the given circle.
z − z1 z − z1
Hence, | z + 1 − i | 2 + | z − 5 − i | 2 = 36 Also, =
z 2 − z1 z 2 − z1
117. Q | z − w | ≤ | z | − | w | Hence, option (c) is correct.
and | z − w | = distance between z and w 2π 2π 1 3
124. ω = cos + i sin =− +i
z is fixed, hence distance between z and w would be maximum 3 3 2 2
for diametrically opposite points. ω is one of the cube root of unity.
⇒ | z − w | < 6 ⇒ || z | − | w || < 6
z +1 ω ω2
⇒ − 6 < |z | − |w | < 6 ⇒ − 3 < |z | − |w | + 3 < 9
∴ ω z +ω 2
1 =0
118. Q z 0 = 1 + 2i
ω 2
1 z +ω
∴ z1 = 6 + 5i ⇒ z 2 = − 6 + 7i
Applying R 1 → R 1 + R 2 + R 3, we get
119. Put ( − i ) in place of i.
z z z
−1
Hence, ω z + ω2 1 =0 [Q 1 + ω + ω 2 = 0 ]
i +1
ω 2
1 z +ω
Chap 01 Complex Numbers 99

Now, applying C 2 → C 2 − C1, C 3 → C 3 − C1, we get (D) Let w = cos θ + i sin θ, then
z 0 0 1
z = x + iy = w +
ω z + ω2 − ω 1 −ω =0 w
⇒ x + iy = 2 cos θ
ω 2
1 −w 2
z + ω −ω 2
∴ x = 2 cos θ and y = 0
⇒ z [(z + ω − ω ) (z + ω − ω ) − (1 − ω ) (1 − ω )] = 0
2 2 2
126. Q x2 − x + 1 = 0
⇒ z [z 2 − (ω 2 − ω ) 2 − (1 − ω 2 − ω + ω 3 )] = 0
⇒ z [z 2 − (ω 4 + ω 2 − 2ω 3 ) − 1 + ω 2 + ω − ω 3 ] = 0 1 ± (1 − 4 ) 1 ± i 3
∴ x= =
⇒ z3 = 0 2 2
1+i 3 1 −i 3
∴ z =0 = and
2 2
125. | z − i | z || = | z + i | z ||
∴ x = −ω , −ω
2

(A) Putting z = x + iy , we get y x 2 + y 2 = 0


∴ α = − ω 2, β = − ω
i. e. Im(z ) = 0
4 ⇒ α 2009
+β 2009
= − ω 4018 − ω 2009
(B) 2ae = 8, 2a = 10 ⇒ 10e = 8 ⇒e =
5 = − ω − ω 2 = − (ω + ω 2 )
(0, 3) = − (− 1) = 1
127. | z − 1 | = | z + 1 | = | z − i |
(0,0)
(–5, 0) (5, 0) ⇒ |z − 1|2 = |z + 1|2 = |z − i |2
⇒ (z − 1 ) ( z − 1 ) = (z + 1 ) ( z + 1 ) = (z − i ) ( z + i )
(0, – 3) ⇒ z z − z − z + 1 = z z + z + z + 1 = z z + iz − i z + 1
⇒ − z − z = z + z = i (z − z )
 16
∴ b 2 = 25 1 −  = 9 From first two relations,
 25
2 (z + z ) = 0 ⇒ Re(z ) = 0 …(i)
x2 y 2
⇒ + =1 From last two relations,
25 9 z + z = i (z − z ) ⇒ 2 Re (z ) = − 2 Im (z )
1
(C) z = 2 (cos θ + i sin θ ) − From Eq. (i), Im(z ) = 0
2 (cos θ + i sin θ ) ∴ z = Re(z ) + i Im(z ) = 0 + i ⋅ 0 = 0
1
= 2 (cos θ + i sin θ ) − (cos θ − i sin θ ) Hence, number of solutions is one.
2
128. We have, | z − 3 − 2i | ≤ 2
3 5
z = cos θ + i sin θ ⇒ | 2z − 6 − 4i | ≤ 4 …(i)
2 2
Now, | 2z − 6 − 4i | = |(2z − 6 + 5i ) − 9i |
≥ || 2z − 6 + 5i | − 9 | …(ii)
(0, 0)
From Eqs. (i) and (ii), we get
3 3, 0
– ,0
2 2 | 2z − 6 + 5i | − 9 | ≤ 4
⇒ − 4 ≤ | 2z − 6 + 5i | − 9 ≤ 4
⇒ 5 ≤ | 2z − 6 + 5i | ≤ 13
Hence, the minimum value of | 2z − 6 + 5i | is 5.
Let z = x + iy , then 129. Q | z | = 1 ∴ z = e iθ
3 5
x = cos θ and y = sin θ  2iz   2ie iθ   2i 
2 2 ∴ Re  2
= Re  2 iθ
 = Re  − iθ iθ 
2 2 1 − z  1 − e  e −e 
 2x   2y 
⇒   +   =1  2i   1 
3 5 = Re   = Re  − 
 − 2i sin θ   sin θ 
4 x 2 4y 2
⇒ + =1 1
9 25 =− = − cosec θ
sin θ
x2 y2
⇒ + =1 Q cosec θ ≤ − 1 ⇒ cosec θ ≥ 1
9 / 4 25 / 4
⇒ − cosec θ ≥ 1 ⇒ − cosec θ ≤ − 1

9 25
= (1 − e 2 ) ⇒ − cosec θ ∈ ( − ∞, − 1 ] ∩ [1, ∞ )
4 4  2iz 
9 16 4 ∴ Re  2
∈ ( − ∞, −1 ] ∩ [1, ∞ )
∴ e2 = 1 − = ⇒ e= 1 − z 
25 25 5
100 Textbook of Algebra

130. Q | z | = 1. Let z = e i θ 134. Given, z 2 + z + 1 = a ⇒ z 2 + z + 1 − a = 0


∴ z − 1 = e iθ − 1 = e iθ /2 ⋅ 2i sin (θ / 2 ) − 1 ± ( 4a − 3 )
− iθ / 2
∴ z=
1 1 ie 2
⇒ = =−
z − 1 2ie iθ /2 ⋅ sin (θ / 2 ) 2 sin (θ / 2 ) Hence, a≠
3
[for a = 3 / 4, z will be purely real]
4
1 i ⋅ e − i θ /2  1   π θ
⇒ = ∴ arg   = −  135. Let z = x + iy , then
1 − z 2 sin (θ / 2 )  1 − z   2 2
 1  π θ z2 ( x + iy ) 2 ( x 2 − y 2 + 2ix )
⇒ = =
arg   = − z − 1 ( x + iy − 1 ) ( x − 1 + iy )
1 − z 2 2
 1  π ( x 2 − y 2 + 2ixy ) ( x − 1 − iy )
∴ Maximum value of arg   = =
1 − z 2 ( x − 1 + iy ) ( x − 1 − iy )

131. Q | x | 2 = x x = (a + b + c ) (a + b + c ) ( x − 1 ) ( x 2 − y 2 ) + 2 xy 2 + i [ 2 xy ( x − 1 ) − y ( x 2 − y 2 )]
=
= (a + b + c ) ( a + b + c ) (x − 1)2 + y 2
= | a | 2 + | b | 2 + | c | 2 + ab + a b + bc + b c + ca + ca …(i)  z2 
Now, Im   =0
| y | 2 = y y = (a + bω + cω 2 ) (a + b ω + c ω 2 )  z − 1
= (a + bω + cω 2 ) ( a + b ω + c ω 2 ) ⇒ 2 xy ( x − 1 ) − y ( x 2 − y 2 ) = 0
⇒ y (2 x 2 − 2 x − x 2 + y 2 ) = 0
= (a + bω + cω ) ( a + b ω + c ω )
2 2
⇒ y (x 2 + y 2 − 2x ) = 0
= | a | 2 + | b | 2 + | c | 2 + ab ω 2 + a bω
+ bc ω 2+ b cω + ca ω 2+ caω …(ii) ⇒ y = 0 or x 2 + y 2 − 2 x = 0
Hence, z lies on the real axis or on a circle passing through the
and| z | 2 = zz = (a + bω 2 + cω ) (a + bω 2 + cω ) origin.
= (a + bω 2+ cω ) (a + b ω 2 + c ω )
136. Given, | z | = 1 and arg (z ) = θ …(i)
= (a + bω 2 + cω ) (a + b ω + c ω 2 ) ⇒ |z | = 1 ⇒ z z = 1
2

= | a | 2 + | b | 2 + | c | 2 + ab ω + a bω 2 1
⇒ z = …(ii)
z
+ bcw + b cω 2 + ca ω + caω 2 …(iii) 1 + z  1+z 
∴ arg   = arg   [from Eq. (ii)]
On adding Eqs. (i), (ii) and (iii), we get  1 + z   1 + 1 /z 
| x | 2 + | y | 2 + | z | 2 = 3 (| a | 2 + | b | 2 + | c | 2 ) = arg (z ) = θ [from Eq. (i)]
+ 0 + 0 + 0 + 0 + 0 + 0 (Q 1 + ω + ω 2 = 0 ) Aliter I
Given, | z | = 1 and arg (z ) = θ
| x |2 + |y |2 + |z |2
∴ =3 ⇒ z = eiθ
|a |2 + |b |2 + |c |2  1 + eiθ 
1 + z
z +z ∴ arg   = arg  − iθ
 = arg (e i θ ) = arg (z ) = θ
132. Q Re(z ) = 1 ∴ =1 ⇒ z + z =2 1 + z  1 + e 
2
Aliter II Given, | z | = 1 and arg (z ) = θ
Since, α , β ∈ R
Let z = ω (cube root of unity)
∴ The complex roots are conjugate to each other, if z1, z 2 are
two distinct roots, then z1 = z 2 or z1 = z 2 1 + z  1 + ω 1+ω
∴ arg   = arg   = arg  2
(Q ω = ω 2 )
∴ Product of the roots = z1z 2 = β 1 + z  1 + ω 1 + ω 
⇒ z1z1 = β  − ω2 
= arg   (Q 1 + ω + ω 2 = 0 )
∴ β = | z1 | = [ Re(z1 )] + Im | z1 |
2 2 2  −ω 
= arg (ω ) = arg (z ) = θ
= 1 + Im | z1 | 2 > 1
1
[Q roots are distinct∴ Im (z1 ) ≠ 0] 137. z 0 = 2α −
α
∴ β > 1 or β ∈ (1, ∞ )
Q 2 |z 0 |2 = r 2 + 2
133. Q (1 + ω ) 7 = ( − ω 2 ) 7 = − ω14 = − ω 2 = 1 + ω 2 2 2
1 1 1
Given, (1 + ω ) = A + Bω ⇒ 1 + ω = A + Bω
7 ∴ 2 2α − = r 2 + 2 ⇒ 2 2α − = α− +2
α α α
On comparing, we get A = 1, B = 1 1 1 1
⇒ 7 |α |2 + − 8 = 0 ⇒ | α | 2 = 1 or ⇒ | α | = 1 or
∴ ( A, B ) = (1, 1 ) |α |2 7 7
Chap 01 Complex Numbers 101

138. (B) z1 ⋅ z = zk is clearly incorrect.


z 10 − 1
ei5π/6 e iπ/6 lim
z →1 z − 1
(C) Expression = =1
10
–1 1
9
 2kπ 
(D) 1 + Σzk = 0 ⇒ 1 + ∑ cos   =0
e–i 5π/6 e– i π/6 k =1  10 
∴ Expression = 2
1/2 1/2
z1 − 2z 2
143. Q =1
3+i 2 − z1z 2
ω= = e iπ /6, P = e inπ /6
2
As z1 ∈ P ∩ H1 ⇒ z1 = 1, e iπ /6, e − iπ /6 ⇒ | z1 − 2z 2 | 2 = | 2 − z1z 2 | 2

As z 2 ∈ P ∩ H 2 ⇒ z 2 = − 1, ei 5π /6, e − i 5π /6 ⇒ (z1 − 2z 2 ) (z1 − 2z 2 ) = (2 − z1z 2 ) (2 − z1z 2 )


iπ / 6 i 5π / 6 ⇒ (z1 − 2z 2 ) (z1 − 2z 2 ) = (2 − z1z 2 ) (2 − z1z 2 )
∠z1Oz 2 = 2 π / 3, where z1 = e , z2 = e
Sol. (Q. Nos. 139-140) ⇒ z1z1 − 2z1z 2 − 2z1z 2 + 4z 2z 2 = 4 − 2z1z 2 − 2z1z 2 + z1z1z 2z 2
Let z = x + iy , S1 : x + y < 16
2 2
⇒ | z 1| 2 + 4 | z 2 | 2 + 4 + | z 1 | 2 | z 2 | 2
( x − 1 ) + i (y + 3 )  ⇒ ( | z1 | 2 − 4 ) (1 − | z 2 | 2 ) = 0
Now, Im  >0
 1 − 3i  Q | z2 | ≠ 1
⇒ S 2 : 3x + y > 0 ⇒ S 3 : x > 0 ∴ | z1 | = 4 or | z1 | = 2
2

⇒ Point z1 lies on circle of radius 2.


√3x + y = 0 (0, 4)
144. Let a = 3, b = −3, c = 2, then
S (a + bω + cω 2 ) 4n + 3 + (c + aω + bω 2 ) 4n + 3 + (b + cω + aω 2 ) 4n + 3
=0
0 60° (4, 0) ⇒(a + bω + cω 2 ) 4n + 3
P
(1, –3)   c + aω + bω 2 ) 4n + 3  4n + 3  b + cω + aω 2 ) 4n + 3 
1 +   +  =0
  a + bω + cω  a + bω + cω 
2 2
 
139. min | 1 − 3i − z | = min | z − 1 + 3i | ⇒ (a + bω + cω 2 ) 4n + 3(1 + ω 4n + 3 + (ω 2 ) 4n + 3 ) = 0
= perpendicular distance of the point (1, − 3 ) from the straight ⇒ 4n + 3 should be an integer other than multiple of 3.
3 −3 3 − 3
line 3 x + y = 0 = = ∴ n = 1, 2, 4, 5
2 2
kπ kπ
145. Q α k = cos  + i sin   = e iπk /7
20 π
140. Area of S =   π × 4 2 +   π × 4 2 =  7  7
1 1
 4  6 3 ∴ α k + 1 − α k = e iπ (k + 1) /7− e iπk /7 = e iπk /7(e iπ /7− 1 )
141. Since, | z | ≥ 2 is the region lying on or outside circle centered at
 π
( 0, 0 ) and radius 2. Therefore, | z + (1 / 2 )| is the distance of z = e iπk /7⋅ e iπ /14⋅ 2 i sin  
 14
from ( − 1 / 2, 0 ), which lies inside the circle.
Hence, minimum value of | z + (1 / 2 )|  π
⇒ | α k + 1 − α k | = 2 sin  
 14
= distance of ( − 1 / 2, 0 ) from ( − 2, 0 )
 π  π
12

 1 
2
∑ |αk + 1 − αk | = 12 × 2 sin  14 = 24 sin  14
=  − + 2 + | 0 − 0 | 2 = 3 / 2 k =1
 2 
Aliter and α 4k − 1 − α 4k − 2 = e iπ ( 4k − 1) /7− e iπ ( 4k − 2 ) /7= e iπ ( 4k − 2) /7(e iπ /7− 1 )
1 1
Q | z + (1 / 2 )| ≥ | z | − ≥ 2 − [Q | z | ≥ 2 ]  π
2 2 = e iπ ( 4k − 2) /7⋅e iπ /14 ⋅ 2i sin  
 14
∴ | z + (1 / 2 )| ≥ 3 / 2  π
⇒ | α 4k − 1 − α 4k − 2| = 2 sin  
142. Clearly, zk10 = 1, ∀ k, where zk ≠ 1  14

 π  π
3
(A) zk ⋅ z j = e i ( 2 π /10) (k + j)
= 1, if (k + j ) is multiple of 10 ∴ ∑ | α 4k −1 − α 4k − 2| = 3 × 2 sin  14 = 6 sin  14
i.e. possible for each k. k =1
102 Textbook of Algebra

12 x
x2 + y 2 − =0
∑ |αk + 1 − αk | or
a
k =1
Hence, =4 1 
3 ∴ Locus of z is a circle with centre  , 0
 2a 
∑ | α 4k − 1 − α 4k − 2| 1
k =1
and radius = , a > 0.
2 + 3i sin θ 2a
146. Let z =
1 − 2i sin θ Also for b = 0, a ≠ 0, we get y = 0.
Q z is purely imaginary ∴ locus is X -axis and for a = 0, b ≠ 0, we get x = 0
∴ locus is Y -axis.
∴ z = −z
 2 + 3i sin θ   2 + 3i sin θ  1 1 1 1 1 1
⇒   =− 
 1 − 2i sin θ   1 − 2i sin θ  148. Let ∆ = 1 −ω − 1 ω = 1 ω ω 2
2 2

1 ω2 ω7 1 ω2 ω
 2 − 3i sin θ   2 + 3i sin θ 
⇒   =− 
 1 + 2i sin θ   1 − 2i sin θ  (Q1 + ω + ω 2 = 0 and ω 3 = 1)
⇒ (2 − 3i sin θ ) (1 − 2i sin θ ) + (1 + 2i sin θ ) (2 + 3i sin θ ) = 0 Applying C1 → C1 + C 2 + C 3, then we get
1
⇒ 4 − 12 sin 2 θ = 0 or sin 2 θ = 3 L 1 L 1
3
M
−1  1 
∴ θ = sin   ∆= 0 ω ω2 (Q 1 + ω + ω 2 = 0)
 3
M
1
147. Q x + iy = 0 ω2 ω
a + ibt
a − ibt = 3 (ω − ω )
2 4
⇒ x + iy = 2
a + b 2t 2 = 3 (− 1 − ω − ω ) (Q ω 3 = 1 and 1 + ω + ω 2 = 0)
a bt
⇒ x= 2 , y =− 2 = − 3 (1 + 2ω )
(a + b t )
2 2
(a + b 2t 2 )
1 x = − 3z = 3k (given) (Q 1 + 2ω = z)
or x2 + y 2 = 2 = ∴ k = −z
a + b 2t 2 a

You might also like